SóProvas



Questões de Probabilidade


ID
6487
Banca
ESAF
Órgão
MTE
Ano
2006
Provas
Disciplina
Raciocínio Lógico
Assuntos

Beatriz, que é muito rica, possui cinco sobrinhos: Pedro, Sérgio, Teodoro, Carlos e Quintino. Preocupada com a herança que deixará para seus familiares, Beatriz resolveu sortear, entre seus cinco sobrinhos, três casas. A probabilidade de que Pedro e Sérgio, ambos, estejam entre os sorteados, ou que Teodoro e Quintino, ambos, estejam entre os sorteados é igual a:

Alternativas
Comentários
  • Se são 3 casas para 5 sobrinhos, é so dividir 3/5
    3/5 = 0,6
  • Não entendi, me ajudem.

    3 casas;
    5 sobrinhos;
    3/5 é a chance de cada sobrinho;
    3/5 x 3/5 é a chance de pedro e sérgio = 9/25
    também é a chance de teodoro e quintino, como é "ou" se soma
    9/25 + 9/25 = 18/25 = 0,75 alternativa E.
  • Probabilidade de Pedro e Sérgio: 3/5*2/4 = 6/20
    Probabilidade de Teodoro e Quintino: 3/5*2/4 = 6/20

    Probabilidade de Pedro e Sérgio ou Teodoro e Quintino: 6/20 + 6/20 = 12/20 = 6/10.

    letra "d" (6/10 = 0,6)
  • Pessoal, sem churumelas vc resolve essa questão com combinação:Fixando Pedro e Sérgio-> temos 1 escolha dentre 3 pessoas restantes-> C(3,1) = 3Fazendo a mesma coisa para Quintino e Carlos -> C(3,1) = 3Número total de combinações possíveis = C(5,3) = 10Logo, como pode ocorrer tanto fixando Pedro e Sérgio como fixando Quintino e Carlos temos a soma das duas possibilidades = 6Probabilidade -> 6/10 = 60%
  • Clareando aos colegas:
    Primeiro Sorteio com Pedro e Sérgio felizardos: Pedro, Sérgio, 3,1


    + = OU

    Segundo Sorteio com Teodoro e Quintino felizardos: Teodoro, Quintino, 3,1

    Total de possibilidades sem Pedro e Sérgio juntos ou Teodoro e Quintino juntos: 5,3 = 10

    Resultados Favoráveis/Resultados Possíveis
    3+3= 6/10

    Resposta: 0,6

    Abraços!
  • As possibilidades são:

     1 - P S T - Favorável                                                                                                                                                                                          2 - P S C - Favorável                                                                                                                                                                                        3 - P S Q - Favorável                                                                                                                                                                                      4 - P T C - NÃO                                                                                                                                                                                                5 - P T Q - Favorável                                                                                                                                                                                        6 - P C Q - NÃO                                                                                                                                                                                                7 - S T C - NÃO                                                                                                                                                                                                  8 - S T Q - Favorável                                                                                                                                                                                        9 - S C Q - NÃO                                                                                                                                                                                              10 - T C Q - Favorável
                                                                                                                                                                                                                             Ou seja, temos 10 possibilidades com iguais chances de ocorrerem, sendo que em 6 delas temos condições favoráveis (P e S ou T e Q), sendo assim 6 de 10 ou 60% ou 0,6
  • muito fácil. por favor...

  • Opa olá boa tarde a resposta é letra D.

    Explicação:

    Seria 2/3 x 2/3 = 4/6 simplificando daria 2/3.

    Dividindo 2 por 3 daria 0,666...

    OBS: Eu coloquei 2/3 x 2/3 por causa do contexto que disse que Pedro e Sérgio poderiam estar entre os sorteados que é o caso do 2/3 já que ele disse depois "ambos" . Que 3 seria as 3 casas.


ID
10717
Banca
ESAF
Órgão
ANEEL
Ano
2004
Provas
Disciplina
Raciocínio Lógico
Assuntos

Todos os alunos de uma escola estão matriculados no curso de Matemática e no curso de História. Do total dos alunos da escola, 6% têm sérias dificuldades em Matemática e 4% têm sérias dificuldades em História. Ainda com referência ao total dos alunos da escola, 1% tem sérias dificuldades em Matemática e em História. Você conhece, ao acaso, um dos alunos desta escola, que lhe diz estar tendo sérias dificuldades em História. Então, a probabilidade de que este aluno esteja tendo sérias dificuldades também em Matemática é, em termos percentuais, igual a

Alternativas
Comentários
  • Use probabilidade condicional P(M/H)=P(M&H)/P(H)
  • faremos a hipótese de serem 100 alunos. dos quais 5 tem dificuldade em matemática e 3 em história e outro que tem dificuladade tanto em Matemática quanto em história,ou seja,6 com dificuldade em matemática no total e 4 com dificuldade em história no total.
    ele afirma que conheceu um menino com dificuldade em história,ou seja, apenas 4. desses 4 meninos que tem dificuldade em historia 1 também tem em matemática .Logo, 1 de 4 ou 1/4 totalizando em % = 25%.
  • Pensei assim:

    A porcentagem dos que têm dificuldade em história é 4% se esse aluno tb tem dificuldade em matemática ele representa 1% desse conjunto então 25%.


    Bons estudos sempre!!! 

  • Bom eu peguei a afirmação do aluno que diz "estar tendo sérias dificuldades em História",
    então ele estaria dentro dos 4% com dificuldade em História.
    Se pegar-mos 100% dos alunos matriculados nesta escola e dividir-mos pelos 4% 
    encontraremos 25%.
  • Segui o mesmo raciocínio do Pedro Henrique.

  • pedro enrique está correto. a msm afirmação

  • Eu fiz, 6×4+1


  • Fácil visualizar usando o Diagrama de Venn...

     

  • Probabiblidade Condicional:

    P (A cond B) = P (A e B) / P (B)

    P (A cond B) = 1% / 4% = 25%


ID
10735
Banca
ESAF
Órgão
ANEEL
Ano
2004
Provas
Disciplina
Raciocínio Lógico
Assuntos

Ana é enfermeira de um grande hospital e aguarda com ansiedade o nascimento de três bebês. Ela sabe que a probabilidade de nascer um menino é igual à probabilidade de nascer uma menina. Além disso, Ana sabe que os eventos "nascimento de menino" e "nascimento de menina" são eventos independentes. Deste modo, a probabilidade de que os três bebês sejam do mesmo sexo é igual a

Alternativas
Comentários
  • seja
    P=a probabilidade de nascer do mesmo sexo
    Ph=probabilidade de nascer menino
    Pm=probabilidade de nascer menina
    entao
    P=ph+pm
    ph=pm=1/2*1/2*1/2 = 1/8
    Assim, p=1/8+1/8 = 1/4
  • Tanto faz nascerem 3 homens ou 3 mulheres, contanto que os 3 sejam do mesmo sexo. Portanto, há 3 eventos independente (os 3 nascimento) e dois eventos mutuamente excludente (os 3 serem meninos ou os 3 serem meninas)

    P(mesmo sexo) = {P(H) e P(H) e P(H)} OU {P(M) e P(M) e P(M)}

    em "linguagem de probabilidade"

    P = P(H).P(H).P(H) + P(M).P(M).P(M)
    P = 1/2.1/2.1/2 + 1/2.1/2.1/2
    P = 1/8 + 1/8
    P = 2/8
    P = 1/4

    Alternativa D
  • Para quem tem dificuldade de pensar matemáticamente, como a questão é pequena, poderia ser feito pelo diagrama de árvore ou desenho.
    Le-se H = homem  M= mulher
    H-H-H
    H-H-M
    H-M-H
    H-M-M
    M-M-M
    M-M-H
    M-H-M
    M-H-H

    ASSIM, TEMOS 2 POSSIBILIDADES EM UM UNIVERSO AMOSTRAL DE 8.
    2/8 = 1/4
  • Probabilidade de nascer 3 meninos + Probabilidade de nascer 3 meninas =

    2*C (3,3) 0.5^3 = 1/4


  • A probabilidade de nascer homem é 50% o mesmo (1/2)

    A Probabilidade de nascer mulher é também 50 % (1/2)

    Logo, como a questão quer saber qual  a probabilidade de nascer três bebês do mesmo sexo podemos somar as três possibilidades de ser homem ( 1/2+1/2+1,2= 3/6). Fazendo esse mesmo cálculo em relação a possibilidade de ser mulher também teremos 3/6, no entanto é só multiplicar 3/6 * 3/6= 9/36, simplificando esse último valor (9/36) por 6 vai nos restar o valor 1/4. Gabarito D.

     

     


ID
10876
Banca
ESAF
Órgão
ANEEL
Ano
2006
Provas
Disciplina
Raciocínio Lógico
Assuntos

Uma empresa possui 200 funcionários dos quais 40% possuem plano de saúde, e 60 % são homens. Sabe-se que 25% das mulheres que trabalham nesta empresa possuem planos de saúde. Selecionando-se, aleatoriamente, um funcionário desta empresa, a probabilidade de que seja mulher e possua plano de saúde é igual a:

Alternativas
Comentários
  • Resultado = 40 % (Possuem Plano) x 25% (Mulheres que possuem Plano) = 1/10
  • Total = 200
    40% tem plano de saúde sendo assim são 80 pessoas
    60% são homens sendo assim são 120.
    São 80 mulheres, 25% delas tem plano de saúde, sendo assim são 20.
    sobrando 60 dos 80 com plano de saúde para serem homens, sendo metade do total de homens.
    a probabilidade de que seja mulher e possua plano de saúde seria dividir o total de mulheres com plano de saúde pelo total de pessoas da empresa.
    20/200 = 1/10
    resposta letra "a"
  • simples:
    60% homens, então 40% mulheres;
    25% das mulheres com plano de saúde;
    0,40 x 0,25 = 0,10 (10% ou 1/10) são mulheres com plano de saúde em relação ao total dos funcionário.
    OBS: 200 funcionário e 60% possuem plano de saúde são informação desnecessária.
  • Funcionários = 200
    Com plano de saúde = 80 (40% de 200)
    Homens = 120 (60% de 200)
    Mulheres= 200-120 = 80
    Mulheres com plano = 20 (25% de 80)

    Regra geral da probabilidade:
    "O que se quer sobre tudo"

    20/200 = 1/10
  • Informações:
    40% de 200 funcionários tem plano de saúde = 80 tem plano
    60% homens = 60% x 200 = 120 homens
    25% da mulheres tem plano
    Colocando a ordem de preenchimento do quadro:


                                        Homem                                                                                                                   Mulher
    Plano de saúde   
    5)60 (pois 40% dos func tem plano de saúde = 80-20 das mulheres)              3) 20 (25% de 80 mulheres com plano)
    Sem plano            6) 60 (120-60                                                                                                    4) 60 (total de mulheres - as com plano)
    Total                     1) 120 (60% homem)                                                                                        2) 80 (pois 200-120)
    Probabilidade que seja mulher = 80 em 200 (80 mulheres do total de funcionários)
    Probabilidade que tenha plano de saúde e seja mulher = 20 em 80 mulheres
    Como os eventos dever acontecer ao mesmo tempo (ser mulher E ter plano de saúde deve-se multiplicar)
    80/200 x 20/80 =  16/160 = 1/10



  • Desculpe, mas no meu comentário eu fiz um quadro, mas está perdendo a configuração quando eu posto, por isso está difícil de visualizar o que tentei explicar.
  • Galera, complementando os ótimos comentários, mostrarei a TABELA DE DUPLA ENTRADA. Tal tabela é utilizada para facilitar a resolução de questões como esta. Vejam:

    Afirmações dadas:

    1) 40% possuem plano de saúde: Isto significa que 40 % do total (80 pessoas) possuem plano de saúde; Logo 120 não têm plano;

     2) 60 % são homens: Isto significa que temos 120 homens, logo temos 80 mulheres;

    3) 25% das mulheres possuem planos de saúde: Neste caso, não são 25 % do total e sim, 25 % das mulheres. Logo, como temos 80 mulheres e 25 % têm plano, são 20 mulheres com plano e 60 sem plano.

    Colocando estes valores na tabela, fica assim:

    ---------------------------HOMEM-----------------MULHER--------------- TOTAL------------------

    COM PLANO-------------------60 -----------------------20---------------------------80---------------------

    SEM PLANO--------------------60------------------------60--------------------------120---------------------

    TOTAL----------------------------120----------------------80---------------------------200---------------------

    Como a questão nos pede a probabilidade de que seja mulher e possua plano de saúde, temos que dividir a quantidade de mulheres com plano (20) pelo total de funcionários da empresa (200). Então: 20/200 = 1/10. Gabarito: Letra: A



    Conheçam e inscrevam-se no meu canal no youtube, pois sou professor de Matemática e gravei alguns vídeos com dicas e bizus de Matemática e Raciocínio Lógico.

    Link do canal: https://www.youtube.com/channel/UC_FQm8aivYBf2q6ga1rxklw

    Face: JULIO CESAR SALUSTINO




  • 200 FUNCIONÁRIOS



    60% * 200 = 120 HOMENS

    40% * 200 = 80 MULHERES 

    25% * 80 MULHERES possuem plano de saúde = 20 MULHERES

    Probabilidade de ser mulher e com plano = 20 / 200 
    P = 1/10
  • Veja que 80 funcionários possuem plano de saúde (40% de 200). Além disso, veja que 120 funcionários são homens (60% de 200), de modo que os 80 restantes são mulheres.

           Como 25% dessas mulheres, ou seja, 20 mulheres possuem plano de saúde. Assim, ao selecionar 1 dos 200 funcionários da empresa, a probabilidade de escolher uma das 20 mulheres com plano de saúde é de 20 em 200, ou seja:

    Probabilidade = 20/200 = 1/10

    Resposta: A

  • Funcionarios (espaço amostral) = 200

    Homens = 60% de 200 = 120

    Mulheres = 40% de 200 = 80

    40% possuem plano de saúde = 80

    80 mulheres = 100% de mulheres, logo 25% de mulheres é igual a 20

    A probabilidade é de 20/200

    P = 1/10

    Alternativa A


ID
13213
Banca
CESPE / CEBRASPE
Órgão
TRT - 16ª REGIÃO (MA)
Ano
2005
Provas
Disciplina
Raciocínio Lógico
Assuntos

A diretoria da associação dos servidores de uma pequena empresa deve ser formada por 5 empregados escolhidos entre os 10 de nível médio e os 15 de nível superior. A respeito dessa restrição, julgue os itens seguintes.

Se a diretoria fosse escolhida ao acaso, a probabilidade de serem escolhidos 3 empregados de nível superior seria maior que a probabilidade de serem escolhidos 2 empregados de nível médio.

Alternativas
Comentários
  • A probabilidade seria a mesma.
    Pois daria 20% para o nivel superior e 20% para o nivel medio.
  • Trata-se de um problema de combinação, vejamos.1º caso: 3 empregados de nivel superior e 2 de nivel medio. (C15,3*C10,2)/C25,52º caso: 3 empregados de nivel superior e 2 de nivel medio. (C15,3*C10,2)/C25,5Reparem que se trata da mesma probabilidade, aproximadamente 38,5%.Um abraço e bons estudos
  • Eu pensei assim....

    Para o nível superior tem-se 15 opções para 3 vagas

    Para o nível médio 10 opções para 2 vagas

    15/3 e 10/2 serião as duas probabilidades que resultariam iguais. 5/1 e 5/1 respectivamente,

    que quando divididos dá 20%.

     

    Se estiver errado me mande mensagem.

     

  • Na verdade, a questão se resolve da seguinte maneira

    São seis hipóteses possíveis no caso da questão

    A comissão pode ser formada por

    5 empregados de nível superior e nenhum de nível médio
    4 empregados de nível superior e 1 de nível médio
    3 empregados de nível superior e 2 de nível médio
    2 empregados de nível superior e 3 de nível médio
    1 empregado de nível superior e 4 de nível médio
    nenhum empregado de nível superior e 5 de nível médio

    Vejam que as chances de serem escolhidos 3 de nível superior e 2 de nível médio fazem parte da mesma hipótese, ou seja, sempre que tivermos 3 empregados de nível superior na comissão, os outros dois serão de nível médio. Claro, pois a comissão é formada por 5 empregados.

    Dessa forma, a chance de serem escolhidos 3 empregados de nível superior para compor comissão, é igual a chance de de serem escolhidos 2 de nível médio, PORQUE SE TRATA DA MESMA HIPÓTESE.

    Como a questão afirma que uma probabilidade é maior que outra, está ERRADA

    Abraço galera,
    Guilherme Gatto, estudante de estatística da UnB
  • eu fiz 5/25 x 4/24 x 3/23 < 10/25 x 9/24, segui o raciocínio errado?

  • Único comentário certo é o da Fernanda Rodrigues.

    Lembre-se que essa questão é do TRT - Técnico Judiciário - Área Serviços Gerais. Ou seja, simples...

    Probabilidade empregados nível superior: 3/15 = 20%

    Probabilidade empregados nível médio: 2/10 = 20%

     

  • Aqui não é estatística, é raciocício lógico. Lembre-se questões de raciocínio lógicos são feitas para serem resolvidas rápidas, eu disse rápidas e não fáceis, logo se demorar muito para resolver ou está errado ou não aprendeu de forma eficiente.

  • 3/15 = 1/5 = 20%

    2/10 = 1/5 = 20%

    Gab Errado

  • https://sketchtoy.com/69355702

  • Gab: Errado

    o que eu quero / total que eu tenho

    3/15 = 0.2 (20%)

    2/10 = 0.2 (20%)

    São iguais


ID
20701
Banca
CESPE / CEBRASPE
Órgão
Banco do Brasil
Ano
2007
Provas
Disciplina
Raciocínio Lógico
Assuntos

Texto para os itens de 1 a 14

1 Os bancos médios alcançaram um de seus
melhores anos em 2006. A rigor, essas instituições não
optaram por nenhuma profunda ou surpreendente mudança
4 de foco estratégico. Bem ao contrário, elas apenas voltaram
a atuar essencialmente como bancos: no ano passado a
carteira de crédito dessas casas bancárias cresceu 39,2%,
7 enquanto a carteira dos dez maiores bancos do país
aumentou 26,2%, ambos com referência a 2005.
É apressado asseverar que essa expansão do
10 segmento possa gerar maior concorrência no setor. Vale
lembrar, apenas como comparação, que a chegada dos
bancos estrangeiros (nos anos 90) não surtiu o efeito
13 esperado quanto à concorrência bancária. Os bancos
estrangeiros cobram o preço mais alto em 21 tarifas. E os
bancos privados nacionais, médios e grandes, têm os preços
16 mais altos em outras 21. O tamanho do banco não determina
o empenho na cobrança de tarifas. O principal motivo da
fraca aceleração da concorrência do sistema bancário é a
19 permanência dos altos spreads, a diferença entre o que o
banco paga ao captar e o que cobra ao emprestar, que não se
altera muito, entre instituições grandes ou médias.
22 Vale notar, também, que os bons resultados dos
bancos médios brasileiros atraíram grandes instituições do
setor bancário internacional interessadas em participação
25 segmentada em forma de parceria. O Sistema Financeiro
Nacional só tem a ganhar com esse tipo de integração. Dessa
forma, o cenário, no médio prazo, é de acelerado movimento
28 de fusões entre bancos médios, processo que já começou.
Será um novo capítulo da história bancária do país.


Gazeta Mercantil, Editorial, 28/3/2007.

Considere que existam 65 diferentes tarifas cobradas pelos bancos no país. Nessa situação, escolhendo-se ao acaso uma dessas tarifas, a possibilidade de que seja selecionada uma daquelas em que "os bancos estrangeiros cobram o preço mais alto" é superior a 33%.

Alternativas
Comentários
  • 21 (os bancos estrangeiros cobram o preço mais alto) / 65 tarifas (total) * 100 = 32%
  • P (tarifa em que os bancos estrangeiros cobram o preço mais alto) = 21/65

    21/65 > 33%

    21/65 > 33/100

    21 x 100 = 2100

    65 x 33 = 2145

    2100 > 2145 ( errado)  

  • espaço  = 65(total de tarifas)

    numero de tarifas relacionadas a bancos extrangeiros = 21

    probalilidade afirmada pela questão  =0,33

    65 x 0,33 = 21,45

    21,45  > 21

     

    gabarito : errado

  • http://sketchtoy.com/69355705

  • Eu usei a Regra de três

    Bancos Totais são: %

    65----------------------------------------------------------- 100

    21 (Os mais caros) ?

    65x= 100 X 21

    65X= 2100

    X= 2100 / 65

    X=32

  • Gabarito: Errado.

    Do texto, 21 cobram a tarifa mais alta.

    No total, são 65 tarifas, então a probabilidade será 21/65. Vamos calcular 33% de 65:

    1% = 0,65. Logo, 3% = 3 x 0,65 = 1,95.

    10% = 6,5. Logo, 30% = 3 x 6,5 = 19,50.

    33% = 19,50 + 1,95 = 21,45.

    Como achamos um valor inferior a 21,45, a probabilidade é INFERIOR a 33%.

    Bons estudos!

  • Galera, gravei um vídeo comentando esta questão

    https://youtu.be/vsLE-aUBpcI


ID
23434
Banca
CESPE / CEBRASPE
Órgão
Caixa
Ano
2006
Provas
Disciplina
Raciocínio Lógico
Assuntos

Texto para os itens de 75 a 80.

       Conheça o título de pagamento único CAIXACAP DA SORTE, da CAIXA CAPITALIZAÇÃO, e dê mais chances à sua sorte. Você escolhe o valor que quer investir, de R$ 200,00 a R$ 900,00, múltiplos de R$ 100,00, paga uma única vez e concorre, todo mês, a 54 prêmios de até R$ 700 mil. E, ao final do prazo de capitalização, você recebe 100% do valor guardado, atualizado pela taxa referencial de juros (TR).
       Além dos sorteios mensais, tem o sorteio nos meses de julho durante a vigência do título, com premiação em dobro: serão 2 clientes contemplados com o prêmio de até R$ 700 mil. Além de tudo isso, em julho de 2007 ocorrerá o sorteio especial, quando você concorrerá durante 1 semana a 7 prêmios, um por dia, de até R$ 1 milhão, como mostra a tabela abaixo, cujos valores estão em reais.
       Os sorteios serão realizados pela Loteria Federal do Brasil, sempre no último sábado de cada mês. Se você for sorteado, continua concorrendo, exceto ao sorteio especial, realizado em julho de 2007.

valor do título: 900
1 prêmio mensal principal: 700 mil
3 prêmios mensais extras: 70 mil
50 prêmios mensais adicionais: 7.000
dupla chance: 700 mil
prêmio especial (jul./2007): 1 milhão

valor do título: 800
1 prêmio mensal principal: 622 mil
3 prêmios mensais extras: 62 mil
50 prêmios mensais adicionais: 6.200
dupla chance: 622 mil
prêmio especial (jul./2007): 888 mil

valor do título: 700
1 prêmio mensal principal: 544 mil
3 prêmios mensais extras: 54 mil
50 prêmios mensais adicionais: 5.400
dupla chance: 544 mil
prêmio especial (jul./2007): 777 mil

valor do título: 600
1 prêmio mensal principal: 466 mil
3 prêmios mensais extras: 46 mil
50 prêmios mensais adicionais: 4.600
dupla chance: 466 mil
prêmio especial (jul./2007): 666 mil

valor do título: 500
1 prêmio mensal principal: 388 mil
3 prêmios mensais extras: 38 mil
50 prêmios mensais adicionais: 3.800
dupla chance: 388 mil
prêmio especial (jul./2007): 555 mil

valor do título: 400
1 prêmio mensal principal: 311 mil
3 prêmios mensais extras: 31 mil 50
prêmios mensais adicionais: 3.100
dupla chance: 311 mil
prêmio especial (jul./2007): 444 mil

valor do título: 300
1 prêmio mensal principal: 233 mil
3 prêmios mensais extras: 23 mil
50 prêmios mensais adicionais: 2.300
dupla chance: 233 mil
prêmio especial (jul./2007): 333 mil

valor do título: 200
1 prêmio mensal principal: 155 mil
3 prêmios mensais extras: 15 mil
50 prêmios mensais adicionais: 1.500
dupla chance: 155 mil
prêmio especial (jul./2007): 222 mil

Com base nas informações relativas ao CAIXACAP DA SORTE apresentadas no texto, julgue os itens a seguir.

A probabilidade de um detentor de um título CAIXACAP DA SORTE ser sorteado durante a vigência do título é igual a 1 dividido por 36 vezes 54, independentemente do número total de detentores desses títulos.

Alternativas
Comentários
  • A probalidade de um detentor de um título ser sorteado DEPENDE do número total de detentores desses títulos. 
  • fiquei com uma dúvida...

    mesmo se a questão falasse dependendo do numero total de detentores, não teria que ser 1 / 54x12 ??

     

     

  • Dica: questões de raciocínio lógico, principalmente as grandes, vá direto a pergunta. As vezes nem precisa do texto.


ID
23443
Banca
CESPE / CEBRASPE
Órgão
Caixa
Ano
2006
Provas
Disciplina
Raciocínio Lógico
Assuntos

Texto para os itens de 75 a 80.

       Conheça o título de pagamento único CAIXACAP DA SORTE, da CAIXA CAPITALIZAÇÃO, e dê mais chances à sua sorte. Você escolhe o valor que quer investir, de R$ 200,00 a R$ 900,00, múltiplos de R$ 100,00, paga uma única vez e concorre, todo mês, a 54 prêmios de até R$ 700 mil. E, ao final do prazo de capitalização, você recebe 100% do valor guardado, atualizado pela taxa referencial de juros (TR).
       Além dos sorteios mensais, tem o sorteio nos meses de julho durante a vigência do título, com premiação em dobro: serão 2 clientes contemplados com o prêmio de até R$ 700 mil. Além de tudo isso, em julho de 2007 ocorrerá o sorteio especial, quando você concorrerá durante 1 semana a 7 prêmios, um por dia, de até R$ 1 milhão, como mostra a tabela abaixo, cujos valores estão em reais.
       Os sorteios serão realizados pela Loteria Federal do Brasil, sempre no último sábado de cada mês. Se você for sorteado, continua concorrendo, exceto ao sorteio especial, realizado em julho de 2007.

valor do título: 900
1 prêmio mensal principal: 700 mil
3 prêmios mensais extras: 70 mil
50 prêmios mensais adicionais: 7.000
dupla chance: 700 mil
prêmio especial (jul./2007): 1 milhão

valor do título: 800
1 prêmio mensal principal: 622 mil
3 prêmios mensais extras: 62 mil
50 prêmios mensais adicionais: 6.200
dupla chance: 622 mil
prêmio especial (jul./2007): 888 mil

valor do título: 700
1 prêmio mensal principal: 544 mil
3 prêmios mensais extras: 54 mil
50 prêmios mensais adicionais: 5.400
dupla chance: 544 mil
prêmio especial (jul./2007): 777 mil

valor do título: 600
1 prêmio mensal principal: 466 mil
3 prêmios mensais extras: 46 mil
50 prêmios mensais adicionais: 4.600
dupla chance: 466 mil
prêmio especial (jul./2007): 666 mil

valor do título: 500
1 prêmio mensal principal: 388 mil
3 prêmios mensais extras: 38 mil
50 prêmios mensais adicionais: 3.800
dupla chance: 388 mil
prêmio especial (jul./2007): 555 mil

valor do título: 400
1 prêmio mensal principal: 311 mil
3 prêmios mensais extras: 31 mil 50
prêmios mensais adicionais: 3.100
dupla chance: 311 mil
prêmio especial (jul./2007): 444 mil

valor do título: 300
1 prêmio mensal principal: 233 mil
3 prêmios mensais extras: 23 mil
50 prêmios mensais adicionais: 2.300
dupla chance: 233 mil
prêmio especial (jul./2007): 333 mil

valor do título: 200
1 prêmio mensal principal: 155 mil
3 prêmios mensais extras: 15 mil
50 prêmios mensais adicionais: 1.500
dupla chance: 155 mil
prêmio especial (jul./2007): 222 mil

Com base nas informações relativas ao CAIXACAP DA SORTE apresentadas no texto, julgue os itens a seguir.

Ao longo dos 36 meses de capitalização, o detentor de um único título CAIXACAP DA SORTE tem a possibilidade de ganhar no máximo R$ 1 milhão.

Alternativas
Comentários
  • pessoal é o seguinte fiz a questão por dedução. Não sei se ajuda ou se estou certo no meu raciocínio. Porém, a título de ajuda pensei:

    vizualizei que juntando todos as possibilidade de prêmios da pessoa que tem um título de R$900. Daria possibilidade de prêmios maior que 1milhão (se o sortudo ganhasse em todos).
    Assim a assertiva do enunciado está inválida. (Incorreta)
  • sacanagem uma questão desse tamanho, não perderia meu tempo lendo..sejamos mais objetivos né!

  • pergunta extremamente idiota....

  • SE EU CHEGO NA PROVA E VEJO UMA QUESTÃO DESSA, EU CHORO

  • O tamanhao é para assustar mesmo. Pense que a pessoa possa ser sortuda e ganhar em tudo, logo pode ganhar mais de 1 milhão.


ID
25165
Banca
CESPE / CEBRASPE
Órgão
TSE
Ano
2007
Provas
Disciplina
Raciocínio Lógico
Assuntos

Para se ter uma idéia do perfil dos candidatos ao cargo de Técnico Judiciário, 300 estudantes que iriam prestar o concurso foram selecionados ao acaso e entrevistados, sendo que, entre esses, 130 eram homens. Como resultado da pesquisa, descobriuse que 70 desses homens e 50 das mulheres entrevistadas estavam cursando o ensino superior. Se uma dessas 300 fichas for selecionada ao acaso, a probabilidade de que ela seja de uma mulher que, no momento da entrevista, não estava cursando o ensino superior é igual a

Alternativas
Comentários
  • Se o número de homens é de 130/300, logo o número de mulheres será de 170/300.
    Dessas 170 mulheres apenas 50 tem nivel superior, portanto restam 120/300 sorbre o total de candidatos.
    120/300=0,4
  • Total de estudantes que iam prestar o concurso = 300
    Total de homens= 130, sendo que somente 70 homesns tinham curso superior
    Total de mulheres( 300- 130)= 130. sendo que 50 mulheres tinham curso superior, logo 120 mulheres nao tinham curso superior. aprobalidade será 120/300= 0,40
  • 300candidatos; 130 homens;300-130=170 : restaram 170 mulheres; das 170 mulheres, 50 tinham curso superior, ou seja,170-50=120. assim sendo 120/300=0,40.
  • 300 ESTUDANTES

    130 HOMENS, 70 HOMENS CURSANDO NIVEL SUPERIOR

    300-130 HOMENS=170 MULHERES, 50 MULHERES CURSANDO NIVEL SUPERIOR.

    170-50=120/300=0,4.

  • 300 - 130 = 170

     

    170 - 50 = 120

     

    120/300 = 0,4

  •                                         300 estudantes


    130 homens                                                170 mulheres


    70 c/ ensino superior                                   50 c/ ensino superior


    60 ñ cursando ensino sup.                          120 ñ cursando ensino sup.


    P = 120 / 300


    P = 0,4 ou 40%

  • Alternativa A:


    300 estudantes = 130 homens + 170 mulheres


    130 homens

    70 cursando superior

    60 não


    170 mulheres

    50 cursando superior

    120 não


    P = 120 / 300 = 0,40


  • LETRA A

  • 2/5 = 0,4 = Letra A


ID
29074
Banca
CESGRANRIO
Órgão
Petrobras
Ano
2008
Provas
Disciplina
Raciocínio Lógico
Assuntos

Pedro está jogando com seu irmão e vai lançar dois dados perfeitos. Qual a probabilidade de que Pedro obtenha pelo menos 9 pontos ao lançar esses dois dados?

Alternativas
Comentários
  • Pelo menos 9 pontos = no mínimo 9 pontos.

    Sendo assim, temos como casos favoráveis:
    3+6); (4+5); (4+6); (5+4); (5+5); (5+6); (6+3); (6+4); (6+5); (6+6). Num total de 10 casos.

    O numero total de possibilidades (espaço amostral)para o lançamento de dois dados perfeitos é de:
    6 x 6 = 36

    Logo, a probabilidade de tal fato ocorrer é dado por:

    P = 10/36 = 5/18
  • A questão está mal elaborada!É passível de ser anulada.
  • Pontos necessários = 9 pontos.

    pontos provaveis:
    3+6; 4+5; 4+6; 5+4; 5+5; 5+6; 6+3; 6+4; 6+5; 6+6. total de 10 casos.

    Veja o nosso espaço amostral e para o lançamento de dois dados perfeitos é de: 6 x 6 = 36

    Agora veja, a probabilidade deste fato ocorrer é dado por:

    P(9) = 10/36 = 5/18

  • O que é importante nessa questão é que, quando invertemos a ordem das probabilidades, por exemplo, (3-6) e (6-3), não podemos inverter tb para o (5-5) e (6-6), senão teremos 12 possibilidades de ocorrência.
  • tanks pra quem ajudou!.. questãozinha sussa!
  • Soma = 09 => (3,6) (4,5) (5,4) (6,3)

    Soma = 10 => (4,6) (5,5) (6,4)

    Soma = 11 => (5,6) (6,5)

    Soma = 12 => (6,6)

     

    Total de casos = 10

     

    10 / 36 = 

    5 / 18

     

    Gabarito: D

     

  • Entendi 9 pontos exatos, hahahaha acontece


ID
44239
Banca
ESAF
Órgão
Receita Federal
Ano
2009
Provas
Disciplina
Raciocínio Lógico
Assuntos

Na antiguidade, consta que um Rei consultou três oráculos para tentar saber o resultado de uma batalha que ele pretendia travar contra um reino vizinho. Ele sabia apenas que dois oráculos nunca erravam e um sempre errava. Consultados os oráculos, dois falaram que ele perderia a batalha e um falou que ele a ganharia. Com base nas respostas dos oráculos, pode-se concluir que o Rei:

Alternativas
Comentários
  • Discordo da Resposta. Sendo Or1, Or2 e Or3 os oráculos - os dois primeiros são os que acertam. Or1 - Or2 - Or3 - ResultadoP - P - G(P)- 33% PerderP - G - P(G)- 33% GanharG - P - P(G)- 33% GanharLogo, o Rei teria 66% de chances de ganhar. Se eu estiver errado me corrijam.
  • 3 oráculos:2 nunca erram1 sempre errao enunciado diz que:2 disseram que o rei perderia1 disse que o rei ganharialevando em consideração que:1) nunca erra - diz que perde2) nunca erra - diz que perde3) sempre erra - diz que ganha100% de chances do rei perder. não há qualquer contradição, logo se conclui que é certo que o rei perderá.Gabarito: Letra "D"
  • Concordo com o Gabriel, se dois nunca erram, não haveria possibilidade desses darem respostas diferentes. Logo, se dois deram respotas iguais, só poderia ser dos dois que não erram.Abraços.
  • Questão bem eleborada temos o uso do conetivo logico conjunçao "e" , que somente tem valor logico verdadeiro quando todas ás proposições são verdadeiras caso contrário o valor logico será falso, desta forma temos: oráculo 01 V, oráculo 02 V  e   oráculo 03 F sendo assim valor logico falso.Resposta letra d.
  • Não concordo com a resposta. 2 oráculos estão sempre certos, 1 está sempre errado. O resultado da consulta é:
    PERDER PERDER GANHAR
    Se o oráculo que errou disse PERDER, logo o correto seriam as chances
    ~PERDER PERDER GANHAR ou seja, 33% chances de perder contra 66% de ganhar.
    Se o oráculo que errou disse GANHAR, logo o correto seriam as chances
    PERDER PERDER ~GANHAR ou seja, 100% chances de perder 

    Como é indecidível quem está certo ou errado, através de lógica acredito que não podemos assumir nenhum dos valores acima (100% perder, 33% de perder ou 66% de ganhar) além de 100% de ganhar, condição nunca satisfeita (máximo 66% de possibilidade de vitória).
  • Vejamos,
    2 Sempre acertam e 1 Sempre erra, conclui-se que dois respodem iguais (certo) e um diferente (errado).

    Se dois dois disseram que o Rei perderia a batalha e um disse que venceria, então temos 100% de chances de derrota.
    Item D
  • Pessoal, se 2 nunca erram, significa que esses dois devem dar a mesma resposta e o outro responderá exatamente o contrário, certo?
    Então, conforme o enunciado 2 disseram que ele perderia a batalha, portanto, se o outro divergiu, FECHOU. O que erra falou que ganharia a batalha... então na verdade este errou, portanto os aqueles 2 disseram a verdade, que ele perderia!!!
    Será que fui claro???
    Rs
  • Resolvendo por lógica: 2 FALAM QUE ELE PERDE E UM FALA QUE ELE GANHA

    PRIMEIRA OPÇÃO:

    1 nunca erra
    2 nunca erra
    3 sempre erra
    Conclusão: 1 e 2 nunca erram e falam que ele perde, o 3 fala que ele ganha mas sempre erra então ele perde = perde a batalha com certeza!
    SEGUNDA OPÇÃO:
    1 nunca erra
    2 sempre erra
    3 nunca erra
    Conclusão: 1 e 2 falam que ele perde, o 1 nunca erra, logo ele perde - o 2 sempre erra, e fala que ele perde, logo ele ganha  - o 3 nunca erra e fala que ele ganha . Enfim, 2 resultados com ele ganhando e 1 com ele perdendo. Não pode ocorrer 2 eventos diferentes simultaneamente!!!
    TERCEIRA OPÇÃO:
    1 sempre erra
    2 nunca erra
    3 nunca erra
    1 e 2 falam que ele perde a batalha. 1 sempre erra, logo ele ganha a batalha  - 2 nunca erra, logo ele perde a batalha - JÁ PARA AÍ. ELE NÃO PODE GANHAR E PERDER A BATALHA AO MESMO TEMPO
    Desta forma, só a opção 1 fica possível, e essa opção demonstra que  ele perdeu a batalha.







  • primeiro caso:
    v 1 p
    v 2 p
    e 3 g ele sempre erra, então é p
    portanto: 3 p (ele perde)

    segundo caso
    v 1 p
    v 2 g
    v 3 p ele sempre erra, então é g
    portanto; 2 g (ele ganha)

    não entendi essa questão.
  • duas resposta deve ser perde.
    tres oráculos dois fala que perde um fala que ganha
    or1                                                  or2                                              or3 
    perde                                            perde                                        ganha
    perde                                           ganha                                        perde
    ganha                                          perde                                         perde 

    Os dois oráculos que falam a verdade terão respostas iguais obviamente.
    então todas as opções que duas respostas sejam identicas só poderia ser perde.
  • Essa nem precisa pensar, se temos dois que nunca erram então assumimos isso como verdade, ou seja dois disseram que ele ia perder, logo o outro vai errar, dizer que ele vai ganhar. Ou seja 100% de chance de perder.

  • O enunciado é claro: 2 SEMPRE acertam. Não tem como um dizer que é Ganha e o outro dizer que Perde.  se 2 disseram que perde, fatalmente 1 que sempre acerta disse  que perde e o outro que sempre acerta irá concordar com este.  Não abre mão para cálculos huasahsasu

  • Os 2 que acertam obrigatoriamente tem que ter a mesma resposta . Sendo assim, a única forma de isso acontecer é:

    Oráculo que NUNCA erra 1: Perde batalha;

    Oráculo que NUNCA erra 2: Perde batalha;

    Oráculo que SEMPRE erra:  Vence Batalha.


    Qualquer outra combinação será contrária ao enunciado.

  • Observe que 2 oráculos devem acertar a previsão e 1 errar. Como apenas 1 disse que o rei ganha a batalha, este único oráculo não pode ter acertado sozinho. Sua previsão deve estar errada, sendo correta a previsão dos demais oráculos. Ou seja, o rei certamente perderia a batalha.

    Resposta: D


ID
44242
Banca
ESAF
Órgão
Receita Federal
Ano
2009
Provas
Disciplina
Raciocínio Lógico
Assuntos

Ao se jogar um determinado dado viciado, a probabilidade de sair o número 6 é de 20%, enquanto as probabilidades de sair qualquer outro número são iguais entre si. Ao se jogar este dado duas vezes, qual o valor mais próximo da probabilidade de um número par sair duas vezes?

Alternativas
Comentários
  • Espaço amostral: S = {1, 2, 3, 4, 5, 6}Seja p(i) a probabilidade de sair o número i.P(6) = 20% = 0,2Logo: p(1) + p(2) + p(3) + p(4) + p(5) = 80% = 0,8Portanto: p(1) = p(2) = p(3) = p(4) = p(5) = 0,8/5p(1) = p(2) = p(3) = p(4) = p(5) = 0,16A probabilidade de o resultado ser par em um lançamento será:p(2) + p(4) + p(6) = 0,16 + 0,16 + 0,20 = 0,52Logo a probabilidade de ocorrer dois números pares em dois lançamentos(independentes) será: 0,52 x 0,52 = 0,2704 = 27,04%
  • Probabilidade de sair o número 6: 20% 
    Probabilidade de sair outro número diferente de 6: 80% / 5 = 16% (para cada número) 
    Probabilidade de sair um número par (2, 4 ou 6) = 16% + 16% + 20% = 52% 
    Agora, a probabilidade de se jogar este dado duas vezes e um número par sair duas vezes é:
    52% x 52% = 27,04% 
    Resposta letra "b"
    bons estudos!
  • Probabilidade de sair o nº 6 = 20% ou 0,2

    Demais números = 80% / 5 = 16% ou  0,16 (cada um)

    Pares = 2, 4, 6

    2 eventos possíveis: PAR & PAR



    1º evento sair PAR = 2 ou 4 ou 6
    0,16 + 0,16 + 0,2 = 0,52

    2º evento sair PAR = 2 ou 4 ou 6
    0,16 + 0,16 + 0,2 = 0,52


    P = 0,52 * 0,52P = 0,2704P = 27,04%

    Valor mais próximo = 27% (alternativa B)
  • Probabilidade de cada número sair: 1 = 16%, 2 = 16%, 3 = 16%, 4 = 16%, 5 = 16%, 6 = 20%           Probabilidade de sair um número par: P(par) = 52%                Probabilidade de sair um número ímpar: P(ímpar) = 48%              Usando a fórmula da probabilidade Binomial, sendo o número de eventos = 2    P(par)=C2,2*P(par)^2*P(ímpar)^0            P(par)=1*(0,52)^2*(0,48)^0        P(par)=0,2704  =   27,04%
  • Nesse tipo de questão sempre dou uma uma checada nas opções antes de responder a questão pra evitar a demora respondendo.
    Apesar de saber como calcular as vezes as alternativas ajudam.

    A possibilidade de se retirar o número par 2x jogando um dado normal é de 25%, se o número que está viciado é par (6) a possibilidade de se retirar um número par é mair que 25% e dentre as opções só há 1 possibilidade que se encaixa nessa estimativa que a alternativa B (27).

     

  • Dizemos que um dado é viciado quando ele tem algum “defeito” que faz com que a probabilidade de alguns resultados serem obtidos ser maior do que a de outros resultados. É o que ocorre aqui: como a probabilidade de sair o 6 é de 20%, resta 80% de chance de sair algum dos outros 5 números do dado. Como a probabilidade destes 5 números é igual, podemos dividir estes 80% restantes entre os 5, obtendo:

    Probabilidade de cada um dos outros números = 80% / 5 = 16%

           Observando os números pares, veja que a probabilidade de obter 2 é 16%, de obter 4 é 16% e de obter 6 é 20%. Portanto, a probabilidade de obter um número par em um lançamento é:

    Probabilidade (par em 1 lançamento) = 16% + 16% + 20% = 52%

           Para obter número par no primeiro E no segundo lançamentos, temos:

    Probabilidade (par nos 2 lançamentos) = 52% x 52% = 0,2704 = 27,04%

    Resposta: B

  • Essa questão foi mal escrita

    a probabilidade de um número par sair duas vezes é diferente de saírem dois números pares

    minha interpretação: um número, par, sorteado aleatoriamente e o mesmo número sorteado novamente

    2, 2

    4, 4

    6, 6


ID
44248
Banca
ESAF
Órgão
Receita Federal
Ano
2009
Provas
Disciplina
Raciocínio Lógico
Assuntos

Em um determinado curso de pós-graduação, 1/4 dos participantes são graduados em matemática, 2/5 dos participantes são graduados em geologia, 1/3 dos participantes são graduados em economia, 1/4 dos participantes são graduados em biologia e 1/3 dos participantes são graduados em química. Sabe-se que não há participantes do curso com outras graduações além dessas, e que não há participantes com três ou mais graduações. Assim, qual é o número mais próximo da porcentagem de participantes com duas graduações?

Alternativas
Comentários
  • Questão envolvendo frações de cursandos, os que fazem mais de um curso são os excedentes ao grupo ( 1 inteiro ). Assim temos:1/4 + 2/5 + 1/3 + 1/4 + 1/3 = 94/60 = 1,566Aproximando o resultado, como está no enunciado:c) 57 %
  • Eu pensei de um jeito bem simplório, mas deu certo:

    Eu considerei que o número total de estudantes era 100. A partir daí achei a quantidade de graduados em cada curso.
    Somando tudo deu 156,66.  Como só tem 100 alunos, significa que  56,66 alunos  possuem mais de uma graduação!
    e como tomei por base o número 100, a porcentagem já está aí:  aprox. 57%
  • Eu pensei de modo semelhante à Andreia, mas escolhi o numero 120, por ser um multiplo das frações:
    120 alunos no total
    matematica = 30
    geologia = 48
    economia = 40
    biologia = 30
    quimica = 40
    Total = 188
    Excedentes = 188 - 120 = 68. Ou seja, 68 alunos fazem mais de uma graduação
    A porcentagm pedida é relaiconada ao numero total de alunos:
    68/120 = aproxim. 0,57
  • 25% -> 1/4
    40% -> 2/5
    33,3% -> 1/3
    25% -> 1/4
    33,3% -> 1/3
    ------------------
    156,6%


    Sabemos que existem apenas graduados em pelo menos 1 ou 2 matérias. Considerando este "pelo menos 1", sabemos que 100% tem, ao menos, 1 graduação. Com isto, pelo menos os outros 56,6% terão necessáriamente 2 graduações.

    156,6% - 100% = 56,6% ~= 57% 

    Alternativa: a)
  • 1/4 + 2/5 + 1/3 + 1/4 + 1/3

    MMC = 60

    Soma = 15/60 + 24/60 + 20/60 + 15/60 + 20/60 = 94/60

    Retira-se o excedente de um inteiro, que será a parcela de pessoas que são graduados em mais de uma formação:

    94/60 - 60/60 = 34/60.

    Logo, 60 está para 100%, assim como 34 está para "x".

    x = (34 x 100) / 60 = 56,6% = aprox. 57%.


ID
44254
Banca
ESAF
Órgão
Receita Federal
Ano
2009
Provas
Disciplina
Raciocínio Lógico
Assuntos

Ao se jogar um dado honesto três vezes, qual o valor mais próximo da probabilidade de o número 1 sair exatamente uma vez?

Alternativas
Comentários
  • chance de sair 1: 1/6chance de não sair 1: 5/61/6 x 5/6 x 5/6 = 25/216agora cabe uma observação: pode parecer óbvio para muitos, mas vou explicar isso porque demorei algum tempo para perceber. devemos multiplicar o resultado 25/216 por 3, uma vez que:1) o 1 pode sair na primeira jogada e não na segunda ou terceira;2) o 1 pode sair na segunda jogada e não na primeira ou terceira;3) o 1 pode sair na terceira jogada e não na primeira ou segunda.dito isso: 25/216 x 3 = aprox. 35%Gabarito: Letra "A"
  • Probabilidade de sair o número 1: 1/6
    Probabilidade de sair um número diferente de 1: 5/6
    Agora, a probabilidade de sair apenas uma vez o número 1, jogando-se o dado três vezes é:
    Probabilidade = (5/6 x 5/6 x 1/6) + (5/6 x 1/6 x 5/6) + (1/6 x 5/6 x 5/6)
    Probabilidade = 3 x 25/216
    Probabilidade = 75/216 = 34,72%

    Gabarito:a
    bons estudos!
     
  • Tbm pode usar C3,1 * (1/6)*(5/6)*(5/6) =0,35
  • fiz mais simples:

    chance de sair o número 1: 1/6 = 16%

    chance de sair o número 1 nos três lançamentos: 1/6 + 1/6 + 1/6 = 1/2 = 50%

    50% - 16% = 34% valor mais próximo da probabilidade 35%

  • c (3,1)

    sim e nao e nao

    nao e sim e nao 

    nao e nao e sim 

    sair 1 (1/6) - nao (5/6) - nao (5/6)  25/216 * 3  = 0,34



  • O assunto específico da questão chama-se probabilidade binomial. Tem um fórmula doidona pra resolver. Ainda estou tentando entender para resolver de um modo mais simples. 


    Definição do Prof. José Luiz de Morais: ´´é a probabilidade de um evento ocorrer um número exato de vezes, em determinado número de tentativas´´.


    Fonte: Livro Matemática e Lógica para concursos, página 471.

    http://www.saraiva.com.br/matematica-e-logica-para-concursos-3867850.html

  • Distribuição Binomial

    Dado um número de eventos independentes n, com K sucessos esperados, sendo p a probabilidade de sucesso em cada evento, temos:

    P (sucessos=K) = Cn,k    x     p^K     x    (1-p)^n-k

    P (sucessos=1) = C3,1    x       (1/6)^1    x     (5/6)^2

    P (sucessos=1) = 3         x        1/6        x       25/36

    P (sucessos=1) = 0.347222222222 ---> aproximadamente 35%

  • Em uma jogada, a chance de obter o número um é de 1 em 6, ou 1/6. Já a chance de não obter o número um é de 5 em 6, ou 5/6. Portanto, a chance de o primeiro número ser o 1 e os dois números seguintes serem diferentes de 1 é:

    Probabilidade(apenas o primeiro ser 1) = (1/6) x (5/6) x (5/6) = 25/216

           Entretanto, devemos considerar ainda que o número 1 poderia ser obtido no 2º ou o 3º lançamentos, ao invés do primeiro. Para isto, basta multiplicar o resultado acima por 3:

    Probabilidade (obter 1 em apenas um dos 3 lançamentos) = 3 x (25/216)

    Probabilidade (obter 1 em apenas um dos 3 lançamentos = 25/72 = 0,347

    Probabilidade (obter 1 em apenas um dos 3 lançamentos = 34,7%

    (aproximadamente 35%)

    Resposta: A

  • Letra (A) = 35%

    DISTRIBUIÇÃO BINOMIAL

    n = 3

    k = 1

    p = 1/6

    q = 5/6

    C3,1 x 1/6 x 25/36 = 25/72 = 35%

  • Eu considerei que existem 3 possibilidades, uma do número 1 sair primeiro, outra de sair em segundo e outra de sair em terceiro, ou seja:

    1.5.5 ou 5.1.5 ou 5.5.1

    "1" é a possibilidade de sair o número 1 e "5" são as possibilidades de sair quaisquer outro valor diferente de 1.

    Como "ou" é soma em probabilidade, temos:

    5^2 três vezes-> 5^2.3= 75

    Se pensarmos nessa mesma questão, sem nenhuma restrição, vamos ter 6 possibilidades pra cada jogada, ou seja:

    6*6*6= 216

    Casos desejados/ casos possíveis= 75/216= 0,35


ID
44755
Banca
ESAF
Órgão
ANA
Ano
2009
Provas
Disciplina
Raciocínio Lógico
Assuntos

Uma urna possui 5 bolas azuis, 4 vermelhas, 4 amarelas e 2 verdes. Tirando-se simultaneamente 3 bolas, qual o valor mais próximo da probabilidade de que as 3 bolas sejam da mesma cor?

Alternativas
Comentários
  • Primeira decisão: Arranjo ou Combinação? Combinação porque a ordem das bolinhas não modifica o conjunto resultado.Primeira conta: Combine cada uma das cores onde há mais de 3 bolinhas e grupos de 3: C5,3 + C4,3 + C4,3 => 18. Ou seja, 18 oportunidades de sair 3 cores repetidas.Segunda conta: Combine as 15 bolinhas e grupos de 3. C15,3 => 15!/(3!*12!). Isso da 455.Temos então 18 chances em 455 possíveis. 0,0396 ou 3,96%
  • A solução do colega está perfeita. Segue outra opção para resolução, sem fórmula:
     
    Para as bolas azuis (5):
     
    5/15 x 4/14 x 3/13 = 2/91
    (4/455 possibilidades de tirar uma bola azul num total de 15 bolas X (lembre-se: quantas vezes pode-se tirar uma outra bola da mesma cor)
    4 possibilidades de tirar uma bola azul num total de 14 bolas (pq já tirei 1) X
    3 possibilidades de tirar uma bola azul num total de 13 bolas (pq já tirei 2))
     

    +
     
    Para as bolas vermelhas (4):
     
    4/15 x 3/14 x 2/13 = 4/455
     
    +
     
    Para as bolas amarelas (4):
     
    Idem vermelhas: 4/455
     
     
    Para as bolas verdes (2): sem possibilidade.

     
    Soma-se os resultados de cada cor:
    2/91 + 4/455 + 4/455 = 18/455

ID
44758
Banca
ESAF
Órgão
ANA
Ano
2009
Provas
Disciplina
Raciocínio Lógico
Assuntos

Na população brasileira verificou-se que a probabilidade de ocorrer determinada variação genética é de 1%. Ao se examinar ao acaso três pesssoas desta população, qual o valor mais próximo da probabilidade de exatamente uma pessoa examinada possuir esta variação genética?

Alternativas
Comentários
  • 1/100 (com variação genética) x 99/100 (sem variação genética) x 99/100 (sem variação genética) = 0,98% x 3 grupos diferentes de pessoas que podem ser formados = 2,94%
  • Ricco, vc pode me explicar pq vc multiplicou duas vezes o 99/100?
    Não consegui entender!!!
    Obrigada..

  • Probabilidade de ter: 1/100 
    Probabilidade de não ter: 99/100.
    P1 pessoa ter e e 2 não terem = (1/100 . 99/100 . 99/100) . P32  [multiplicamos pela permutação dos três elementos, sendo 2 repetidos, para abarcarmos todas as possibilidades] = 9801/1000000 . 3 = 29403/1000000 = 0,0294 = 2,94%

  • Multiplica por 3 pq são 3 combinações possíveis. Quando eu escolho 3 pessoas, as amostras podem sair de 3 maneiras diferentes:

    1% , 99%, 99%

    ou

    99%, 1%, 99%

    ou

    99%, 99%, 1%


    Por esse motivo se multiplica por 3.
  • Questão de distribuição binomial.    Combinação de n, k  vezes p^k vezes q^n-k

    onde: n número de casos, k = número de casos favoráveis (sucessos) p = probabilidade de sucesso, q = probabilidade de fracasso ;  
    C3,1 x 0,01 x 0,99^2                                              
                                                                                                                                        3 x 0,01 x 0,9801                                                                                                                                                                          0,0294 que é a forma unitária de 2,94%
  • Digamos que selecionamos as pessoas A, B e C da população. A chance de cada uma ter a variação genética é de 1%, de forma que a chance de cada uma não ter a variação genética é de 99%.

           Para que A tenha a variação E a pessoa B não tenha E a pessoa C não tenha, as chances são de:

    Probabilidade (A ter, B e C não) = 1% x 99% x 99%

           Da mesma forma, para que B tenha a variação E a pessoa A não tenha E a pessoa C não tenha, temos:

    Probabilidade (B ter, A e C não) = 99% x 1% x 99%

           Por fim, para que apenas C tenha a variação:

    Probabilidade (C ter, A e B não) = 99% x 99% x 1%

           A probabilidade de que apenas A, OU apenas B, OU apenas C tenha a variação genética, basta somarmos as 3 acima, obtendo:

    Probab. (só A, só B ou só C) = 1%x99%x99% + 99%x1%x99% + 99%x99%x1%

    Probab. (só A, só B ou só C) = 3x(1%x99%x99%) = 0,0294 = 2,94%

    Resposta: D


ID
47671
Banca
ESAF
Órgão
SEFAZ-SP
Ano
2009
Provas
Disciplina
Raciocínio Lógico
Assuntos

Considere que numa cidade 40% da população adulta é fumante, 40% dos adultos fumantes são mulheres e 60% dos adultos não-fumantes são mulheres. Qual a probabilidade de uma pessoa adulta da cidade escolhida ao acaso ser uma mulher?

Alternativas
Comentários
  • Calculando-se a porcentagem total das mulheres em relação à população inteira, teremos a porcentagem de chances de escolher uma mulher dentro desta população.Segundo o enunciado:Dentre os 40% fumantes 40% são mulheres = 0,4 x 0,4 = 0,16 ou 16%Dentre os 60% não fumantes 60% são mulheres = 0,6 x 0,6 = 0,36 ou 36%Somando a população feminina = 52 %Então a probabilidade é 1 em 52%Alternativa b) 52%
  • para quem é sádico como eu:suponhamos que a populaçao seja de 100 habitantes.40% dos habitantes são fumantes, logo há 40 fumantes na cidade40% dos fumantes são mulheres, logo 16 mulheres fumantes60% dos não-fumantes são mulheres, não-fumantes= 60 pessoas, dessas 60 pessoas que não fumam, 60%são mulheres(60 por cento de 60 dá 36)total de mulheres que fumam = 16total de mulheres que não fumam=36total de mulheres=5252% da população são mulheres, então, ao se escolher ao acaso uma pessoa, há 52% de probabilidade de ser uma mulher, seja ela fumante ou não.
  • montar o quadrinho ajuda bastante. Suponha 100 pessoas na população:
      homem mulher total
    fuma 24 16 40
    não fuma 24 36 60
    total 48 52 100
  • tudo certinho, apenas um detalhe, a probabilidade é 52 em 100
    Eventos desejados
    _________________= 0,52= 52%

    Eventos Possíveis

    [ ]s
  • Seja,

    O:fumante

    A:mulher

    Lei da probabilidade total: P(A) = P(A|O)P(O) + P(A|Õ)P(Õ) 

    P(O) = 40% fumante ------- P(Õ)=60% não fumante

    P(A|O) = 40% ser mulher dado fumante ------- P(A|Õ)=60% ser mulher dado não fumante

    pela fórmula: P(A)=(0.4)(0.4)+(0.6)(0.6)= 0.52 ou 52% ser mulher

  • De cada 100 adultos desta cidade, 40 são fumantes. Destes 40 adultos fumantes, 40%, ou seja, 16 são mulheres.

    Dos adultos não fumantes (100 – 40 = 60 adultos), sabemos que 60% são mulheres. Isto é, 60% x 60 = 36 são mulheres.

    Portanto, de cada 100 adultos da cidade, 16 são mulheres fumantes e 36 são mulheres não fumantes, totalizando 16 + 36 = 52 mulheres.

    Temos um total de 100 adultos, dos quais 52 atendem a condição do enunciado (são mulheres). A probabilidade de escolher uma mulher é:

    Probabilidade = 52 / 100 = 0,52 = 52%

    Resposta: B

  • 40%.40% + 60%.60%

    16 + 36 = 52%


ID
47674
Banca
ESAF
Órgão
SEFAZ-SP
Ano
2009
Provas
Disciplina
Raciocínio Lógico
Assuntos

Dados da Questão: 29 - Considere que numa cidade 40% da população adulta é fumante, 40% dos adultos fumantes são mulheres e 60% dos adultos não-fumantes são mulheres. Qual a probabilidade de uma pessoa adulta da cidade escolhida ao acaso ser uma mulher?

Considerando os dados da questão anterior, qual a porcentagem das mulheres adultas que são fumantes?

Alternativas
Comentários
  • Usando os dados obtidos na resolução da questão anterior temos:Total das fumantes (40%) dentro do universo fumante (40%) = 0,4 x 0,4 = 0,16 ou 16%Total das mulheres dentre os adultos (*obtido na resolução da questão anterior) = 52 %Então porcentagem de fumantes adultas mulheres = 16% / 52% = 4/13Alternativa d) 4/13
  • o enunciado não foi muito especifico na pergunta.ele quer saber a porcentagem dentre as mulheres ou dentre os fumantes?"qual a porcentagem das mulheres adultas que são fumantes?"não posso saber de que é a porcentagem pedida.o enunciado ficaria melhor assimentre as mulheres, qual a porcentagem de fumantes?mas como a intenção é dificultar, cabe a nós ficarmos ligados.
  • A questão anterior que o enunciado se refere é essa: Q15888
  • Montar quadrinho ajuda: resposta: 16/52 = 4/13

     

    homem

     

     

     

    mulher

     

     

     

    total

     

     

     

    fuma

     

     

     

    24

     

     

    16

     

     

    40

     

     

     

    não fuma

     

     

     

    24

     

     

    36

     

     

    60

     

     

     

    total

     

     

     

    48

     

     

     

    52

     

     

     

    100

     

     

     

  • Pra resolver corretamente a questão, teria de resolver a anterior. Como não fiz, encontrei a resposta 16% de mulheres fumantes em relação aos adultos e não à população. A ESAF foi muito trambiqueira nesta questão, porque não foi clara na pergunta.
  • 29- Considere que numa cidade 40% da população adulta
    é fumante, 40% dos adultos fumantes são mulheres e
    60% dos adultos não-fumantes são mulheres. Qual a
    probabilidade de uma pessoa adulta da cidade escolhida
    ao acaso ser uma mulher?
    a) 44%
    b) 52%
    c) 50%
    d) 48%
    e) 56%
    30- Considerando os dados da questão anterior, qual a
    porcentagem das mulheres adultas que são fumantes?
    a) 60%
    b) 40%
    c) 7/13
    d) 4/13
    e) 9/13

    FUMANTES -- 40% da populacao adulta, logo
    Não FUMANTES--60% da população adulta
    dos FUMANTES=40% são mulheres -- 40%.40% =16%
    dos nao fumantes = 60% são mulheres -- 60%.60% = 36%
    portanto as mulheres são 16%+36%=52%
    se as mulheres são 52% então  relação mulher fumante/mulheres = 16%/52%=4/13 ( numero puro )

    Não raras são as vezes é melhor clicar e ver a prova  - sempre que estranhar algo, pega a prova!
  • o enunciado não foi muito claro, deveria ter dito "qual a porcentagem das mulheres que são fumantes entre as mulheres ..."


ID
49198
Banca
FUNIVERSA
Órgão
PC-DF
Ano
2009
Provas
Disciplina
Raciocínio Lógico
Assuntos

Um trielo é uma disputa entre três participantes, a exemplo do duelo, em que participam duas pessoas. Suponha que, certa manhã, os senhores X, Y e Z encontram-se para resolver uma disputa, em que, a igual distância uns dos outros, atirarão com pistolas, um após o outro, um único tiro por vez, obedecendo a certa ordem, até que apenas um permaneça vivo. Sabe-se que o senhor X acerta um tiro em cada três, que o senhor Y acerta dois tiros em cada três e que o senhor Z nunca erra. Para ser justo, o trielo será iniciado com o senhor X atirando, seguido do senhor Y, se ainda estiver vivo, depois pelo senhor Z, se ainda estiver vivo, e assim sucessivamente até restar vivo apenas um desafiante. Para aumentar suas chances de sobrevivência na disputa, o melhor que o senhor X deverá fazer, do ponto de vista lógico, é

Alternativas
Comentários
  • se X atirar para o alto, não correrá o risco de ficar mais duas rodadas sem acertar um tiro, pois ele só acerta 1 em 3, se acertar o tiro em Y, Z o matará(o X), pois Z nunca erra. Se acertar o tiro em Z, Y o matará, pois Y acerta 2 em cada 3 tiros, ou seja, até o X poder acertar novamente, Y o terá acertado, ganhando o trielo.Atirando pra cima, restarão 2 tiros, um tiro, ele acertará, o outro ele errará, fica com 50% de chance de ganhar. Caso Y acerte Z, X poderá atirar novamente, mesmo se X errar, o próximo tiro será certeiro. Já Y, que agora só dispõem de 1 acerto para 2 tiros, estará em pé de igualdade com X.
  • Admiro muito o raciocínio de pessoas como você, Paullo. Um dia eu espero chegar a esse nível. Parabéns!
  • Achei a questão mal elaborada: ela não diz traz o famoso termo "nessa ordem". Não há como saber a ordem dos erros e dos acertos.
  • Chances:
    . X = 1 em 3 tiros
    . Y = 2 em 3 tiros
    . Z = 3 em 3 tiros

    CORRETA - LETRA E
    Se X atirar para o chão (errando o tiro) sua possibilidade passa a ser de acertar 1 em 2 tiros...
    Y pela lógica irá atirar em Z para tentar eliminá-lo, porque se Z continuar vivo e o mirar, ele irá morrer (Z nunca erra), assim teremos 2 hipóteses:
    1. Se Y acertar Z sua possibilidade passa a ser de acertar 1 em 2 tiros (Se igualando a X. Porém X tem vantagem, pois na próxima rodada será o primeiro a efetuar o tiro)
    2. Se Y não acertar Z sua possibilidade passa a ser de acertar 2 em 2 tiros. Sabendo disso e para eliminar o que melhor atira, Z atirará em Y, eliminando-o.
    Assim X terá que contar com a sorte de acertar o próximo tiro, matando Z, porque se errar, ele morre (Z nunca erra).

  • Letra E) Correta

    Ele errando o tiro, automaticamente força o y a a tirar na proxima rodada em Z ( visto q se y n atirar em z, morrerá nas mão se z na 3 * rodada).
     

     

  • Confesso que tentei fazer essa questão algumas vezes e pra mim ela não faz muito sentido. A única coisa que pra mim talvez justificasse a alternativa correta é o fato de que x é o pior atirador e por isso a chance dele errar o alvo é maior do que a possibilidade de acertar. Também não há garantia de que ele acerte Z como sugere algumas das alternativas. Em tese, y tem mais chance de fazê-lo.

  • Quanto mais o X tiver chance de atirar, maior as chances de enfrentar o Y e Z de igual. Sendo a probabilidade igual ao Y no próximo duelo, e ao Z no terceiro duelo.


ID
49201
Banca
FUNIVERSA
Órgão
PC-DF
Ano
2009
Provas
Disciplina
Raciocínio Lógico
Assuntos

Uma loja vende tintas em dez cores diferentes. Se cinco clientes compram uma lata de tinta cada um, é correto afirmar que

Alternativas
Comentários
  • Não é a toa que esta questão foi anulada, há duas respostas corretas (D e E).

    A alternativa D é um ótimo exercício. A probablidade de pelo menos 2 cores serem iguais é superior a 80%


ID
50305
Banca
CESPE / CEBRASPE
Órgão
DPF
Ano
2009
Provas
Disciplina
Raciocínio Lógico
Assuntos

De acordo com o jornal espanhol El País, em 2009 o
contrabando de armas disparou nos países da América Latina,
tendo crescido 16% nos últimos 12 anos. O crime é apontado
como o principal problema desses países, provocando uma
grande quantidade de mortes. O índice de homicídios por
100.000 habitantes na América Latina é alarmante, sendo, por
exemplo, 28 no Brasil, 45 em El Salvador, 65 na Colômbia, 50
na Guatemala.

Tendo como referência as informações apresentados no texto
acima, julgue o item que se segue.

Se, em cada grupo de 100.000 habitantes da Europa, a probabilidade de que um cidadão desse grupo seja assassinado é 30 vezes menor que essa mesma probabilidade para habitantes de El Salvador ou da Guatemala, então, em cada 100.000 habitantes da Europa, a probabilidade referida é inferior a 10-5.

Alternativas
Comentários
  • Comentário: Questão fácil. Assunto: Probabilidade (Regra do ou).Probabilidade (Brasil) = 28/100000 = 0,00028 Probabilidade (Guatemala) = 50/100000 = 0,0005Probabilidade (Colômbia) = 65/100000 = 0,00065 Probabilidade (El Salvador) = 45/100000 = 0,00045Probabilidade (Europa) = (Probabilidade (El Salvador) + Probabilidade (Guatemala))/30Probabilidade (Europa) = (0,00045 + 0,0005)/30Probabilidade (Europa) = 0,00095/30 = 0,00003167 = 3,167 x 10-5(este valor é superior a 10-5)
  • 30/45=1,5 É A PROBABILIDADE EM RELAÇÃO EL SALVADOR E EUROPA

    30/50 = 1,66 QUE É A PROBABILIDADE EM RELAÇÃO GUATEMALA EUROPA .

    JÁ QUE É 30 VEZES MENOR ESSE INDICE DA EUROPA SE COMPARADO A ESSES DOIS LUGARES. SOMANDO 1,5+1,66=3,16.

    SE DIVIDIRMOS 1/100000=0,00001, PORTANTO 3,16 NÃO É INFERIOR A 0,00001.

  • Vamos lá...

    Mais uma vez estão complicando nas explicações por não chegarem na resposta certa.

    Compara-se a Europa com a Guatemala OU El Salvador e não os dois países juntos.

    Vamos então esquecer o menor valor e comparar a Europa com o maior valor, ou seja, 50 (Guatemala). Se for inferior a 10 elevado a 5ª potência negativa já conseguiremos responder a questão, certo pessoal?

    Então, dividindo 50 homicídios (Guatemala) por 30 o valor será de 1,6 homicídios por 100.000 habitantes na Europa. E isso significa 0,000016 ou seja, MAIOR que 0,00001.
  • Olá !! O comentário do Erik foi quase perfeito. Vejamos:
    Europa                  El Salvdor                                              Guatemala
    30X menor             45/100.000=0,45x10^-5                        50/100.000=0,50X10^-5


    Dessa fora como temos a conjunção OU ( tanto faz um qto outro) pegaremos o MENOR VALOR e não o maior, pois a qeustão afirma que a europa tem índice menor que 10^-5. Então pegaremos o menor valor e dividiremos por 30. (30 X menor)

    Daí temos: 0,45x10^-5 / 30 = 0,015 X10^-5.

    Falsa a questão.
  • Eu fiz diferente:

    A probabilidade de El salvador OU Guatemala: 0,00045+0,0005-(0,00045*0,0005)= 0,000949  

    (OU: soma das duas probabilidades menos os elementos comuns das duas)

    Como Europa é 30 vezes menors que a probabilidade de um OU outro fiz: 0,000949 /30= 0,000316 = 3Imagem 025.jpgque é maior que 10Imagem 025.jpg.

    Logo, no meu raciocínio, questão ERRADA mesmo.

    Alguem sabe REALMENTE, uma forma diferente? Eu fiz assim, quem sabe não esteja certo tbm.

  • Nem é fácil esta questão, segue mais uma resolução:

    Probabilidade para habitantes de El Salvador ou da Guatemala: 45 a 50 para cada 100.000 habitantes

     
    Probabilidade para habitantes da Europa: (45 a 50)/30 para cada 100.000 habitantes; que resulta em 1,5 a 1,67 para cada 100.000 habitantes
    (1,5 a 1,67)/100.000 = (1,5 a 1,67)/105 = (1,5 a 1,67) x 10-5 que é maior que 10-5.

    Logo, este item está errado.

    FONTE: http://raciociniologico.50webs.com/ESCRIVAOPF2009/ESCRIVAOPF2009.html#Questão 01
  • Galera, tem horas que eu entro nas explciacoes de raciocinio logico e parece que vai dar no na cabeca de tanta complciacao que as pessoas arrumam para explicar algumas coias que me parecem bem obvias
    Vou tentar explicar com ofiz a questao ok?
    vamos la:
    45 a cada 100.000 em El Salvador o que equivale a  dizer que e' 45/100.000 correto?
    50 a cada 100.000 na Guatemala o que equivale a  dizer que e' 50/100.000 correto?
    mas perae gente...
    45/100.000 = 45 x 10-5 e 50/100.000 = 50 x10-5 correto?
    entao como a pergutna usa o OU devemos somar essas duas potencias
    45 x 10-5 + 50 x10-5 correto?
    como a possibilidade 'e 30 vezes menor devemos divir essa soma por 30 e encontraremos o resultado, correto?
    ( 45 x 10-5 + 50 x10-5 )/30
    mas perai gente... independente do resultado, ja da pra ver que vai dar mais que 10-5 correto? Entao daqui nao seria nem necessario fazer o calculo para concluir que a acertiva e' falsa
    mas vamos la para ficar mais claro...
    da para smplificar o 30 com o 45 ficando:
    (15 x 10-5 + 50 x10-5 )/10
    agora da para simplificar o 10 com o 50 ficando:
    15 x 10-5 + 5 x10-5 ... quanto da essa conta??? nao faco a minima ideia.. .a unica coisa que sei e' que e' maior que 10-5
    epero que tenha simplificado para alguns
    bons estudos

  • Meu amigo Vascaíno, Leandro, você estava indo muito bem até se perder nos cálculos de: ( 45 x 10-5 + 50 x10-5 )/30
    NÃO DÁ PRA SIMPLIFICAR 30 POR 45!!! REPARA QUE EU TENHO UMA SOMA.
    Seria como querer simplficar isto: (45x + 50x)/30
    eu simplesmente somo 45 + 50, ficando 95.
    10-5/30
    Aí eu teria 95/30 X 
    10-5  número certamente maior do que  10-5
  • Meu Deus... cada coisa que leio.
    • 10-5 nada mais é que 1/100.000 (1 a cada 100.000 habitantes)
    • se é 30 vezes menor, será 45/30 ou 50/30 a cada 100.000 habitantes
    • ambas as frações são maiores que 1
    Fim de papo
  • Resolução de raciocínio lógico da prova de escrivão da PF:
    http://www.equipealfaconcursos.com.br/blog/2013/04/arrebentando-com-a-cespeunb-2013-rlm/
  • Dica de material de estudo:

    https://mega.co.nz/#F!phsmlRDT!o3Wv4YfIei25f_y2SZMwAA


  • De acordo com o enunciado, em cada grupo de 100.000 habitantes da Europa, a probabilidade de que um cidadão desse grupo seja assassinado é 30 vezes menor que essa mesma probabilidade para habitantes de El Salvador ou da Guatemala, assim:



     

    A resposta é: Errado.



  • Galera escreve uma bíblia para ensinar algo simples ¬¬

    Homicídio em EL salvador = 45/100.000
    Homicídio na Europa = El salvador dividido por 30 ou (45/30) =1,5/100.000

    1/100.000 = 10(elevado a -5)

    1,5 . 10Imagem 025.jpg>>>>>>> 10 (elevado a -5)

  • eu nao entenderia nunca aqui no QC que essa droga de digitalização era dez elevado a menos cinco, pq nao escreveram assim? (10)^-5

    seria bem mais oportuno.

  • Galera, sem complicar:

    Antes de qualquer coisa, tem que ficar claro pra você que 10^(-5)=1/100.000, isso é aritimética básica, você tem que saber.

    Probabilidade de alguém ser assassinado em El Salvador: 45/100.000

    Probabilidade de alguém ser assassinado na Europa: (45/100.000)/30, faça a divisão entre frações e chegará ao resultado simplificado de 3/200.000.

    Agora, como você compararia 3/200.000 com 1/100.000 para saber qual probabilidade é maior?.

    Não precisa nem fazer divisão. Olhe para a fração 1/100.000, multiplique o numerador e o denominador por 2 (repare que a fração se mantém a mesma depois dessa operação, é como se tivéssemos multiplicado a fração por 1), você chegará à fração 2/200.000. Qual é maior? 2/200.000 ou 3/200.000? Se não acertar agora, temos um problemão hein.

  • Sinceramente eu acredito que o "ou" da questão não é pra somar as probabilidades de El Salvador e da Guatemala, conforme a explicação do professor Vinícius Werneck, pois se trata de probabilidade distintas, universos distintos, estou acreditando mais na explicação dos colegas de que tem que pegar a probabilidade do maior entre El Salvador e Guatemala, que é a da Guatemala, divididir por 30 e verificar se o resultado é maior ou menor que 10 ^ -5

     

    Porque tem que ser a probabilidade da maior?

    Porque se a probabilidade maior fosse 30 vezes menor que a quantidade da Europa, então obrigatoriamente a probabilidade menor também seria

  •  

                                                                             50           +          45                       

    P(Europa) = P(guatema) + P(El Salvador)        100 000              100 000      =     0,0005 + 0,00045​     = 0,00095  =                     

                                         30                                         30                                             30                        30

    = 0,000032 > 0,00001

     

    gab errado

  • Questão que requer um pouco de atenção à leitura do enunciado, vejamos:

     

    Ela quer saber se na Europa a cada 100.000 habitantes há mais homicídios, que 30 vezes a fração resultante dos homicídios de El Salvador ou da Guatemala, simplificando:

    10^-5 = 1/100.000

     

    30/45 "El savador" = 1,5

    30/50 "Guatemala" = 1,66

     

    ou seja ambas são maiores que 1, apresentada no enunciado "10^-5", por isso, assertiva errada.

  • 10 elevado a -1 é 0,1 pois dividindo 1 por 10 temos 1/10 (um décimo) ou 0,1

    então:

    10 elevado a -2 será 0,01

    10 elevado a -3 sera 0,001

    10 elevado a -4 sera 000001

    10 elevado a -5 sera 0,00001

    Regrinha básica... a vírgula corre para a esquerda conforme dividimos por 10.

  • Bom, eu fiz da seguinte maneira:

    Probabilidade El Salvador --> 45/100.000 = 0,00045

    Probabilidade Guatemala --> 50/100.000 = 0,00050

    • Agora, veremos quais valores teremos ao dividi-los por 30 (já que na Europa é 30 vezes menor)

    0,00045/30 = 0,000015 = 15^-5

    0,00050/30 = 0,000016 = 16^-5

    • Portanto, diante dos dados expostos, a probabilidade de homicídio na Europa não será inferior a 10^-5.

    Acredito que seja esse o raciocínio.

    Corrijam-me caso haja algum equívoco.

    Abraços!

  • Pensei dessa forma:

    Na Guatemala para cada 1 habitante entre 5000 é assassinado.

    Ora, se na Europa esse número é 30x Menor, então:

    Para cada Habitante assassinado da Europa teremos 150.000 -> 1/ 150000 = 15.10^-4 = 0,0015 e este número é maior que 10^-5 = 0,00010.

  • A probabilidade para habitantes de El Salvador ou da Guatemala = 95/100.000

    Se a questão fala que a probabilidade de que um cidadão da Europa seja assassinado é 30 vezes menor em um mesmo numero de habitantes, então, é só dividir a probabilidade acima por 30.

    P = (95/100.000)/30 = 3,16 x 10^-5

  • Quando a questão informou que algo é inferior a um exponente negativo, já pode marcar errado, o resultado vai ser próximo de zero. 10 elevado a -5 = 0,00001, quase impossível algo ser inferior a isso

  • Era só se ligar que está comparando a Europa com a Guatemala OU El Salvador e dizendo que os valores são iguais. Hora, a própria questão diz em seu enunciado que a Guatemala e El Salvador tem taxas distintas.


ID
53653
Banca
CESPE / CEBRASPE
Órgão
TRT - 17ª Região (ES)
Ano
2009
Provas
Disciplina
Raciocínio Lógico
Assuntos

Em 2007, no estado do Espírito Santo, 313 dos 1.472
bacharéis em direito que se inscreveram no primeiro exame
do ano da Ordem dos Advogados do Brasil (OAB) conseguiram
aprovação.
Internet: (com adaptações).

Em 2008, 39 dos 44 bacharéis provenientes da
Universidade Federal do Espírito Santo (UFES) que fizeram a
primeira fase do exame da OAB foram aprovados.
Internet: (com adaptações).

Com referência às informações contidas nos textos acima, julgue
os itens que se seguem.

Se um dos bacharéis em direito do estado do Espírito Santo inscritos no primeiro exame da OAB, em 2007, fosse escolhido aleatoriamente, a probabilidade de ele não ter sido um dos aprovados no exame seria superior a 70% e inferior a 80%.

Alternativas
Comentários
  • 1472 - 313 = 11591159/1472 X 100 = 78,7...
  • a probabilidade dele estar entre os aprovados é ~21%, entao, não estar entre os aprovados seria ~79%
  • 313 EQUIVALE A QUANTOS POR CENTO DE 1.472 ? --> PARA SE CHEGAR AO TOTAL DE APROVADOS
    ISTO É
    313 = X/100 * 1.472
    313= 1472X/100
    313=14,72X
    313/14,72=X
    21,26=X --> TOTAL DE APROVADOS. O QUE SE QUER É O TOTAL DE NÃO APROVADOS
    100-21,26 = 78,74 --> ESTÁ ENTRE 70 E 80
  • Dos 1472 bacharéis em direito do estado do Espírito Santo, 313 foram aprovados no exame da OAB. Logo, os 1159 bacharéis restantes não foram aprovados. A questão quer saber se a probabilidade dos que não foram aprovados é superior a 70% e inferior a 80%.


    P (não aprovados OAB) = 1159/1472 > 70%

    1159/1472 > 70/100 (corta zero com zero para facilitar)

    1159 x 10 = 11590

    1472 x 7 = 10294

    11590 > 10294 (certo)


    P (não aprovados OAB) = 1159/1472 < 80%

    1159/1472 < 80/100 (corta zero com zero para facilitar)

    1159 x 10 = 11590

    1472 x 8 = 11776

    11590 < 11776 (certo)


    Resposta: certo. 

  • 1472-313 = 1159 são o total de reprovados.

    70% de 1472 = 1030 que é < 1159

    80% de 1472 = 1175, 6 que é > 1159

  • Porcentagem é sempre regra de 3.

    Como quer saber os que não foram aprovados.

    1472 (total) - 313 (aprovados) = 1159 (reprovados)

    1472 é 100% dos que fizeram e quero saber quanto % é 1159, então:

    1472 ------ 100

    1159 ------- x (porque não sei quanto % é)

    Daí, só multiplicar cruzado.

    1472x = 115900

    A partir disso, você tem que dividir o valor que está com x pelo valor depois do sinal de igual:

    X= 115900 ÷ 1472

    X= 78,73 %

    A questao disse que o valor será maior que 70% e menor que 80%. Por isso está CERTO.

    ABRAÇO, ESPERO TER AJUDADO! (:


ID
53656
Banca
CESPE / CEBRASPE
Órgão
TRT - 17ª Região (ES)
Ano
2009
Provas
Disciplina
Raciocínio Lógico
Assuntos

Em 2007, no estado do Espírito Santo, 313 dos 1.472
bacharéis em direito que se inscreveram no primeiro exame
do ano da Ordem dos Advogados do Brasil (OAB) conseguiram
aprovação.
Internet: (com adaptações).

Em 2008, 39 dos 44 bacharéis provenientes da
Universidade Federal do Espírito Santo (UFES) que fizeram a
primeira fase do exame da OAB foram aprovados.
Internet: (com adaptações).

Com referência às informações contidas nos textos acima, julgue
os itens que se seguem.

Considerando que, na primeira fase do exame da OAB de 2008, 87,21% dos bacharéis em direito da Universidade Federal de Pernambuco (UFPE) tenham sido aprovados, a probabilidade de se escolher ao acaso um dos aprovados entre os bacharéis da UFPE que fizeram esse exame será maior que a probabilidade de se escolher ao acaso um dos aprovados entre os bacharéis da UFES e que também fizeram o exame da OAB.

Alternativas
Comentários
  • % de aprovados da UEFS = 39/44 x 100 = 88,63... portanto maior.
  • O Percentual de aprovados da UFPE(87,21%) NÃO é maior que o percentual de aprovados da UFES(88,63%), mas a resposta não é essa, pois a questão não informa o número de aprovados na UFPE, apenas o percentual de aprovados, logo não podemos afirmar nada.(Portanto ERRADO).
  • E acho que essa questão ficou meio confusa e daria recurso. pois o enunciado fala de bachareis de 2007 e de 2008 da UFES. Se fizermos as contas, 313 dos 1472 aprovados de 2007 equivale a 21,2% e em 2008 39 dos 44 aprovados equivale a 88% se somarmos tudo daria 313+39=352 aprovados em 1472+44=1516 bacharéis, que equivale a 23% que é menor que 87%.
  • fiz assim 39/44=87,21%

    logo 5/44= 0,11...=11%

     

  • Simplificando.

    UFPE- 87,21% Aprovados

     

    UFES- 39/44 Aprovados= 39/44: 88,63%

     

    Resultado ao escolher aleatoriamente os  aprovados entre os bacharéis da UFES que fizeram esse exame  a porcentagem será maior que entre os bacharéis da UFPE.

     

    ERRADO

  • Faz pelo método cruzado, é mais fácil multiplicar do que dividir.

    87,21 é o mesmo que dizer: 87/100 > 39/44 . Agora multiplica cruzado e veja se é maior, a conta ficará: 3.828 > 3.900 podemos ver que não é maior, logo item ERRADO.


ID
54010
Banca
CESPE / CEBRASPE
Órgão
TRT - 17ª Região (ES)
Ano
2009
Provas
Disciplina
Raciocínio Lógico
Assuntos

Julgue os itens seguintes, acerca de contagem e probabilidades.

Se, em um concurso público com o total de 145 vagas, 4.140 inscritos concorrerem a 46 vagas para o cargo de técnico e 7.920 inscritos concorrerem para o cargo de analista, com provas para esses cargos em horários distintos, de forma que um indivíduo possa se inscrever para os dois cargos, então a probabilidade de que um candidato inscrito para os dois cargos obtenha uma vaga de técnico ou de analista será inferior a 0,025.

Alternativas
Comentários
  • 46vagas por 4140 SOMADO com 99vagas por 7920. A resposta é 0,0236.
  • 1) P(A) ou P(B)- eventos excludentes: P(A) + P(B)- eventos não excludentes: P(A) + P(B) - P(A e B)2) P(A) e P(B)- eventos independentes: P(A) x P(B)- eventos dependentes: P(A) x P(B/A) Resposta correta da questão:- 46/4.140 + 99/7.920 - propabilidade das pessoas estarem escritos nos dois.- Assim, teremo com certeza menos que 0,0236(valor exato não é possível de calcular, pois faltam maiores dados)
  • P(A U B) = [P(A) + P(B)] - [P(A) * P(B)] , então, P(A U B) = [(46/4140 + 99/7920) - (46/4140 * 99/7920)] 

    logo,

    P(A U B) = (0,0111 + 0,0125) - (0,0111 * 0,0125)

    P(A U B) = (0,0236) - (0,0001)

    P(A U B) = 0,0235

    0,0235 < 0,025
  • Rafael, pelo que sei os eventos são mutuamente excludentes... não há intersecção!!!
  • Acredito que amigo acima tenha se equivocado. os eventos nao sao excludentes, pois ha a possibilidade de se inscrever para os 2 cargos, conforme explicito na questao. 
  • Acredito que não há de se considerar a intersecção, o total de inscritos é por prova, em nada interfere se o candidato fez a outra prova também.

    Chance de ser aprovado numa prova: 46 vagas/4140, 

    Chance de ser aprovado na outra prova: 99/7920.

    Chance de ser aprovado em uma ou outra: 46/4140 + 99/7920 = 0,0236

  • Total de vagas= 145

    Total de inscritos=12060

    145/12060= 0,012

  • 4140 inscritos para tecnicos -> 46 vagas

    7920 inscritos para analistas -> 99 vagas

    Desses tecnicos, de 0 ate 4140 podem ter feito a prova de analista.

    Considerando a pior situacao: 4140 fizeram ambas as provas

    P1 = 46/4140=0,01111111 (Tecnico)

    P2 = 99/4140=0,023913043 (Analista)

    Lembrando: inscrito para os dois cargos (para os 2, ou um ou outro?) obtenha uma vaga de técnico ou de analista -> olhando a pior situacao para a interseccao.

    P=P1+P2 = 0,035024154>0,025

    ERRADO -> Teria que ser ANULADA

     

  • Técnico: 46/4140 = 0,011

    Analista: 99/7920 = 0,012

    0,011 + 0,012 = 0,023

    0,023 < 0,025

    CERTO

  • Eu resolvi fazendo a razão do numero de inscritos pelo número de vagas de cada cargo!!

    A concorrência ficou:

    Para técnico : 90 candidatos para cada 1 vaga

    Para analista : 80 candidatos para cada 1 vaga

    Calculando a probabilidade :

    1/90 + 1/80 = 17/720 = 0,023

  • A probabilidade dos que ESTEJAM INSCRITOS NOS 2 passe em uma ou outra. Precisaria sim da interseção e pelo enunciado não é possível saber qual a interseção


ID
54016
Banca
CESPE / CEBRASPE
Órgão
TRT - 17ª Região (ES)
Ano
2009
Provas
Disciplina
Raciocínio Lógico
Assuntos

Julgue os itens seguintes, acerca de contagem e probabilidades.

Se, em determinado tribunal, há 54 juízes de 1.º grau, entre titulares e substitutos, então a quantidade de comissões distintas que poderão ser formados por 5 desses juízes, das quais os dois mais antigos no tribunal participem obrigatoriamente, será igual a 35.100.

Alternativas
Comentários
  • Como a quetao refere-se a comissoes, entao nao importa a ordem em que os juizes serao postos.Assim utilizamos combinacao de 52 para escolher 3, pois devemos excluir os 2 juizes ja selecionados.52/3!49! = 52.51.50/6 = 22100
  • Cheguei ao mesmo resultado, Rebeca! Da maneira que o "Wiwi" fez está incorreto. Pois ele considerou uma Permutação (troca) de lugares entre os juízes como outra comissão. Por exemplo, uma comissão formada pelos juízes A1, A2, J52, J51 e J50 é igual a uma comissão formada pelos juízes A1, A2, J50, J51 e J52.
  • Retiramos os dois mais antigos juízes.Ficaram 52 juízes e 3 VagasEntão a quantidade de comissões distintas que poderão serformados por 3 desses juízes (mais 2 antigos) seráC(52,3)=52!/(3!*49!)=50*51*52/6=22100
  • Então esta questão foi anulada?

  • Data vênia, caros colegas, o colaborador Walter Prestes deu a informação correta. Não se trata de permutação. É probabilidade simples.

    São 3 vagas a serem preenchidas por 52 pessoas. Esqueçam as outras 2 vagas, que não mudam.

    Logo, 52x51x50 possibilidades, que dá 132.600 possibilidades distintas. A questão está ERRADA e não foi anulada por este motivo!

    Espero ter ajudado.

  • O comentário do Walter está correto!!!! e consta como ruim, os outros errados e estão como bom, tomem cuidado !!!!
  • Essa questão trata de uma combinação de 52 elementos 3 a 3, o que dá 22100 formações de comissões!!! Não importa a ordem dos elementos nesse caso!!! Portanto, a resolução do Walter e das pessoas que o acompanharam está errada... A resolução da questão 28 do link http://pt.scribd.com/doc/39188726/Simulado-XXVIII-PCF-Area-6-PF-CESPE reforçou ainda mais essa ideia!!! Deem uma conferida!

    Bom estudo,

    Caros colegas.
  • Demorou para eu eu entendesse isso... 
    A ordem não importa
    É uma combinação em que devem ser escolhidos 3(já escolhi 2, não preciso incluí-los na combinação) de 52 (54 - 2 mais antigos, que ficam de fora)
    C 52,3 = 52X51X50 / 3X2X1
    C 52,3 = 22.100
    ERRADO
  • SAVIO isso e' uma formula cara...
    Procura no google Analise Combinatoria, Combinacao e procura alguma explicacao ou video aula que vc vai entender o porque do 6...
    Combinacao de 52 3 a 3
    (C352)=       n!       onde n=52 e k=3
                  K!(n-k)!

    o gabarito 22.100 esta corretissimo
  • Apenas reforcnado o comentario, a explicacao do wlater esta errada, pois o raciocionio que ele utilizou foi para PERMUTACAO, e nao cabe formula de permutacao nesse tip ode questao, pois no caso a permutacao considera uma comissao formada por 5 juizes, A, B, C, D , E  diferente de uma comissao formada pelos mesmos 5 juizes em ordem diferente, pore exemplo comissao 1 =A, B, C ,D ,E em permutacao 'e diferente da comissao 2 = D,C , A ,B, E, o que no caso, nao corresponde a verdade. Pois as duas comissoes sao identicas mudando apeans a ordem. Nesses casos, em que a ordem nao importa, nao pode ser resolvido o problema por permutacao e sim por combinacao. Por isso o gabarito esta correto pois temos assim:
    5 juizes sendo dois ja definidos como os mais antigos sobram 52, 51 e 50 possibilidades para as 3 comissoes restantes
    A , B, 52 , 51, 50
    Todavia, como ja foi dito, esse problema nao pode ser resolvido pelo principio fundamental da contagem onde simplesmente mutiplicam-se  os numeros, pois a ordem do arranjo nao importa. Entao deve ser resolvido por COMBINACAO
    ou seja, Combinacao de 52 3 a 3
    C353 = 52 x 51 x 50= 22.100
                       3!
    52x51x50 (simplifica 51 com 3 e 50 com 2)
       3x2
    pode calcular ai... 52 x 17 x25 e ve se nao vai dar 22.100
    Quem acertou essa questao utilizando permutacao acertou na cagada
  • C52.3

  • Gabarito: Errado.

    Você tem 54 juízes e precisa formar um grupo com cinco. Desses cinco, dois já foram escolhidos que são os mais antigos. Então restam 52 juízes e 3 vagas. A ordem de distribuição não importa, então se trata de uma combinação.

    C52,3 = (52 x 51 x 50)/(3 x 2 x 1) = 52 x 17 x 25 = 22100.

    Bons estudos!


ID
67165
Banca
ESAF
Órgão
Receita Federal
Ano
2009
Provas
Disciplina
Raciocínio Lógico
Assuntos

Para acessar a sua conta nos caixas eletrônicos de determinado banco, um correntista deve utilizar sua senha constituída por três letras, não necessariamente distintas, em determinada sequência, sendo que as letras usadas são as letras do alfabeto, com exceção do W, totalizando 25 letras. Essas 25 letras são então distribuídas aleatoriamente, três vezes, na tela do terminal, por cinco teclas, em grupos de cinco letras por tecla, e, assim, para digitar sua senha, o correntista deve acionar, a cada vez, a tecla que contém a respectiva letra de sua senha. Deseja-se saber qual o valor mais próximo da probabilidade de ele apertar aleatoriamente em sequência três das cinco teclas à disposição e acertar ao acaso as teclas da senha?

Alternativas
Comentários
  • As vezes, o excesso de informações no enunciado acaba atrapalhando o raciocínio.O que devemos atentar aqui é que temos 5 teclas e independente de quais letras estão na tecla (5 letras por tecla), temos uma chance em 5 para acertar a letra da senha, logo temos:1/5 X 1/5 X 1/5 = 1/125 = 0,008
  • Resposta Letra C de COMO LÓGICA PODERIA NÃO EXISTIR..rs - Brincadeiras a parte, segue abaixo a reolução da questão.

    Fonte: http://beijonopapaienamamae.blogspot.com.br/2010/01/dia-22-de-janeiro-questao-22.html

    Bom, a questão parece ser mais complicada do que realmente é. Olha só, as 25 letras estão distribuídas em 5 teclas, cada tecla com 5 letras. Então, o que ele deve escolher é A TECLA CERTA, ou seja, a tecla que tem a letra da senha.
      Então, deveremos escolher, dentre as 5 teclas da tela, qual que tem a letra da senha.   P (acertar a tecla) = 1 (tecla que preciso acertar) / 5 (quantidade total de teclas) = 1/5   Como são 3 letras, teremos:   P = 1/5 . 1/5 . 1/5 = 1/125 = 0,008
  • Gabarito: Letra E

    Na primeira tecla apertada ao acaso temos 5 das 25 letras disponíveis. Portanto, a chance dessa tecla conter a primeira letra da senha (que pode ser qualquer uma das 25) é de 5 em 25, isto é, P = 5/25 = 1/5.

    Da mesma forma, a chance da segunda tecla apertada ao acaso conter a segunda letra da senha é de 5 em 25, ou seja, P = 1/5. Analogamente, a chance da terceira tecla apertada conter a terceira letra da senha é P = 1/5.

    A chance de acertar a primeira E acertar a segunda E acertar a terceira letras da senha é dada pela multiplicação dessas probabilidades, pois temos três eventos independentes entre si:
    P = 1/5 x 1/5 x 1/5 = 0,008


    Fonte: ESTRATÉGIA CONCURSOS

  • Se em cada tecla vc tem a possibilidade de escolha de duas letras, então, 1/2 e neste caso ele apertou duas vezes a mesma tecla, fica 1/4, na seguinte ele apertou outra tecla com a mesma possibilidade, 1/2, resolvendo 1/4 . 1/2 = 1/8.

  • São 3 (três) letras, dentre 25 (vinte e cinco). Porém, você vai digitá-las uma de cada vez e pode repeti-las, ou seja, são eventos independes.

    Quando você for tentar o primeiro dígito da senha, terá a possibilidade de 1 (um) acerto entre as 5 (cinco) teclas existentes (1/5) e assim também ocorre na escolha dos outros dois dígitos. Logo,

    1/5 x 1/5 * 1/5 = 1/5 = 0,2 * 0,2 * 0,2 = 0,08


ID
67171
Banca
ESAF
Órgão
Receita Federal
Ano
2009
Provas
Disciplina
Raciocínio Lógico
Assuntos

Três amigas participam de um campeonato de arco e fl echa. Em cada tiro, a primeira das amigas tem uma probabilidade de acertar o alvo de 3/5, a segunda tem uma probabilidade de acertar o alvo de 5/6, e a terceira tem uma probabilidade de acertar o alvo de 2/3. Se cada uma das amigas der um tiro de maneira independente dos tiros das outras duas, qual a probabilidade de pelo menos dois dos três tiros acertarem o alvo?

Alternativas
Comentários
  • probabilidade de somente a 1 acertar = 3/5 x 1/6 x 1/3 = 3/90probabilidade de somente a 2 acertar = 2/5 x 5/6 x 1/3 = 10/90probabilidade de somente a 3 acertar = 2/5 x 1/6 x 2/3 = 4/90probabilidade de nenhuma acertar = 2/5 x 1/6 x 1/3 = 2/90A probabilidade de pelo menos dois tiros acertarem o alvo é a probabilidade total de acertos (90) menos a soma das probabilidades acima 90/90 - 19/90 = 71/90
  • probabilidade da 1ª e da 2ª acertar (e da 3ª errar) = 3/5 x 5/6 x 1/3 = 15/90

    probabilidade da 1ª e da 3ª acertar (e da 2ª errar) = 3/5 x 1/6 x 2/3 = 6/90

    probabilidade da 2ª e da 3ª acertar (e da 1ª errar) = 2/5 x 5/6 x 2/3 = 20/90

    probabilidade de todos acertarem = 3/5 x 5/6 x 2/3 = 30/90

    A probabilidade de pelo menos dois tiros acertarem o alvo é (15+6+20+30)/90 = 71/90


  • RESPOSTA LETRA ==> B.

    Fonte: http://beijonopapaienamamae.blogspot.com.br/2010/01/dia-05-de-janeiro-questao-05_05.html



    Bem, a questão nos pede 'a probabilidade de pelo menos dois dos três tiros acertarem o alvo'. O que isso quer dizer?


    Que, podem ser as seguintes possibilidades:
    1) a primeira errar e as outras duas acertarem;
    2) a segunda errar e as outras duas acertarem;
    3) a terceira errar e as outras duas acertarem;
    4) as três acertarem.

    A grande sacada da questão é que você tem que incluir a probabilidade do acerto E TAMBÉM A PROBABILIDADE DO ERRO!     Assim, temos: 1) a primeira errar e as outras duas acertarem = 2/5 . 5/6 . 2/3 = 20/90 2) a segunda errar e as outras duas acertarem = 3/5 . 1/6 . 2/3 = 6/90 3) a terceira errar e as outras duas acertarem = 3/5 . 5/6 . 1/3 = 15/90 4) as três acertarem = 3/5 . 5/6 . 2/3 = 30/90   Probabilidade = (20 + 6 + 15 + 30)/90 = 71/90
  • Para que pelo menos dois tiros acertem o alvo, é preciso que uma dessas situações ocorra:

    1. As três amigas acertem. Aqui, a probabilidade é dada pela multiplicação das três probabilidades:

    2. A primeira e segunda amigas acertarem, e a terceira errar. Note que a probabilidade da terceira errar é de 1 – 2/3 = 1/3. Assim:

    3. A primeira e terceira amigas acertarem, e a terceira errar. Note que a probabilidade da segunda errar é de 1 – 5/6 = 1/6. Assim:

    4. A segunda e terceira amigas acertarem, e a primeira errar. Note que a probabilidade da primeira errar é de 1 – 3/5 = 2/5. Assim:

           Assim, a probabilidade de pelo menos 2 acertarem é:

    P = P + P + P + P

    P = 1/3 + 1/6 + 1/15 + 2/9

    P = 30/90 + 15/90 + 6/90 + 20/90

    P = 71/90

    Resposta: D

  • gente, minha esposa não conseguiu compreender o porque de 1-5/6 ser igual a 1/6 e não igual a 4/6, assim como o mesmo raciocínio dos outros cálculos. Poderiam explicar ?

  • tássio castro, o 1 é a mesma coisa que 6/6, perceba que simplificando essa fração você voltará para o número 1. Sem ofensa mas o problema da sua esposa é a matemática básica, subtração de frações você precisa dar um jeito de colocar o denominador das duas frações iguais "denominador é o número de baixo"

    Dessa forma:

    1 - 5/6 é o mesmo que 6/6 - 5/6 = 1/6

    Basicamente o que fizemos foi transformar o 1 em 6/6 pra que o denominador fique igual ao da outra fração '5/6'

  • Não tem como acertar pela técnica que calcula o único resultado que o enunciado não quer? E E E

  • Rodrigo Teles

    Ele não quer E E A, E A E e A E E

    Pode fazer assim tb


ID
67552
Banca
ESAF
Órgão
Receita Federal
Ano
2009
Provas
Disciplina
Raciocínio Lógico
Assuntos

O número de petroleiros que chegam a uma refinaria ocorre segundo uma distribuição de Poisson, com média de dois petroleiros por dia. Desse modo, a probabilidade de a refinaria receber no máximo três petroleiros em dois dias é igual a:

Alternativas
Comentários
  • Josiane

    O ponto dos concursos tem essa prova resolvida. Não consegui copiae e colar pois o pdf está protegido. Se vc quiser me mande um email que eu mando de volta com o atachado.

    OK?

    abs, Gino

    donatogino@gmail.com
  • Resolvi essa questão usando a seguinte fórmula que achei no wikipédia (http://pt.wikipedia.org/wiki/Distribui%C3%A7%C3%A3o_de_Poisson):



    λ = 2; t= 2

    Como e^(-λt)=e^(-4) é uma constante, vou chama-la de Q - fica mais fácil de digitar. Lembrando que 0!=1. Também temos que λt =4

    Como ele quer até 3 petroleiros, temos P(0) + P(1) + P(2) + P(3) = [(Q*4^0)/ 0!] + [(Q*4^1)/ 1!] + [(Q*4^2)/ 2!] + [(Q*4^3)/ 3!] = Q + 4Q + 8Q + (64Q/6) 
    =142Q/6 = 71Q/3 = [71e^(-4)] / 3

    Espero ter ajudado

    Marcelo
  • Distribuição de Poisson:
    Usa-se para calcular o número de sucessos ocorridos em um intervalo de tempo ou espaço.
    Geralmente a questão fornece: a média (M’) de ocorrência de sucessos em um certo intervalo(não necessariamente o intervalo pedido na questão); o intervalo (N), que deve ser considerado para o cálculo, e o número de sucessos (S) procurados.

    Dados da questão:
    M’=2 petroleiros por dia
    N= 2 dias
    Neste caso a média de ocorrência dada é de 2 petroleiros por dia, porém a questão pede que seja calculada a probabilidade para 2 dias e não 1, portanto, multiplicaremos a média de ocorrência dada (que é diária) por 2 dias, na equação usaremos a média como 4 petroleiros a cada 2 dias.
    M = 2xM’= 2(2)=4 petroleiros a cada 2 dias

    A questão pede a probabilidade de a refinaria receber ATÉ 3 petroleiros, ou seja, ela quer a probabilidade de receber 0 petroleiros + a probabilidade de receber 1 petroleiro + a probabilidade receber 2 petroleiros + a probabilidade de receber 3 petroleiros. Calcularemos a probabilidade para os sucessos 0, 1, 2 e 3 petroleiros e somaremos estes valores.
    S= 0, 1, 2 e 3 

    EQUAÇÃO:

    P(S)=[(M^S)(e^-M)]/S! 
                             

    OBS:(M^S= M
    elevado aS)
            (e^-M = e
    elevado a –M)
    P(S)é a probabilidade de ocorrer o sucesso S
    S é o número de sucessos que se deseja
    M é a média de sucessos para o intervalo pedido pela questão
    e=2,71828...

    CÁLCULOS:
    PARA S=0
    P(0)=
    [(4^0)(e^-4)]/0! = e^-4

    PARA S=1
    P(1)=
    [4(e^-4)]/1!= 4e^-4

    PARA S=2
    P(2
    )=[(4^2)(e^-4)]/2!=8e^-4

    PARA S=3
    P(3)
    =[(4^3)(e^-4)]/3!= [32(e^-4)]/3

    P TOTAL= e^-4
    + 4e^-4 + 8e^-4 + [32(e^-4)]/3=  [71( e^-4)]/3

  • Com base nos cálculos dos colegas acima torna-se verdadeira a alternativa:

    Gabarito: "C"

  • Aplicando a fórmula de Poisson, temos:

    Resposta: C

  • https://sabermatematica.com.br/provaauditorrfb.html

  • questao dessa e pular e partir pra proxima

  • questão do capeta, que Deus me abençoe na hora da prova, amém!


ID
67555
Banca
ESAF
Órgão
Receita Federal
Ano
2009
Provas
Disciplina
Raciocínio Lógico
Assuntos

Em um experimento binomial com três provas, a probabilidade de ocorrerem dois sucessos é doze vezes a probabilidade de ocorrerem três sucessos. Desse modo, as probabilidades de sucesso e fracasso são, em percentuais, respectivamente, iguais a:

Alternativas
Comentários
  • As possibilidades são {(sss),(ssf),(sfs),(fss),(fff),(ffs),(fsf),(sff)}, pelo enunciado o fracasso ocorre mais vezes(podemos perceber pela multiplicidade da ocorrência de dois sucessos), normalmente ocorre 3 vezes 2 sucessos e 1 vez 3 sucessos, mas a questão relata que se ocorrer 1 vez 3 sucessos, então ocorreram 12 vezes 2 sucessos, ficando assim as possibilidades:{(sss),(ssf),(ssf),(ssf),(ssf),(sfs),(sfs),(sfs),(sfs),(fss),(fss),(fss),(fss),....}, ou seja, sucesso na primeira e segunda posições fracasso na terceira posição ocorre quatro vezes ao invés de uma só,sucesso na primeira e terceira posições e fracasso na segunda ocorre também quatro vezes ao invés de uma,e assim por diante, logo fracasso ocorre 4 vezes mais do que sucesso, e aí teremos sucesso 1/5 = 20% e fracasso 4/5 = 80%
  • Não entendi muito bem essa explicação não. Existe alguma outra maneira de solucionar essa questão?
  • Chamarei o evento "sucesso" de S e o evento "fracasso" de F. A probabilidade de S, chamarei de "p". A probabilidade de F, chamarei de "1-p", já que o somatório das duas probabilidades deve ser 100% ou simplesmente 1. Usarei a seguinte notação para representar potências de 2 e 3: "p^2" para p ao quadrado e "p^3" para p ao cubo.Para ocorrer 2 sucessos, temos as seguintes possibilidades:1) SSF => p . p . (1-p) => p^2 - p^32) SFS => p . (1-p) . p => p^2 - p^33) FSS => (1-p) . p . p => p^2 - p^3Logo, a probabilidade de ocorrerem 2 sucessos e 1 fracasso é 3 vezes (p^2 - p^3).Para ocorrer 3 sucessos, temos apenas 1 possibilidade:1) SSS => p . p . p => p^3O enunciado diz que a probabilidade de ocorrerem dois sucessos é doze vezes a probabilidade de ocorrerem três sucessos. Então temos que:3(p^2 - p^3) = 12p^3, basta resolver a equação:3p^2 - 3p^3 = 12p^3, Dividindo os lados por 3p^2;1 - p = 4p1 = 5pp = 1/5 ou 0,20 ou 20%.Probabilidade de sucesso (S) = p = 20%Probabilidade de fracasso (F) = 1 - p = 80%Logo, temos que a resposta é a letra d.
  • Questao linda, prontinha e de facil entendimento!

    http://www.pontodosconcursos.com.br/admin/imagens/upload/4663_D.pdf

  • Vamos chamar de acertos (A) e erros (E). Temos que a probabilidade de 2 A é doze vezes a probabilidade de 3 A:

    Probabilidade de 3 A = x

    Probabilidade de 2 A = 12x

    Este é um problema binomial (afinal podemos acertar ou errar). Para tal tipo de probabilidade, uso a mesma regra do anagrama com letras repetidas, multiplicado pelas probabilidades. Então, temos "três provas", onde temos:

    Caso 1)  2 Acertos em 3 = AAE (pode ser escrito de 3 maneiras)

    Caso 2)  3 Acertos em 3 = AAA (uma maneira)

    Não temos ainda a informação das probabilidades de acerto e erro. Então vamos aplicar as letras em seus lugares:

    Caso 1) 3.(A.A.E) = 12x

    Caso 2) 1.(A.A.A) = x

    Chegamos então ao seguinte sistema de equações:

    3.(A2.E) = 12x

    A = x

    Simplificando a primeira, podemos dividir ambos os lados por 3:

    A2.E= 4x

    A = x

    Agora, vamos substituir o x:

    A2.E= 4A3

    (cortamos o A2)  ---> E = 4A

    Conclusão: sabemos que ao somar as probabilidades de Acertos e Erros, temos naturalmente 100% (1 inteiro).

    Ora, se A+E = 1  e E = 4A  então:

    A = 1/5 (20%)

    E = 4/5 (80%)


  • A melhor resposta foi, ao meu ver, a do Alexandre. Entretanto a formatação ficou confusa.
    Segue arrumada:

    1º) "sucesso"= S, "fracasso"=F; 

    2º) A probabilidade de S ="p", a de F="1-p" (perceba que S+F tem que ser 100%, testando: p+1-p = 1[ok!]);

    3º) Para ocorrer 2 sucessos, temos as seguintes possibilidades:

    a) SSF => p . p . (1-p) => p^2 - p^3

    b) SFS => p . (1-p) . p => p^2 - p^3

    c) FSS => (1-p) . p . p => p^2 - p^3

    .'. A probabilidade de ocorrerem 2 sucessos e 1 fracasso é 3 vezes (p^2 - p^3) 

    => 3(p^2-p^3).

    d) Para ocorrer 3 sucessos, temos apenas 1 possibilidade:1) SSS => p . p . p => p^3

    4º) O enunciado diz que a probabilidade de ocorrerem dois sucessos é doze vezes a probabilidade de ocorrerem três sucessos

    Então temos que:
    3(p^2 - p^3) = 12p^3, basta resolver a equação:
    3p^2 - 3p^3 = 12p^3, Dividindo os lados por 3p^2, temos:

    1 - p = 4p => 5p = 1=> p= 1/5 ou 0,20 ou 20%.

    Probabilidade de sucesso (S) = p = 20%
    Probabilidade de fracasso (F) = 1 - p = 80%

    Logo, temos que a resposta é a letra d.

  • Letra D

     

    Considerando:
    Sucesso: X
    Fracasso: 1 – X
    Número de provas: n

     

    Distribuição binomial:
    Cn,x . X^x . (1 – X)^1–x

     

    Como a questão informou que a probabilidade de ocorrerem 2 sucessos (e 1 fracasso) é 12x a probabilidade de ocorrerem 3 sucessos (e 0 fracasso), teremos:

     

    12 (C3,3 . X^3 . (1 – X)^0) = C3,2 . X^2 . (1 – X)^1
    12 (1 . X^3 . 1) = 3 . X^2 . (1 – X)

    12X^3 = 3X^2 . (1 – X)

    4X = (1 – X)

    5X = 1

    X = 0,2

     

    Sendo fracasso 1 – X, fracasso = 0,8.

     

    Bons estudos!

  • Seja x a probabilidade do sucesso, a probabilidade do fracasso será então (1-x), certo? Há 3 maneiras de ocorrer o sucesso: Seja S o sucesso e F o fracasso: 
    SSF 
    SFS 
    FSS 
    E uma única maneira de ocorrer os 3 sucessos: SSS 
    Temos então a equação: 
    3x²(1-x) = 12x³  As 3 maneiras de ocorrer 2 sucessos e 1 fracasso é = a 12 vezes a probabilidade de ocorrer 3 sucessos

    dividindo tudo por x², fica: 
    3(1-x) = 12x, dividindo agora tudo por 3: 
    1-x = 4x 
    5x = 1 
    x = 1/5 = 0,2 = 20% 
    x (a probabilidade do sucesso)= 1/5 = 0,2 (=20%). Logo, a probabilidade do fracasso é 1 - 0,2 = 0,8 = 80%. Espero ter ajudado. 

    __________________________________________________________________________________

    https://br.answers.yahoo.com/question/index?qid=20100223070946AAxjv4h + adaptações em negrito

  •          Veja que o número de tentativas é n = 3. Foi dito que a probabilidade de k = 2 sucessos é doze vezes a probabilidade de k = 3 sucessos.

    Logo, a probabilidade de sucesso é p = 20%. A probabilidade de fracasso é 1 – p = 80%.

    Resposta: D

  • Fórmula: P(x) = Cn,s x p^s x q^ n-s , onde S é o meu sucesso

    Dados da questão:

    n= 3

    p

    q = 1-p

    Eq. dada pela questão:P(2) = 12 x P(3)

    Resolução

    P(2), onde meu sucesso é = 2

    P(2) = C3,2 x p² x (1-p)¹

    P(2) = 3 x p² x (1-p)

    P(2) = 3p² x (1-p)

    P(3), onde meu sucesso(s) é = 3

    P(3) = C3,3 x p³ x (1- q) ⁰

    P(3) = 1x p³ x 1

    P(3)= p³

    agora é só substituir na equação: P(2) = 12 x P(3)

    3p² x (1-p) =12 x

    3p ² x (1 - p) = 12p² x p

    1- p = 12p² x p / 3p²

    Obs: Basta usar a decomposição de potência (propriedade da potenciação)

    12p³ = 12p² x p

    simplifica 12p² por 3p² = 4

    1- p = 4 x p

    4p= 1 - p

    4p + p = 1

    5p = 1

    P = ⅕

    P = 0,2

    P = 0,2 x 100%

    P = 20%

    q = 1 - p

    q = 1 - 0,2

    q = 0,8

    q = 0,8 x 100%

    q = 80%

  • 1) P(x=2) = 12 . P(x=3)

    N = 3

    2) Vamos calcular agora P (x=2)

    Lembrando que: K = 2 / N = 3 / P = sucesso / Q = fracasso = (1 - p)

    PS. Usei a fórmula padrão de probabilidade da distribuição binomial => (P=x) = (n! / k! n-k!) . p^k . (1-p)^n-k

    P(x=2) = (3! / 2! . 3-2!) . p^2 . (1-p)^1 (esse 1 é resultado do n - k da fórmula)

    P(x=2) = 3 . p^2 . (1-p)^1

    3) Vamos calcular agora P (x=3)

    Lembrando que: K = 3 / N = 3 / P = sucesso / Q = fracasso = (1 - p)

    P(x=3) = (3! / 3! . 3-3!) . p^3 . (1-p)^0 (esse 0 é resultado do n - k da fórmula)

    P(x=3) = 1 . p^3. 1

    4) Vamos substituir os dois valores encontrados para P(x=2) e P(x=3) na primeira fórmula

    P(x=2) = 12 . P(x=3)

    3 . p^2 . (1-p)^1 = 12 . 1 . p^3. 1

    3 . p^2 . (1-p) = 12 . p^3

    Isolando o p: 1 - p = 12p^3 / 3p^2

    Depois de resolver a fração do final: 1 - p = 4p

    1 = 4p + p

    1 = 5p

    p = 1/5

    p = 0,2

    Se p é 0,2 (20%), então q é 0,8 (80%) pois q = (1-p)

    Lembrando: p é sucesso e q é fracasso

  • Essa tem que ter braço

  • ESAF não brincava mesmo

  • S: sucesso; F: fracasso

    12*S*S*S = F*S*S + S*F*S + S*S*F

    a probabilidade para F*S*S = S*F*S = S*S*F, logo F*S*S + S*F*S + S*S*F = 3*F*S*S

    12*S*S*S = 3*F*S*S

    4*S*S*S = F*S*S

    4*S = F

    S + F = 1 (definição de evento binomial, só há duas possibilidades S ou F, logo S+F=100%)

    S + 4S = 1

    5S = 1

    S = 1/5 = 20%, assim F = 4/5 = 80%


ID
72052
Banca
CESGRANRIO
Órgão
IBGE
Ano
2010
Provas
Disciplina
Raciocínio Lógico
Assuntos

Três dados comuns e honestos serão lançados. A probabilidade de que o número 6 seja obtido mais de uma vez é

Alternativas
Comentários
  • Alguém conhece outro jeito de resolver essa?!
  • Pede-se a probabilidade que número 6 seja obtido mais de uma vez, portanto:1.Pode sair duas vezes5/6 x 1/6 x 1/6 (sendo 1/6 a probabilidade de sair o "6" e 5/6 a probabilidade de sair um número qualquer tirando o "6") x Permutação de 3 com repetição de 2 (isto pq o número "6" pode sair em qualquer ordem e o 6 se repete duas vezes) TEMOS: 5/6 x 1/6 x 1/6 x 3 = 15/2162. Pode sair as três vezes. 1/6 x 1/6 x 1/6 = 1/216Somando as duas probabilidades temos: 15/216 + 1/216 = 16/216Não se se ficou claro, mas é a forma como faço este tipo de questão.
  • Pode ser aplicada a fórmula da LEI BINOMIAL: ...............................k..........n-kP(E)= ___n!___ . ( P ) . (1-P) ............k!.(n-k)!onde,n= 3, quantidade de vezes que os dados são lançados;k= 2 e 3, número 6 obtido mais de uma vez. (pode acontecer ao jogar dois dados ou três dados)p= probabilidade de ocorrer o evento (1/6)para k=2; P¹(E)= ____3!____ .(1/6)².(1-1/6)³-² = 15/216..............2!.(3-2)! para k=3;P²(E)= ____3!___ .(1/6)³.(1-1/6)³-³ = 1/216.............3!.(3-3)!P(E)= P¹(E)+ P²(E)= 15/216 + 1/216 = 16/216 (letra d)
  • P (número 6 nenhuma vez) = 5/6 . 5/6 . 5/6 = 125/216 (restam 5 dos 6 números que poderão aparecer!)
     
    P (número 6 uma vez) = 3 . (1/5 . 5/6 . 5/6) = 75/216 (pode acontecer o número 6 em 3 oportunidades: na 1ª, na 2ª, ou na 3ª jogada. Por isso, precisamos multiplicar o resultado dos dados por 3)
     
    Então:
    P (número 6 mais de uma vez) = 1 – [P (número 6 nenhuma vez) + P (número 6 uma vez)]
    P = 1 – [125/216 + 75/216]
    P = 1 – 200/216
    P = 216/216 – 200/216
    P = 16/216
     
    Resposta correta: letra D.

    fonte:
    http://beijonopapaienamamae.blogspot.com.br/2010/04/dia-16-de-abril-questao-106.html
  • Três dados comuns e honestos serão lançados. A probabilidade de que o número 6 seja obtido mais de uma vez é: A probabilidade de que seja obtido 2 vezes mais a probabilidade de que seja obtido 3 vezes. Usando a distribuição binomial de probabilidade:

    f(k;n,p)={n\choose k}p^k(1-p)^{n-k}\,

    Acha-se a probabilidade de que seja obtido 2 vezes:
     

    f(2;3,\frac{1}{6})={3\choose 2}\times\left(\frac{1}{6}\right)^2\times\left(1-\frac{1}{6}\right)^{3-2}\,
    =\frac{3!}{2!\cdot\left(3-2\right)!}\,\!\times\frac{1}{36}\times(\frac{5}{6})^{1}\,
     
    =\frac{3\times2!}{2!\cdot\left(1\right)!}\,\!\times\frac{1}{36}\times\frac{5}{6}\,
     
    =\frac{3}{1}\times\frac{1}{36}\times\frac{5}{6}=\frac{15}{216}=\frac{5}{72}\,
     

    Agora a probabilidade de que seja obtido 3 vezes:


    f(3;3,\frac{1}{6})={3\choose 3}\times\frac{1}{6}^3\times(1-\frac{1}{6})^{3-3}\,

    =\frac{3!}{3!\cdot\left(3-3\right)!}\,\!\times\frac{1}{216}\times(\frac{5}{6})^{0}\,

    =\frac{3!}{3!}\times\frac{1}{216}\times1\,

    =1\times\frac{1}{216}\times1=\frac{1}{216}\,


    Assim, a resposta é:



    =\frac{15}{216}+\frac{1}{216}=\frac{16}{216}\,
     
  • Vamos tentar pensar mais objetivo com essa questão.

    Existem 4 possibilidades de a premissa do enunciado acontecer:

    Tirar 6 nos 3 dados;   ------>     1/6 * 1/6 * 1/6 = 1/216

    Tirar 6 no 1º e 3º dado; ------->  1/6 * 5/6 * 1/6 = 5/216

    Tirar 6 no 1º e 2º dado; ---------> 1/6 * 1/6 * 5/6 = 5/216

    e

    Tirar 6 no 2º e 3º dado; ---------> 5/6 * 1/6 * 1/6 = 5/216


    Soma-se as alternativas e obtem-se o seguinte resultado: 16/216

    Letra D
  • Só vim entender mesmo com Daniel Aleixo
  • Probabilidades: 

    P ("ser 6") = 1/6

    P ("não ser 6) = 5/6


    Como ele quer a probabilidade do nº 6 ser encontrado mais de uma vez, a ocorrência deste fato nenhuma vez ou uma vez não nos interessa. Sendo assim, o conjunto de probabilidades fica assim:


    Ocorrendo duas vezes:

    S ; S ; N = 1/6 * 1/6 * 5/6 = 5/216 

    S ; N ; S = 1/6 * 5/6 * 1/6 = 5/216

    N ; S ; S = 5/6 * 1/6 * 1/6 = 5/216


    Ocorrendo 3 vezes:

    S ; S ; S = 1/6 * 1/6 * 1/6 = 1/216

    S ; S ; S = 1/6 * 1/6 * 1/6 = 1/216

    S ; S ; S = 1/6 * 1/6 * 1/6 = 1/216


    Como são 3 possibilidades de ocorrência de 2 nº 6, e 3 possibilidades de ocorrência do nº 6, multiplica suas probabilidades por 3, logo:

    5/216 * 3 = 15/216

    1/216 * 3 = 1/216


    Como se trata de um evento união (ou um, ou outro), P( 2 U 3 ) = 15/216 + 1/216 = 16/216!

  • A probabilidade de que o nº 6 seja obtido mais de uma vez é de 


    S = 6

    N = demais números


    1º possibilidade:


    S S N = 1/6 * 1/6 * 5/6 = 5/216 

    ou

    S N S = 1/6 * 5/6 * 1/6 = 5/216

    ou

    N S S = 5/6 * 1/6 * 1/6 = 5/216

    MMC = 15/216


    2º possibilidade


    S S S = 1/6 * 1/6 * 1/6 = 1/216

    15/16 + 1/16 = 16/216

  • Mas Rafael, como surge esse 216/216? 

  • _____+______+______==> 06

       1º           2º         3º

    Na 1ª opção temos (1, 2, 3, 4) opções

    Na 2ª opção temos (1, 2, 3, 4) opções

    Na 3ª opção temos Sobrou apenas uma possibilidade

    4*4*1==> 16

    6*6*6==>216

    16/216

     

     

     

     

  • são 216 possiveis resultados, as chances de todos os 3 dados terem o número 6 seria de 36/216, então como as chances de dois de 3 dados terem o número 6 é menor do que de os 3 terem o número 6?

  • Possibilidades:

    a)dados 1 e 2 saírem 6 

    b)dados 1 e 3 saírem 6

    c)dados 2 e 3 saírem 6

    d)dados 1,2 e 3 saírem 6

    Calculando:

    probabilidade de ocorrer a: prob (dado 1 sair 6) =1/6 *prob (dado 2 sair 6)= 1/6 *prob (dado 3 não sair 6) = 5/6

    probabilidade a = 1/6*1/6*5/6 = 1/216

    -------------------------------------------------------------

    Probabilidade de ocorrer b e c é o mesmo raciocínio. Logo,

    probabilidade b = 1/216 probabilidade c= 1/216

    ----------------------------------------------------------------------

    Probabilidade de ocorrer d: prob (dado 1 sair 6) =1/6 *prob (dado 2 sair 6)= 1/6 *prob (dado 3 sair 6) = 1/6

    probabilidade d = 1/6*1/6*1/6 = 1/216

    -----------------------------------------------------------------

    Probabilidade do evento = prob a + prob b + prob c + prob d = 16/216

     

  • Uma forma pratica de pensar:

    PRIMEIRO; quantos lances dos dados possível 6 . 6 . 6 = 216


    SEGUNDO eventos

    1 ; 6 ; 6 primeiro evento

    6 ; 1 ; 6 segundo evento 3

    6 ; 6 ; 1 terceiro evento

    +

    2 ; 6 ; 6 primeiro evento

    6 ; 2 ; 6 segundo evento 3

    6 ; 6 ; 2 terceiro evento

    +

    3 ; 6 ; 6 primeiro evento

    6 ; 3 ; 6 segundo evento 3

    6 ; 6 ; 3 terceiro evento

    +

    4 ; 6 ; 6 primeiro evento

    6 ; 4 ; 6 segundo evento 3

    6 ; 6 ; 4 terceiro evento

    +

    5 ; 6 ; 6 primeiro evento

    6 ; 5 ; 6 segundo evento 3

    6 ; 6 ; 5 terceiro evento

    +

    6 ; 6 ; 6; último evento 1


    (3.5) + 1 = 16 assim 16/216

  • d-

    1°- total possibilidades: 6*6*6 = 216

    ________________________________________________________________________________________________________________________________________________

    2°- 6 mais 2x. o 6 pode cair 2 ou 3 vezes

    ________________________________________________________________________________________________________________________________________________

    3°- o 6 caindo 2x:

    1/6 * 1/6 * 5/6= 5/216.

    (POR QUE 5/6? PORQUE SO ESTAMOS CONSIDERANDO SE O 6 CAIR 2 VEZES. SE AS 2 PRIMEIRAS SERAM 6, É NECESSARIO Q A TERCEIRA SEJA QUALQUER N° MENOS 6)

    ________________________________________________________________________________________________________________________________________________

    4°- a combinacao 1/6 * 1/6 * 5/6 pode ser qualquer 1 das 3 ordens. logo:

    5/216*3= 15/216

    ________________________________________________________________________________________________________________________________________________

    5°. agora considerar quando cair 6 3x.

    1/6* 3 = 1/216.

    ________________________________________________________________________________________________________________________________________________

    6° somam-se as possibilidades: 15/216 + 1/216 = 16/216. a questao nao exige simplificacao de fracoes. entao R: 16/216

  • Pessoal vou fazer de um jeito bem explicado, porque os comentário aqui tá uma confusão total, vamos por passos.

    -> Temos 3 dados que a princípio serão lançados 1 de cada vez;

    -> Temos de analisar os dados saindo "6" nos 3 lançamentos (mais de 1 vez);

    -> Temos de analisar os dados saindo "6" em 2 lançamentos somente (mais de 1 vez);

    1) Aqui vamos considerar que irão sair "6" em todos os lançamentos

    # 1º Lançamentos temos: 1/6 (1 chance em 6);

    # 2º Lançamentos temos: 1/6 (1 chance em 6);

    # 3º Lançamentos temos: 1/6 (1 chance em 6);

    # Cálculo: 1/6 * 1/6 * 1/6 = 1/216

    2) Aqui vamos considerar que irão sair "6" em somente 2 lançamentos;

    Obs: Considerando que não vai sair o "6" no 1º lançamento

    # 1º Lançamentos temos: 5/6

    -> Corresponde a probabilidade de não sair 6, ou seja 5/6 (5 chances em 6), pois não queremos que saia o "6";

    # 2º Lançamentos temos: 1/6 (1 chance em 6, pois queremos que saia o "6");

    # 3º Lançamentos temos: 1/6 (1 chance em 6, pois queremos que saia o "6");

    # Cálculo: 5/6 * 1/6 * 1/6 = 5/216

    3) Aqui vamos considerar que irão sair "6" em somente 2 lançamentos;

    Obs: Considerando que não vai sair o "6" no 2º lançamento

    # 1º Lançamentos temos: 1/6 (1 chance em 6, pois queremos que saia o "6");

    # 2º Lançamentos temos: 5/6

    -> Corresponde a probabilidade de não sair 6, ou seja 5/6 (5 chances em 6), pois não queremos que saia o "6";

    # 3º Lançamentos temos: 1/6 (1 chance em 6, pois queremos que saia o "6");

    # Cálculo: 5/6 * 1/6 * 1/6 = 5/216

    4) Aqui vamos considerar que irão sair "6" em somente 2 lançamentos;

    Obs: Considerando que não vai sair o "6" no 3º lançamento

    # 1º Lançamentos temos: 1/6 (1 chance em 6, pois queremos que saia o "6");

    # 2º Lançamentos temos: 1/6 (1 chance em 6, pois queremos que saia o "6");

    # 2º Lançamentos temos: 5/6

    -> Corresponde a probabilidade de não sair 6, ou seja 5/6 (5 chances em 6), pois não queremos que saia o "6"

    # Cálculo: 5/6 * 1/6 * 1/6 = 5/216

    5) Cálculo geral das probabilidades:

    # TOTAL: 1/216 + 5/216 + 5/216 + 5/216 = 16 / 216

    Obs: Se os senhores ainda não entenderem manda mensagem aqui pra mim mesmo.

  • Estranho!

    São três dados. A sequência é irrelevante, a resposta deveria ser 6/216

    Agora, se fosse apenas 1 dado, jogado por três vezes, aí sim seria 15/216

    Vida que segue...


ID
84400
Banca
ACEP
Órgão
BNB
Ano
2006
Provas
Disciplina
Raciocínio Lógico
Assuntos

Um globo contém 9 bolas numeradas com algarismos distintos de 1 a 9. Sorteia-se, ao acaso, com reposição, três bolas do globo. Qual a probabilidade de que o resultado do sorteio seja a seqüência 3, 3, 3?

Alternativas
Comentários
  • O DETALHE DESTA QUESTÃO, QUE PARECE SER BEM SIMPLES, É QUE ELE AFIRMA QUE HÁ REPOSIÇÃO DAS BOLAS. BOM, SE SÃO 9 BOLAS E RETIRAMOS 1 AO ACASO, TEMOS UMA PROBABILIDADE DE 1/9 DE QUE A BOLA SEJA "3". LOGO, REPONDO-A E EXECUTANDO POR 3 VEZES ESSA OPRAÇÃO, TEREMOS 1/9 X 1/9 X 1/9 = 1/729. mAS QUE FIQUE CLARO QUE ISSOM SÓ FOI ADEQUADO VISTO QUE HOU REPOSIÇÃO DAS BOLINHAS. ESPERO TER AJUDADO.

ID
84403
Banca
ACEP
Órgão
BNB
Ano
2006
Provas
Disciplina
Raciocínio Lógico
Assuntos

Seja N o número de anagramas da palavra "AEIOUBCDF", cuja última letra à direita seja uma consoante. Denotemos por P a probabilidade de escolher-se aleatoriamente um dentre estes anagramas que contenha exatamente duas vogais juntas. Os valores de N e P são, respectivamente:

Alternativas
Comentários
  • 05 vogais04 consoantes 09 posições, sendo a última uma CONSOANTE fixa (que pode ser qualquer uma das 4 letras)8 x 7 x 5 x 4 x 3 x 2 x 1 (B)8 x 7 x 5 x 4 x 3 x 2 x 1 (C)8 x 7 x 5 x 4 x 3 x 2 x 1 (D)8 x 7 x 5 x 4 x 3 x 2 x 1 (E)........4 x 8! = 161.280 V C V C V C (V V) C V C V C (V V) C V C V C (V V) C V C V C (V V) C V C V C V C (estas são as únicas 4 formas de termos EXCLUSIVAMENTE DUAS VOGAIS JUNTAS. No entanto, a última letra à direita poderá ser qualquer uma das 4 consoantes, teremos que multiplicar por 4) 5 x 3 x 4 x 2 x 3 x 1 x 2 x 1 (B)5 x 3 x 4 x 2 x 3 x 1 x 2 x 1 (C)5 x 3 x 4 x 2 x 3 x 1 x 2 x 1 (D)5 x 3 x 4 x 2 x 3 x 1 x 2 x 1 (E) ..720 x 4(cada consoante) = 2.880 x 4(cada posição das vogais jutas) = 11.520 formas aparecerão EXATAMENTE DUAS VOGAIS JUNTAS.AGORA É SÓ CALCULAR A PROBABILIDADE:PROB = aquilo que eu quero / o que pode acontecerPROB = 11.520 / 161.280PROB = 1 / 14 (simplificando)
  • alguem poderia resolver melhor esta questao? pois pra quem ta começando ta dificil esse raciocínio...
  • Questão mal formulada.

    N = 4 x 8! , como o colega informou acima.
    mas P = 1 , pois todas as possibilidades contêm duas vogais juntas. O colega acima disse que as vogais tem de estar exclusivamente juntas, mas a questão diz exatamente.
  • Discordo da posição do colega acima, embora continue achando a o comentário anterior complexo.
    Se todoas as alternativas fossem válidas contaríamos também três vogais juntas e esta não á uma opção válida para a segunda proposição: exatamente duas vogais juntas. Faço minhas a pergunda:alguém pode resolver esta questão de uma forma mais fácil de entender? 
  • V V V V V
    5 4 3 2 1   = 120

    C C C C
    1 2   3  4   = 24

    120*24 = 2880

    Considerando o primeiro comentário onde temos que a sequencia de V C V C V C VV C pode ser organizada em mais 4 maneiras temos 4 * 2880 = 11520
  • Bom pessoal, vou tentar deixar mais clara a resolução.

    A primeira parte é um pouco mais simples. São 9 letras, sendo 5 vogais e 4 consoantes. Como à da direita deve ser consoante, primeiro devemos escrever esta quantidade de possibilidades para a letra da direita:

    ___ . ___ . ___ . ___ . ___ . ___ . ___ . ___ . 4

    Assim, para as outras 8 letras fazemos a distribuição:

    8 . 7 . 6 . 5 . 4 . 3 . 2 . 1 . 4 = 8! . 4

    Assim, são 4 . 8! anagramas com a letra da direita sendo consoante.

     

    Agora, a questão pergunta a probabilidade de escolher um dentre estes anagramas que contenha duas vogais juntas.

    Pensemos assim: Suponha que as duas primeiras letras sejam vogais:

    V  V  ___ ___ ___ ___ ___ ___ ___

    Perceba que a partir de então, todas as letras devem estar alternadas entre vogais e consoantes, caso contrário, apareceriam duas ou mais vogais juntas novamente.

    V V C V C V C V C

     

    Agora, suponha que a 2ª e 3ª letras sejam vogais. As outras letras devem novamente estar alternadas entre vogais e consoantes:

    C V V C V C V C V

    Mas, note que esta possibilidade não é permitida, pois a letra da direita é vogal e não consoante!

     

    "Caminhando" com a dupla de vogais e alternando as outras letras entre vogais e consoantes, e tomando o cuidado de escolher apenas as que terminam com consoante, encontramos 4 sequências:

    (V V) C V C V C V C

    V C (V V) C V C V C

    V C V C (V V) C V C

    V C V C V C (V V) C 

    Assim, para cada sequência podemos permutar as vogais ENTRE SI, e as consoantes ENTRE SI:

    Permutação das vogais entre si: 5!

    Permutação das consoantes entre si: 4!

     

    Assim, o total de possibilidades com EXATAMENTE DUAS VOGAIS JUNTAS e a ÚLTIMA LETRA SENDO CONSOANTE é:

    4 . 5! . 4!

     

    Utilizando a primeira parte da questão, calculamos a probabilidade:

    P = (Casos favoráveis) / (Casos possíveis)

    P = (4 . 5! . 4!) / (4 . 8!) => Simplificando por 4 e estendendo os fatoriais 4! e 8! até 5!, temos:

    P = (5! . 4 . 3 . 2 . 1) / (8 . 7 . 6 . 5!) => Simplificando por 5!, temos:

    P = (4 . 3 . 2 . 1) / (8 . 7 . 6) => Como 3 . 2 = 6, simplificando por 6, temos:

    P = (4 . 1) / (8 . 7) => Agora, simplificando por 4, temos:

    P = (1) / (2 . 7) =>

    P = 1/14

    Alternativa C

     

     

  • Ver explicação:

    http://beijonopapaienamamae.blogspot.com.br/2010/03/dia-15-de-marco-questao-74.html


ID
89122
Banca
ESAF
Órgão
MTE
Ano
2010
Provas
Disciplina
Raciocínio Lógico
Assuntos

Considere os dados da questão anterior.

____________________________________________________________________________________________________________________________
Dados da questão de 6.

Em uma amostra aleatória simples de 100 pessoas de uma população, 15 das 40 mulheres da amostra são fumantes e 15 dos 60 homens da amostra também são
fumantes. Desejando-se testar a hipótese nula de que nesta população ser fumante ou não independe da pessoa ser homem ou mulher, qual o valor mais próximo
da estatística do correspondente teste de qui-quadrado?
____________________________________________________________________________________________________________________________

Ao se escolher ao acaso cinco pessoas da amostra, sem reposição, a probabilidade de exatamente quatro delas serem homens fumantes é dada por:

Alternativas
Comentários
  • O conteúdo da alternativa b nada mais é que:[C(15,4) x C(85,1)] / C(100,5)
  • http://www.pontodosconcursos.com.br/admin/imagens/upload/4995_D.pdf

    resolucao. Questao 7
  • - Comentário do prof. Arthur Lima (ESTRATÉGIA CONCURSOS)

    1) Temos 15 homens fumantes no grupo de 100 pessoas. Para escolher 4 homens fumantes, basta calcular a combinação de 15, 4 a 4: C(15,4).

    2) Para que a outra pessoa não seja um homem fumante, temos 85 possibilidades (40 mulheres, fumantes ou não, e mais os 45 homens não fumantes).

    3) Assim, temos 85 x C(15,4) possibilidades de escolher 5 pessoas, sendo exatamente 4 homens fumantes. A quantidade de formas de se escolher 5 pessoas em um grupo de 100 é dado pela C(100,5).

    4) Portanto, a probabilidade de escolher 5 pessoas, contendo exatamente 4 homens fumantes, é:
    P = favoráveis/ total = 85 x C(15,4)/ C(100,5)

    5) Veja que na letra B temos Cm,k CN-m,n-k /CN,n, sendo N=100, n=5, m=15 e k=4. Substituindo as letras N, n, m e k pelos valores dados nessa alternativa, temos: C15,4 C100-15, 5-4 / C100, 5 = C15,4 C85, 1 / C100, 5 = C15,4 85 / C100, 5




    Gabarito: Letra B

  • Temos 15 homens fumantes no grupo de 100 pessoas. Para escolher 4 homens fumantes, basta calcular a combinação de 15, 4 a 4: C(15,4).

           Para que a outra pessoa não seja um homem fumante, temos 85 possibilidades (40 mulheres, fumantes ou não, e mais os 45 homens não fumantes).

           Assim, temos 85 x C(15,4) possibilidades de escolher 5 pessoas, sendo exatamente 4 homens não fumantes.

           A quantidade de formas de se escolher 5 pessoas em um grupo de 100 é dado pela C(100,5).

           Portanto, a probabilidade de escolher 5 pessoas, contendo exatamente 4 homens não fumantes, é:

           Veja que na letra B temos C C /C, sendo N=100, n=5, m=15 e k=4. Substituindo as letras N, n, m e k pelos valores dados nessa alternativa, temos:

    C C / C = C C / C = C 85 / C

           Portanto, esta é a resposta.

    Resposta: B


ID
89374
Banca
FUNRIO
Órgão
PRF
Ano
2009
Provas
Disciplina
Raciocínio Lógico
Assuntos

Os acidentes automobilísticos ocorridos em duas autoestradas (E1 e E2) são classificados, pela idade do motorista que provoca o acidente, em três faixas etárias distintas (A, B e C). As quantidades de acidentes nas faixas etárias A, B e C seguem, nessa ordem, uma progressão aritmética decrescente para a estrada E1, e uma progressão geométrica de razão 0,5 para a estrada E2. Sabendo-se que 51% de todos os acidentes ocorrem na estrada E1, a probabilidade de um motorista pertencente à faixa etária B provocar um acidente é de

Alternativas
Comentários
  • E1 = 51% = (X1-R) + X1 + (X1+R) Sendo R a proporção aritmética .E1 = 51% = 3 X1 então X1 = 17% (Correspondente a faixa etária da estrada E1)E2 = 100% - 51% = 49%considerando a razão 0,5E2 = 49% = 2X + X + X/2 então X e2 = 14%Percentual faixa etária B = 17% + 14% =31% 31%/100% = 0,31
  • Comentário: Considere os dados seguintes:

    E1

    Faixa etária B: x

    Faixa etária A: x – r

    Faixa etária C: x + r

    Total: x + x – r + x + r = 3x

    E2

    Faixa etária A: y

    Faixa etária B: 2y

    Faixa etária C: 4y

    Total: y + 2y + 4y= 7y

    Como 3x =51% e 7y =49%, então temos que x = 0,17 e y = 0,07.

    Em relação à categoria B, temos:

    E1 = x = 0,17

    E2 = 2y = 2 . 0,07 = 0,14

    Solução: 0,17 + 0,14 = 0,31

    GABARITO: LETRA D


ID
106858
Banca
CESGRANRIO
Órgão
Petrobras
Ano
2008
Provas
Disciplina
Raciocínio Lógico
Assuntos

Pedro está jogando com seu irmão e vai lançar dois dados perfeitos. Qual a probabilidade de que Pedro obtenha pelo menos 9 pontos ao lançar esses dois dados?

Alternativas
Comentários
  • 9 Pontos: (6,3);(3,6);(5,4);(4,5)10 Pontos: (6,4);(4,6);(5,5);11 Pontos: (6,5);(5,6);12 Pontos: (6,6);Soma-se as possibilidades dos pontos e dividi-se pela totalidade possível:10 / 36 (simplifica por 2)R= 5 / 18
  • São 6 X 6 = 36 resultados possíveis ao se jogar 2 dados.Probabilidade de, no mínimo, 9 pontos. As possibilidades são:9 pontos: 4 resultados possíveis => 6+3 ou 5+4 ou 3+6 ou 4+510 pontos: 3 resultados possíveis => 6+4 5+5 ou 4+611 pontos: 2 resultados possíveis => 6+5 ou 5+612 pontos: 1 resultado possível => 6+6Total de resultados possíveis com, no mínimo, 9 pontos: 10.Prob (mínimo 9 pontos) = 10/36 = 5/18 (Alternativa D)
  • Dois dados perfeitos = cada dado tem 6 lados, e são dois dados, então 6 x 6 = 36

    Ao lançar os dados, o máximo que podemos ter é 12 (considerando que saia 6 de um dado e 6 de outro).

    Pelo menos 9 pontos = possibilidade da soma dos dois dados ser 9, 10, 11 ou 12.

    Probabilidade da soma ser 9: (3,6 - 4,5 - 5,4 - 6,3) = 4/36
    Probabildiade da soma ser 10: (4,6 - 5,5 -6,4) = 3/36  

    Probabildiade da soma ser 11: (5,6 - 6,5) = 2/36
    Probabildiade da soma ser 12: (6,6) = 1/36

    4/36 + 3/36 + 2/36 + 1/36 = 10/36

    10/36 = 5/18              

    Fonte: 
    http://www.youtube.com/watch?v=g3FItNqbyf4
  • Temos 10 combinações (resultados favoráveis) para que Pedro obtenha pelo menos 9 pts ao lançar os dois dados ao mesmo tempo, são eles:

    (3,6); (6,3); (4,5); (5,4); (4,6); (6,4); (5,5); (6,5); (5,6) e (6,6).

    Como são dois dados perfeitos, ou seja possuem 6 lados, teremos um total de 6 x 6 = 36 resultados possíveis.



    Letra D.




  • Possui uma controversia nesse caso,

    Se o exercicio pergunta quais as probabilidades de se alcancar 9 pontos entao vejamos:

    probabilidades sao 4

    (3,6) (4,5) (5,4) ( 6,3)= 4opcoes

    2 dados: 6x6=36

    logo: 4]36= 1]9

    Altenativa A

    Mas se for seguido pelos numeros totais do dado soma-se as opcoes:

    4]9+3]36+2]36+1]36= 10]36 = 5]18 alternativa D.

  • Essas questões que tem "pelo menos" costuma exigir muito do raciocínio.

  • 10cp/36cf

    5/18

  • Resposta: alternativa D.

    Comentário do prof. Felipe Cardoso no YouTube (Professor em Casa) 2:44s

    https://youtu.be/UJMyTw7iOV4


ID
107338
Banca
CESPE / CEBRASPE
Órgão
TRT - 16ª REGIÃO (MA)
Ano
2005
Provas
Disciplina
Raciocínio Lógico
Assuntos

A diretoria da associação dos servidores de uma pequena empresa deve ser formada por 5 empregados escolhidos entre os 10 de nível médio e os 15 de nível superior. A respeito dessa restrição, julgue os itens seguintes.

Se a diretoria fosse escolhida ao acaso, a probabilidade de serem escolhidos 3 empregados de nível superior seria maior que a probabilidade de serem escolhidos 2 empregados de nível médio.

Alternativas
Comentários
  • Na verdade a probabilidade seria igual.

    2/10=0,20 e 3/15=0,20

     

  • Não entendi, pois o espaço amostral não é de 25? Sendo assim tenho que calcular a probabilidade de 15/25 x 14/25 x 13/25 = 0,17 que é probabilidade de ser escolher ao acaso 3 empregados de nível superior. No segundo caso 10/25 x 9/25 = 0,14 que é a probabilidade de se escolher ao acaso dois empregados de nivel médio.

    Então não sei por que a resposta do gabarito pe ERRADA.

    Por favor quem dominar o assunto, me ajude e aponte onde está o erro da minha dedução.
  • Fernando, 
    não importa a ordem em que são escolhidos.
    então, num grupo de 5, entre Nível Médio (NM) e Nível Superior (NS), eu posso ter:
    a) 0 NM e 5 NS; ou
    b) 1 NM e 4 NS; ou
    c) 2 NM e 3 NS; ou
    d) 3 NM e 2 NS; ou
    e) 4 NM e 1 NS, ou
    f) 5 NM e 0 NS.

    O problema não quer saber exatamente a probabilidade de acontecer o caso "c", mas a probabilidade de, ao escolher 3 Nível superior, comparar com a probabilidade de ter 2 Nível médio, portanto, a probabilidade é a mesma!! e de quanto seria isso?

    Simples, TOTAL: faz combinação de 25 em 5 => 25! / 20!5!

    depois, vê em cada caso,
    NM: combinação de 10 em 2 => 10! / 8!2!
    NS: combinação de 15 em 3 => 15! / 12!3!
    Agora é só multiplicar uma pela outra (que é o espaço amostral) e dividir pelo total de combinações possíveis calculado anteriormente, só não vou fazer pois dá um trabalhinho e não foi exatamente o que pediu a questão hehehhehe

    espero ter ajudado
    abraços

  • Assertiva ERRADA pois é igual:

    Probabilidade de escolher UMA pessoa de nível superior = 15 / 25 ou (3 / 5)
    Probabilidade de escolher UMA pessoa de nível médio = 10 / 25 ou (2 / 5)

    Assim, como são 3 de superior e 2 de médio ficaria:

    (Superior)     3 / 5 x 2 / 4 x 1 / 3 =  1 / 10
    (médio)     2 / 5 x 1 / 4 = 1 / 10

    A probabilidade é igual para os dois casos.
  • Acredito que seja assim o cálculo.

    Se a diretoria é formada por 5 membros, temos então:


    * Probabilidade de 3 empregados c/ ensino superior


    S S S M M ( 15/25 * 14/24 * 13/23 * 10/22 * 9/21)


    * Probabilidade de 2 empregados c/ ensino médio


    M M S S S (10/25 * 9/24 * 15/23 * 14/22 * 13/21) 


    Dará o mesmo resultado.


  • Nem precisa fazer conta. São a mesma coisa..kkkk

  • 3/15= 1/5 e 2/10= 1/5 nem maior e nem menor, seria igual.

  • Primeiro: calcular o espaco amostral de cada situacao

    Nivel Superior -> Total 15 pessoas, formar comissoes com 3: C(15,3) = 455 comissoes com 3 de ensino superior;

    Nivel Medio -> Total 10 pessoas, formar comissoes com 2: C(10,2) = 45 duplas de ensino medio possiveis de serem formadas.

    Assim, o espaco amostral final sera: E=45*455=20475 (que sao IGUAIS) ->20475 possibilidades para escolher SSSMM em qualquer ordem.

    Segundo: calcular o UNIVERSO possivel de comissoes com 5 entre as 25 pessoas

    C(25,5) = 53130

    E, terceiro, responder a pergunta: pegar um trio de ensino superior eh a mesma chance de pegar uma dupla de ensino medio, tendo o Universo  U de comissoes?

    P(S)=20475/53130 = 0,3854 =38,54%

     

    Na hora da prova nem precisaria fazer calculo: precisaria ter em mente que pegando 3 de ensino superior IMPLICA AUTOMATICAMENTE em pegar 2 do ensimo medio, ou seja, mesma probabilidade (pois so tem 2 grupos para se escolher).

  • Galera o jeito que nosso colega Frederico Guimarães comentou é o correto de se fazer, a outra forma é mais rápida mas nem sempre vai dá certo de fazer. Depende muito de interpreção da questão, no caso a forma que fizeram mais resumida deu certo porque quando se quer 5 na equipe logo as duas exigência daram certo, porque tendo 3 de um e 2 de outro dará 5 da equipe sempre.

  • Achei a questão dúbia.

    Como não cravaram que o evento é "serem escolhidos (exatamente) 3 empregados de nível superior" e "serem escolhidos (exatamente) 2 empregados de nível médio", concluí que o cenário era com pelo menos essa quantidade, que satisfaz o enunciado também, mas não bateu com o gabarito.

    Enfim, CESPE sendo CESPE.


ID
114367
Banca
ESAF
Órgão
SUSEP
Ano
2010
Provas
Disciplina
Raciocínio Lógico
Assuntos

Um estudo indica que, nas comunidades que vivem em clima muito frio e com uma dieta de baixa ingestão de gordura animal, a probabilidade de os casais terem filhos do sexo masculino é igual a 1/4. Desse modo, a probabilidade de um casal ter dois meninos e três meninas é igual a:

Alternativas
Comentários
  • Parece que essa questão não tem resposta.Vejam a resolução do Sérgio (ponto dos concursos):"Trataremos aqui da Probabilidade Binomial, haja vista que as características do evento em tela se enquandram perfeitamente neste modelo!Reparem que se a probabilidade de nascer um menino é de 1/4, deduz-se que a probabilidade de nascer uma menina será de 3/4. Confere?Vou chamar P(H) e P(M), para Homem e Mulher. Ok?Teremos: P(H)=1/4 e P(M)=3/4.Daí, aplicando a probabilidade binomial para o nascimento de 2 Homens (meninos) em 5 nascimentos, teremos:P(2H) = (Combinação de 5, 2 ) x (P(H))^2 x (P(M))^3Onde houver "^", leia-se "elevado a". Ok?Fazendo as continhas, teremos que: P(2H) = 135/512Este resultado não está contemplado entre as alternativas de resposta! A questão é nula.Idêntico resultado seria encontrado, caso tivéssemos usado a fórmula para calcular a probabilidade do nascimento de 3 meninas, ou seja, para calcular P(3M): contas iguais e mesma resposta: 135/512.De um jeito ou de outro, a questão é anulável!":|
  • O meu deu 27/1024

    (1/4)^2 x (3/4)^3 = 27/1024

  • Permutação com repetição, pois ele não disse em que ordem as crias seriam geradas: P5(2,3) ou seja, permutação de 5 elementos com repetição de 2H e 3M, que dá um resultado de 10.

    Daí é só fazer as multiplicações das probabilidades individuais e multiplicar por 10, resultado 135/512.

    Questão sem resposta.

  • Usando a fórmula da distribuição binomial temos:

    Chamemos de “sucesso” ter um filho do sexo masculino: probabilidade igual a 1/4.

    Chamemos de “fracasso” ter um filho do sexo feminino: probabilidade igual a 3/4.

    P (sucesso) = 1/4

    Q (Fracasso) = 3/4

    N = 5

    K = número de sucessos (neste caso é 2)

    Fórmula: N/K * P* Q

    = 5/2 * 1/43/4

    = 5/2 * 1/16 * 27/64

    = 135 / 2.048

    Agora teremos que encontrar o número de repetições dos eventos, que é de em 5 elementos se repetem 2 homens e 3 mulheres.

    5! / 3! 2! = 10.

    135 *10 / 2048 = 1.350 / 2.048 Simplificando 135/512.


ID
114370
Banca
ESAF
Órgão
SUSEP
Ano
2010
Provas
Disciplina
Raciocínio Lógico
Assuntos

Uma urna contém bolas vermelhas, azuis, amarelas e pretas. O número de bolas pretas é duas vezes o número de bolas azuis, o número de bolas amarelas é cinco vezes o número de bolas vermelhas, e o número de bolas azuis é duas vezes o número de bolas amarelas. Se as bolas diferem apenas na cor, ao se retirar ao acaso três bolas da urna, com reposição, qual a probabilidade de exatamente duas bolas serem pretas?

Alternativas
Comentários
  • 1º) Devemos estipular um valor para a cor que não é citada (Vermelha)Vermelhas = 1AzuisAmarelasPretas2º) O número de bolas pretas é duas vezes o número de bolas azuis, o número de bolas amarelas é cinco vezes o número de bolas vermelhas, e o número de bolas azuis é duas vezes o número de bolas amarelas.Vermelhas = 1Azuis = 10Amarelas = 5Pretas = 20Total = 36 bolas3º) Se vamos retirar ao acaso três bolas da urna, com reposição, qual a probabilidade de exatamente duas bolas serem pretas? Preta x Preta x "qualquer outra cor" x Permutação de 3 com 2 elementos repetidos20/36 x 20/36 x 16/36 x 3x2/2x1 (tentem simplificar ao máximo!)5/9 x 5/9 x 4/9 x 3100/243:)
  • Um video muito bem fundamentado do "eu vou passar" sobre a questao

    http://www.youtube.com/watch?v=jie2pLzgJ-o
  • Vermelha (v) = v

    Azul (a) = 2y = 2x5v = 10v

    Amarelo (y) = 5v

    Preto (p) = 2a = 2x10v = 20v

    Estabeleçamos então valor para "v":

    v = 1

    Então:

    v = 1

    a = 10

    y = 5

    p = 30

    total: 36

    Bolas pretas (p): 20/36 => 5/9

    Bolas não pretas (ñp): 16/36 => 4/9

    Possibilidades de serem exatamente duas pretas:

    p x p x ñp = 5/9 x 5/9 x 4/9 = 100/729

    p x ñp x p = 5/9 x 4/9 x 5/9 = 100/729

    ñp x p x p =  4/9 x 5/9 x 5/9 = 100 /729

    --------------------------------------------------------

    3 x 100/729 = 300/729 

    Simplificando por 3 = 100/243

  • Ótima explicação!

  • Chamando de P, AZ, AM e V o número de bolas Pretas, Azuis, Amarelas e Verdes, temos:

    P = 2AZ

    AM = 5V

    AZ = 2AM

           Podemos escrever tudo em função de V. Veja:

    AZ = 2AM = 2x(5V) = 10V

    P = 2AZ = 2x(10V) = 20V

           Portanto, o total de bolas é:

    Total = P + AZ + AM + V = 20V + 10V + 5V + V = 36V

           Temos 36V bolas, das quais 20V são pretas. A chance de retirar uma bola preta é de 20V/36V = 20/36 = 5/9. Como o exercício diz que devemos repor a bola (“com reposição”), a chance de tirar uma segunda bola preta é também 5/9. E a chance da terceira bola não ser preta é de 16V/36V = 16/36 = 2/9.

           Assim, a probabilidade da primeira E da segunda bolas serem pretas E da terceira bolas não ser preta é:

           Veja que este é o caso onde temos Preta – Preta – Não Preta. Devemos ainda permutar esses 3 resultados, com a repetição de 2:

           Portanto, a probabilidade de ter 2 bolas pretas e uma não preta, em qualquer ordem, é:

    Resposta: B


ID
129496
Banca
CESPE / CEBRASPE
Órgão
CEHAP-PB
Ano
2009
Provas
Disciplina
Raciocínio Lógico
Assuntos

A probabilidade de serem encontrados defeitos em uma casa popular construída em certo local é igual a 0,1. Retirando-se amostra aleatória de 5 casas desse local, a probabilidade de que em exatamente duas dessas casas sejam encontrados defeitos na construção é

Alternativas
Comentários
  • Probabilidades:Casa com defeito = 1/10Casa sem defeito = 9/10"Exatamente 2 dessas 5 casas sejam encontrados defeitos na construção":2 casas com defeito e 3 casas sem defeito1/10 * 1/10 * 9/10 * 9/10 * 9/10 * Permutação de 5 com 2 e 3 elementos repetidos (Deve-se multiplicar pela permutação pois pode ser a 1ª e a 2ª casas com defeito, ou a 3ª e a 5ª, ou a 1ª e a 4ª, etc)729/100000 * 5*4*3!/3!*20,00729 * 10= 0,0729 , isto é, "inferior a 0,15."Letra "a".;)
  • Cara, vocês só explicam a parte matemática.. Não explicam por que chegaram a esta conta.... aff..
  • Fiz da seguinte forma:

    São 5 casas, exatamente 2 com defeito (A), e 3 sem defeito (B)

    probabilidade de defeito: 0,1
    probabilidade sem defeito: 1- 0,1 = 0,9

    Logo, temos -> Probabilidade: 0,1 x 0,1 x 0,9 x 0,9 x 0,9

    No entanto, as casas podem ser escolhidas de várias formas
    Ex: AABBB ou ABABB

    Repare que teremos exatamente essa configuração de 5 casas escolhidas, porém, elas podem se permutar.
    então, eu resolvi tal qual um anagrama, onde A repete 2 vezes e B repete 3 vezes...

    Logo: 0,1 x 0,1 x 0,9 x 0,9 x 0,9 x 5! = 0,0729
                           3!2! 

    Resposta letra A
  • Ok! Legal seu comentário mas em se tratando de multiplicação de resusltados parciais seria dispensável o calculo por anagrama. No final vai dar o mesmo resultado.
  • como eu me enrolei um pouco para entender, procurei algo detalhado e assim matei minha curiosidade em alguns detalhes. segue  explicação que achei e espero ajudar.

    "Será que é uma questão de Probabilidade Binomial? Vamos ver!
    1) Temos eventos excludentes? Sim PH, ou a casa tem algum defeito (P = 0,1) ou está tudo ‘ok’ com ela (P = 1 – 0,1 = 0,9).
    2) A questão pede a probabilidade de uma determinada quantidade de eventos? Sim, são 2 dentre 5 casas devem ter defeitos.
     
    Então, precisamos fazer assim:
    Probabilidade = 10 . 0,01 . 0,729 = 0,0729 (inferior a 0,15) "    "
    fonte: professor ph - http://beijonopapaienamamae.blogspot.com.br/2010/07/dia-16-de-julho-questao-197.html
  • É caso de distribuição binomial, cujas probabilidades de cada ensaio de Bernoulli são:

    sucesso: casa com defeito => p = 0,1

    fracasso: casa sem defeito => q = 1 - p = 0,9

    A probabilidade de ter 2 sucessos (consequentemente 3 fracassos) em 5 ensaios é C(5;2)*(0,1)^2*(0,9)^3 = 0,079.

    Resposta: a.

    Opus Pi.

  • Obrigada IZABELA.

    Só entendi a questão com seu comentário. Explicação clara.

    Bons estudos.

  • Dos inúmeros modos de se resolver essa questão, mostro para vocês o modo da Probabilidade Binominal.

    Características da PROBABILIDADE BINOMINAL:
    *Sempre haverá o EVENTO MENCIONADO(sucesso) e o EVENTO ANTAGÔNICO(fracasso); (Eventos mutuamente excludentes)
    Exemplos de eventos mutuamente excludentes entre si: Nascimento de MENINO ou MENINA, Retirada de bola de cor VERMELHA ou AZUL de dentro de uma urna, ACERTOS ou ERROS, entre outros.
    *Haverá um evento se repetindo determinado número de vezes;
    *Para o evento haverá apenas dois resultados possíveis - chamados de sucesso ou fracasso.
    O sucesso ou o fracasso explica o nome do tipo da probabilidade. Binominal, dois resultados apenas.
    Relacionamos SUCESSO com aquilo que o examinador quer que ocorra no problema, por exemplo: "Qual a probabilidade de exatamente duas das sete bolas retiradas sejam AZUIS em uma urna com bolas AZUIS e VERMELHAS". Já o FRACASSO será sempre o antagônico, ou seja, bolas VERMELHAS em uma urna de bolas AZUIS e VERMELHAS.

    Essa questão é um clássico exemplo de Probabilidade Binominal. 

    Resolver...

    1º PASSO: Coletar o número de vezes que o evento irá ocorrer.
    O enunciado diz que "Retirando-se amostra aleatória de 5 casas desse..."
    Percebe-se facilmente que o EVENTO se repetirá CINCO VEZES

    2º PASSO: Coletar a Probabilidade do Evento Mencionado e quantas vezes o examinador quer que esse evento ocorra.
    O enunciado diz: "A probabilidade de serem encontrados defeitos em uma casa popular construída em certo local é igual a 0,1"
    Logo, a Probabilidade do Evento Mencionado ocorrer é de 10% ou 0,1.
    O enunciado diz também: "...a probabilidade de que em exatamente duas dessas..."
    Logo, o examinador quer que EXATAMENTE duas das casas escolhidas estejam com falha na construção.

    3º PASSO: Coletar a Probabilidade do Evento Antagônico e quantas vezes o examinador quer que esse evento ocorra.
    Esse passo é quase sempre por inferência.
    Como a probabilidade de ter falha na construção é 10%, a Probabilidade do Evento Antagônico é dada por 100% - 10% que é igual a 90% ou 0,9.
    Também a quantidade de vezes de ocorrer o evento antagônico é tomada por inferência.
    Como a quantidade de vezes que o examinador quer quer ocorra o evento mencionado é igual a 2, então a quantidade de vezes que deve ocorrer o evento antagônico é dada por 5 - 2 que é igual a 3.

    4º PASSO: Aplicar a fórmula.
    FÓRMULA: P = Cn,s x P(Evento Mencionado)^s x P(Evento Antagônico)^f
    Onde:
    n é o número de repetições do evento;
    s é o numero de repetições do evento mencionado(sucesso);
    f é o número de repetições do evento antagônico(fracasso);

    Teremos,
    P = C5,2 x (0,1)² x (0,9)³
    P = 10 x 0,01 x 0,729
    P = 0,0729

  • GABARITO: A

    A questão envolve Distribuição Binominal.

    1º Passo: Maneiras distintas de como 2 casas com defeito na construção podem estar distribuídas dentre as 5 selecionadas: C5,2

    2º Passo: Probabilidade da casa ter defeito na construção: 0,1 ou 1/10

    Como preciso de 2 casas com defeito, essa probabilidade se repete 2 vezes: (1/10) x (1/10)

    3º Passo: Probabilidade da casa NÃO ter defeito na construção: 0,9 ou 9/10

    As outras 3 casas, necessariamente, NÃO podem ter defeito na construção, logo, essa probabilidade se repete 3 vezes: (9/10) x (9/10) x (9/10)

    Assim, temos que:

    C5,2 x (1/10) x (1/10) x (9/10) x (9/10) x (9/10) = 0,0729

  • LETRA A

  • essa dava pra matar na lógica: se a probabilidade de ter erro em uma casa é 0,1, logicamente a probabilidade de ter erro em duas casas será menor que 0,1. Quando fala que tem que ser em EXATAMENTE duas casas aí fica mais baixa ainda. // De certa forma: 1/10 . 1/10 . 9/10 . 9/10 . 9/10 tudo isso podendo permutar, então multiplica por 5!. porém deve haver a divisão pelos números que se repetem, ou seja, 2! (1/10) e 3! (9/10). Resultado: 0,0729 (bem menor do que 0,1). Bora pensa fora da caixa!!!

ID
131500
Banca
CESPE / CEBRASPE
Órgão
Caixa
Ano
2010
Provas
Disciplina
Raciocínio Lógico
Assuntos

Saul e Fred poderão ser contratados por uma empresa. A probabilidade de Fred não ser contratado é igual a 0,75; a probabilidade de Saul ser contratado é igual a 0,5; e a probabilidade de os dois serem contratados é igual a 0,2. Nesse caso, é correto afirmar que a probabilidade de

Alternativas
Comentários
  • O comentário do professor acima está otimo. Mas tem outra forma bem simples. Dá para resolver até de cabeça, sem nenhum cálculo.1) Interpretação dos eventos1.1) Como a questão falou que podem ocorrer eventos mútuos, então os eventos serão "dependentes" e/ou "não mutuamente exclusivos"1.2) Esta questão tem várias alternativas, a resolução de uma alternativa será de real importância para a resolução das demais. Isso é muito comum, ou seja, devemos resolver uma alternativa e somente depois poderemos resolver as demais. Agora o difícil, às vezes, é achar o ponto de partida. 2) Passo a passo para uma boa resolução:- Leia a questão e as alternativas- Retire os dados, utilizando variáveis que facilitam a resolução, mapeando sempre para 'a' e 'b'(isso facilita o raciocínio):Dados:a = 0,25 - equivale a frase "Fred ser contratado"b = 0,50 - equivale a frase "Saul ser contratado"a e b = 0,203) Visualização dos conjutos utilizando textos:3.1) Imagine o conjunto 'a' representado por "[ ] ' , ou seja, tente imaginar o conjunto sendo envolvido pelas chaves.3.1) Imagine o conjunto 'b' representado por "{ } ', ou seja, tente imaginar o conjunto sendo envolvido pelos colchetes.3.2)Imagine a intersecção entre os conjuntos por "{ ], ou seja, tente imaginar a intersecção entre os dois conjuntos.4) Visualizando a questão na forma de conjuntos:conjunto 'a' ----> [0,75]conjunto 'b' ----> {0,50}intersecção entre os conjuntos 'a' e 'b' -------> {0,20]5) Visualização completa(A e B):0,25[ {0,20] } 0,50de outra forma, bem mais especifica [0,05 {0,20] 0,30 } 6) Opções possíveis, respondento apenas com a visualização, sem efetuar nenhum cálculo:a e b = 0,20a ou b = 0,05+0,2+0,3=0,55a e não b = Só a = 0,25 - 0,20 = 0,05b e não a = Só b = 0,5 - 0,2 = 0,3Realmente é muito simples, então vejamos as alternativas:a) errado.pelo menos um dos dois = a ou b = 0,55b) errado.a = 0,25 - equivale a frase "Fred ser con
  • continuação anterior....Realmente é muito simples, então vejamos as alternativas:a) errado.pelo menos um dos dois = a ou b = 0,55b) errado.a = 0,25 - equivale a frase "Fred ser contratado"c) erradob e não a = Só b = 0,5 - 0,2 = 0,30d) errado.a e não b = Só a = 0,25 - 0,20 = 0,05Bons estudos.
  • Nao entendi, vejam o que fiz :

    Usei a "regra do e "   P(A e B)= P(A) . P(B)
    Como o exercicio diz ...Saulo ser contratado E Fred nao ser contratado....     =>   P(A)= Prob. de Saulo ser contratado 0,5       P(B)= Prob. de Fred não ser contratado 0,75,
    Logo 0,5 . 0.75 = 0,375

    Nao entendi pq o resultado dá errado.
    Se alguem puder me explicar, agradeço

  • Considere os eventos:S: Saul ser contratado.F: Fred ser contratado.Foi dito que p(~F) = 0,75 e p(S) = 0,5. Além disso, p(S e F) = 0,2.Analisemos cada alternativa.a) A probabilidade de pelo menos um dos dois ser contratado é p(S ou F) = p(S) + p(F) - p(S e F). Sabemos que p(F) = 1 - p(~F), logo p(F) = 1 - 0,75 = 0,25. Assim, p(S ou F) = 0,5 + 0,25 - 0,2 = 0,55, que é diferente de 0,75.Alternativa incorreta.b) Vimos na alternativa anterior que p(F) = 0,25, que é diferente de 0,5.Alternativa incorreta.c) A probabilidade de Saul ser contratado e Fred não ser contratado é p(S e ~F) = p(S ou F) - p(F) = 0,55 - 0,25 = 0,30.Alternativa correta.d) A probabilidade de Fred ser contratado e Saul não ser contratado é p(~S e F) = p(S ou F) - p(~S). Como p(~S) = 1 - p(S) = 1 - 0,5 = 0,5, segue-se que p(~S e F) = 0,55 - 0,5 = 0,05, que é diferente de 0,1.Alternativa incorreta.e) Vimos que p(~S) = 0,5, que é diferente de 0,25.Alternativa incorreta.Letra C.Opus Pi.Nota: esta questão pode ser resolvida de forma muito rápida usando os diagramas de conjunto (diagramas de Venn), mas ainda não é possível colocar figuras nos comentários. Por isso a resolução de forma algébrica.
  • Pessoal, percebi que é possível inserir figura. Aí vai a representação para o problema:Opus Pi.
  • A questão informa que a probabilidade de FRED não ser contratado é 0,75. Sendo assim a probabilidade de FRED ser contratado vai ser 0,25.

    A probabilidade de SAUL ser contratado é 0,50 conforme informado na questão. Então a probabilidade de SAUL não ser contratado é 0,50.

    A probabilidade de os dois serem contratados é 0,20. Então 0,20 é a interseção entre o SAUL e o FRED, é a probabilidade que está em um e no outro. Fazendo o diagrama de Venn fica fácil para visualizar. Não consegui incluir a imagem aqui.

    Se 0,20 é a probabilidade de os dois serem contratados, a probabilidade de apenas SAUL ser é 0,50 - 0,20 = 0,30.

    Opção correta: letra C

  • GOSTARIA QUE O GRUPO POSTASSE A RESOLUÇÃO DA QUESTÃO.. 


    ATENCIOSAMENTE, OBRIGADA.

  • Não entendi a o gabarito NEM o comentario do professor.

    No comentario do professor ele considerou o conectivo OU (p(s OU f)) isso implica dizer que pode acontecer o primeiro OU o segundo evento. A alternativa "C" pede a possibilidade de ocorrer os dois eventos JUNTOS (Saul ser contratado E Fred não),

    O correto seria usar o  conectivo E, pois os dois eventos tem que ocorrer (contratar Saul E não contratar fred), e neste caso a formula é P(A e B) = P(a) * P(b) que daria um resultado de 0,37 (que não existe...)

    A solução proposta pela Jaqueline também esta errada. Ela calculou corretamente a probabilidade de SAUL ser contrato, a alternativa C pede a probabilidade de Saul ser contratado E Fred não ser.

    Se alguém souber explicar ajudar ai...


  • Resolvi essa questão usando a teoria dos conjuntos, mas não se adicionar figuras aqui :(

    Seguindo o enunciado temos:

    Fred não ser contratado 0,75, então a probabilidade de Fred ser contratado é igual a 0,25.

    Saul ser contratado 0,5. 

    Ambos 0,2. Então temos:

    F=0,25

    S=0,5

    Ambos = 0,2

    Logicamente a probabilidade de APENAS UM ser contratado é igual ao valor total menos a interseção, logo teremos:

    F= 0,25 - 0,2 = 0,05

    S= 0,5 - 0,2= 0,3

    Traduzindo para a sentença poderíamos afirmar as seguintes conclusões. 

    I) a probabilidade de Fred ser contratado e Saul não é igual a 0,05.

    II) A probabilidade de Saul ser contratado e Fred não é igual a 0,3. (Exatamente o que está na alternativa C)


    Comentando as alternativas:

    A) pelo menos um dos dois ser contratado é igual a 0,75. (Somando a possibilidade de apenas um dos dois ser chamado temos 0,05 + 0,3 = 0,35)   B) Fred ser contratado é igual a 0,5. (Pelo cálculo temos que é 0,05 + 0,2 = 0,25)    C) Saul ser contratado e Fred não ser contratado é igual a 0,3. Correta.   d) Fred ser contratado e Saul não ser contratado é igual a 0,1. Essa probabilidade é igual a 0,05    E) Saul não ser contratado é igual a 0,25. Se a probabilidade de Saul ser contratado é igual a 0,3 e ambos é 0,2 então o total é 0,5, ou seja, probabilidade de ser contratado é igual a 0,50 e logicamente 0,50 de não ser contratado. Alterei este último comentário devido a observação da Michelle. Obrigado

  • Rafael Meireles, seu comentário foi bastante útil, obrigada!

    Só não entendi o comentário da última alternativa, que vc escreveu: e) Saul não ser contratado é igual a 0,25. Se a probabilidade de ser contratado é igual a 0,5 e ambos é 0,2 então o total é 0,7, ou seja, probabilidade de não ser contratado é igual a 0,30.

    A probabilidade de Saul ser contratado não seria 0,5 ao invés de 0,7 como vc comentou? Por que o 0,2 da contratação de Saul e Fred já está "imbutido"no 0,5 não é? Assim, a probabilidade de Saul não ser contratado seria 0,5 (1 - 0,5). Não altera a resposta, só o motivo pelo qual a letra e) está errada.

  • Aqui nessa questão, e como não sou bom, é um problema de português. Basta verificar que na cara, Saul não ser contratado é realmente igual a 0,30 pois quando colocado os 0,20 contra os 0,50 o problema nos mostra que Saul, melhor, S, é colocado em primeiro plano do argumento e está mais próximo da resposta correta, já que os dois serem contratados é parte composta (0,05 porque?).

  • Fred não ser: 0,75

    Saul ser contratado: 0,5

    Fred e Saul ser contratado: 0,2

     

    Quando a questão diz "...a probabilidade de Saul ser contratado é igual a 0,5", nessa probabilidade é incluido a probabilidade de Saul e Fred serem contratados juntos. Então se a questão também afirma que "...a probabilidade de os dois serem contratados é igual a 0,2...", deve-se fazer a subtração (0,5-0,2) obtendo-se a probabilidade de apenass Saul ser contratado. E se a questão afirma que a probabilidade de Fred não ser contratado é de 0,75, então a probabilidade de ele ser contratado é 0,25, incluindo-se,nesta, a probabilidade de os dois serem contratados juntos, então, se 0,25 é a de ser contratado, 0,05 será aprobabilidade de apenas Fred ser contratado.

    l----------------------------------------------------------------------------------------------------l

    l                     fred                                             saul                                         l 

    l            l----------------------l--------------l---------------------------------l                      l

    l             l         0,05          l        0,2   l      0,3                           l                      l

    l             l----------------------l-------------l---------------------------------l                      l

    l                                                                                   0,45                               l

    l-----------------------------------------------------------------------------------------------------l

     

    gabarito C

     

  • eu fiz assim: 75/100 x 50/100 = 0,375 ( letra c)

  • Estou com a mesma dúvida do Walter e do Lucas:


    "Saulo ser contratado E Fred nao ser contratado....    => 

     P(A)= Prob. de Saulo ser contratado 0,5      

    P(B)= Prob. de Fred não ser contratado 0,75,

    Logo 0,5 . 0.75 = 0,375


    Alguém para nos ajudar?

  • P ( Fred ñ ser contratado.) 0,75 --- 75

    P (Saul ser contratado.) = 0,5 -- 5

    P (Fred e Saul) = 0,2 --- 2

    ai você desenha os círculos de intercessão

    Como a probabilidade dos dois serem contratados 2% . Depois é só subtrair.

    75 - 2 = 73

    5 - 2 = 3

    gabarito letra C) espero ter ajudado.

  • LETRA C

  • Segue um vídeo que achei de um professor explicando essa questão

    https://www.youtube.com/watch?v=IIiwN65chKU

  • saul ser contratado e fred não ser = 0,3 (C) // pois a probabilidade de saul ser contratado é 0,5 - 0,2 que é a interseção (probabilidade de os dois serem contratados).

ID
136018
Banca
ESAF
Órgão
MPOG
Ano
2010
Provas
Disciplina
Raciocínio Lógico
Assuntos

Um viajante, a caminho de determinada cidade, deparou-se com uma bifurcação onde estão três meninos e não sabe que caminho tomar. Admita que estes três meninos, ao se lhes perguntar algo, um responde sempre falando a verdade, um sempre mente e o outro mente em 50% das vezes e consequentemente fala a verdade nas outras 50% das vezes. O viajante perguntou a um dos três meninos escolhido ao acaso qual era o caminho para a cidade e ele respondeu que era o da direita. Se ele fi zer a mesma pergunta a um outro menino escolhido ao acaso entre os dois restantes, qual a probabilidade de ele também responder que é o caminho da direita?

Alternativas
Comentários
  • Supondo que o primeiro fale somente verdade,o segundo somente mentira e o terceiro 50% verdade,50% mentira.Entao para cada situacao temos duas alternativas por causa do terceiro menino. Vamos montar uma tabela verdade:MENINO 1 MENINO 2 MENINO 3D.............E...........DD.............E...........EE.............D...........DE.............D...........ETOTAL DE PROBABILIDADES=12TOTAL DE AMOSTRAGENS=4 ====>4/12=1/3
  • Como 1 (mente), 1 (fala a verdade) e 1 (mente ou fala a verdade), temos as respectivas respostas:

    1- Esquerda
    2- Direita
    3- Esquerda ou direita

    O primeiro ja respondeu "esquerda", entao só nos resta a pessoa "2" e a pessoa "3". Totalizando 3 possibilidades (Direita, Esquerda ou Direita). Como a quetão pede a possibilidade da resposta esquerda, seria um de três, ou seja, 1/3.

    Espero ter ajudado.
  • Fiz diferente:

    Para termos como resposta duas "direitas", ou ambos os meninos falam a verdade, ou ambos mentem. Para isso, queremos: P(2 falarem a verdade) + P (2 mentirem).

    Sendo:
    menino 1 é o que sempre fala a verdade
    menino 2 é o que sempre mente
    menino 3 é o 50%/50%


    P (2 falarem a verdade):
    Hipotese 1: menino 1 é escolhido primeiro, depois menino 3 (e ele falou a verdade) = 1/3 * 0,5/2 = 0,5 / 6
    Hipotese 2: menino 3 é escolhido primeiro (e ele falou a verdade) e depois é escolhido o menino 1=  0,5/3 * 1/2 = 0,5/6

    Prob. Total de falarem a verdade = hipotese 1 + hipotese 2 = 0,5/6 + 0,5/6 = 1/6

    P (2 falarem mentirem):

    Hipotese 3: menino 2 é escolhido primeiro, depois menino 3 (e ele mentiu) = 1/3 * 0,5/2 = 0,5 / 6
    Hipotese 4: menino 3 é escolhido primeiro (e ele mentiu) e depois é escolhido o menino 2=  0,5/3 * 1/2 = 0,5/6

    Prob. Total de mentirem = hipotese 3 + hipotese 4 = 0,5/6 + 0,5/6 = 1/6

    Logo, a Prob. de todos mentirem ou de todos falarem a verdade é de 1/6 + 1/6 = 2/6 ou 1/3

    espero ter ajudado! bons estudos!!!


  • Hipótese 1: Escolher o menino que sempre fala a verdade. Neste caso a probabilidade de um dos outros 2 restantes falar direita é:
    1/2 (Probabilidade de escolher o menino que fala 0,5 da verdade) x 1/2 (Probabilidade deste menino falar direita)
    Obs.: Nesta hipótese o que mente sempre irá dizer esquerda
    Hipótese 2: Escolher o menino que sempre mente. Neste caso a probabilidade de um dos outros 2 restantes falar direita é:
    1/2 (Probabilidade de escolher o menino que fala 0,5 da verdade) x 1/2 (Probabilidade deste menino falar direita)
    Obs.: Nesta hipótese o que fala a verdade sempre irá dizer esquerda
    Hipótese 3: Escolher o menino que diz 0,5 da verdade. Neste caso se o menino estiver falando a verdade: então a probabilidade de um dos outros 2 dizer direita é: 1/2 (Probabilidade de escolher o menino que sempre fala a verdade)*1(100% de chance dele falar direita)
    Obs.: Nesta hipótese o que mente sempre irá dizer esquerda
    Se o menino está mentindo, a probabilidade de escolher outro que fale direita é: 1/2 (Probabilidade de escolher o que sempre mente) *1(100% Chance dele falar direita).
    Obs.: Nesta hipótese o que fala a verdade sempre irá dizer esquerda
    Ou seja na terceira hiótese tempos sempre 1/2

    Probabilidade final: Probabilidade de escolher um dos meninos * (p1+p2+p3)= 1/3 * (1/2*1/2 + 1/2*1/2 + 1/2) = 1/3 Letra D

    Fonte: http://www.pontodosconcursos.com.br/admin/imagens/upload/4959_D.pdf


  • Pessoal, é muito mais simples.

         Indiferentemente de o viajanter ter escolhido o garoto que fala a verdade ou mente ou verdade/mente, a questão pergunta qual a probabilidade de, ao escolher entre os 2 restante, a resposta ser mantida (direita). Inicialmente, há 4 opcoes, 2 direitas e 2 esquerdas. Após a escolha do 1o. garoto, haverá 3 opções, com apenas 1 possibilidade de se manter a escolha do 1o. garoto. Portanto, 1/3.
  • Questão muito difícil!

  • 1- fala a verdade

    1-mente

    1-mente 50% e fala a verdade 50%

    O viajante perguntou a um dos três meninos escolhido ao acaso qual era o caminho para a cidade e ele respondeu que era o da direita. 
    Nao sabemos qual o menino que fala.
    Se ele fi zer a mesma pergunta a um outro menino escolhido ao acaso entre os dois restantes, qual a probabilidade de ele também responder que é o caminho da direita?
    A probabilidade é 1/3 pois nao sabemos quem é quem no enunciado. 

  • SABENDO-SE QUE  A POSIÇÃO DOS MENINOS V E F SE EXCLUEM( OU SEJA, UM JAMAIS VAI FALAR DIREITA JUNTO COM O OUTRO) E QUE O MENINO DUPLO TEM 50% DE CHANCE DE FALAR DIREITA EM QUALQUER CASO:

    HIP 1:  1o)  FOI O  MENINO V   -->  ;  LOGO :  TENHO 50% DE CHANCE DE ACERTAR O MENINO DUPLO E 50% DE CHANCE DE O MENINO DUPLO FALAR -->;  1/2 x1/2= 1/4;

    HIP 2:  1o)  FOI O  MENINO F  -->    ;  LOGO :  TENHO 50% DE CHANCE DE ACERTAR O MENINO DUPLO E 50% DE CHANCE DE O MENINO DUPLO FALAR -->;  1/2 x1/2= 1/4;

    HIP 2:  1o)  FOI O  MENINO DUPLO  -->    ;  LOGO : QUAL SEJA A VERDADE OU MENTIRA, SEI QUE UM VAI APONTAR DIREITA, E OUTRO , ESQUERDA, LOGO TENHO 50% DE CHANCE;

    TIRANDO A MÉDIA:  (1/4 + 1/4 + 1/2) / 3  =  1/3

    GABARITO : D

     

  •         Veja que a chance de o primeiro menino ser o que fala sempre a verdade é 1/3. Da mesma forma, a chande de o primeiro menino ser o que sempre mente também é 1/3, bem como a chance dele ser o que mente ou fala a verdade é também 1/3. Observe ainda que, após ter sido escolhido o primeiro menino, cada um dos dois meninos restantes tem 1/2 de chance de ser o segundo escolhido.

                   A partir disso, vamos analisar todos os possíveis casos onde as duas crianças podem falar “direita” (que pode ser uma verdade ou uma mentira):

    - o primeiro menino ser o que sempre fala a verdade (probabilidade = 1/3) e o segundo ser o que mente ou fala a verdade (probabilidade = 1/2) e, neste caso, ele falar a verdade (probabilidade = 50% = 1/2):

    P = (1/3) x (1/2) x (1/2) = 1/12

    - o primeiro menino ser o que sempre mente (probabilidade = 1/3) e o segundo ser o que mente ou fala a verdade (probabilidade = 1/2) e, neste caso, ele mentir (probabilidade = 50% = 1/2):

    P = (1/3) x (1/2) x (1/2) = 1/12

    - o primeiro menino ser o que mente ou fala a verdade (probabilidade = 1/3), neste caso ele ter dito a verdade (probabilidade = 1/2), e o segundo menino ser o que sempre fala a verdade (probabilidade = 1/2):

    P = (1/3) x (1/2) x (1/2) = 1/12

    - o primeiro menino ser o que mente ou fala a verdade (probabilidade = 1/3), neste caso ele ter mentido (probabilidade = 1/2), e o segundo menino ser o que sempre mente (probabilidade = 1/2):

    P = (1/3) x (1/2) x (1/2) = 1/12

                   Ao todo, a probabilidade de o segundo menino falar “direita” é:

    P = 1/12 + 1/12 + 1/12 + 1/12 = 4/12 = 1/3

    Resposta: D


ID
136033
Banca
ESAF
Órgão
MPOG
Ano
2010
Provas
Disciplina
Raciocínio Lógico
Assuntos

Em uma pequena localidade, os amigos Arnor, Bruce, Carlão, Denílson e Eleonora são moradores de um bairro muito antigo que está comemorando 100 anos de existência. Dona Matilde, uma antiga moradora, fi cou encarregada de formar uma comissão que será a responsável pela decoração da festa. Para tanto, Dona Matilde selecionou, ao acaso, três pessoas entre os amigos Arnor, Bruce, Carlão, Denílson e Eleonora. Sabendo-se que Denílson não pertence à comissão formada, então a probabilidade de Carlão pertencer à comissão é, em termos percentuais, igual a:

Alternativas
Comentários
  • Quantidade de grupos possíveis: C4,3 = 4!/3!.1! = 4

    Quantidade de grupos sem Carlão: 1

    Quantidade de grupos com Carlão: 3

    Probabilidade de Carlão pertencer à comissão: 3/4 = 0,75 = 75%

    Portanto, a resposta é letra "e".

    http://raciociniologico.50webs.com/APO2010/APO2010.html#Questão 07

  • Da pra fazer assim tb:

    1 - tudo o que não pode (Carlão ficar de fora)


    Tudo que não pode:
    3/4 . 2/3 . 1/2 = 6/24 = 1/4


    1 - 1/4 = 0,75 = 75%
  • Sabe-se de antemão que Denilson não participará, então podemos exclui-lo da resolução, a comissão poderá ser formada por  Anor, Bruce, Carlão e Eleonora.

    O “Espaço Amostral”, que é o conjunto de todas as comissões possíveis de serem formadas  com os 4 candidatos, será dado pela fórmula de Combinação de 4 elementos tomados em grupos de 3 a 3 (usaremos Combinação pois a ordem das pessoas nas comissões não importa para diferenciá-las).

    Combinação de N elementos tomados P a P= (N!)/[P!(N-P)!] =4!/[3!1!]=4
    Nosso ESPAÇO AMOSTRAL=4

    O “número de eventos favoráveis” é o conjunto de comissões formadas, dentre aquelas 4 pessoas, com a presença de Carlão obrigatoriamente em cada uma delas. Calcularemos por Combinação de 3 elementos (Anor, Bruce e Eleonora) tomados 2 a 2, haja vista que Carlão já ocupa uma posição na comissão, ou seja, só há disponibilidade de mais 2 elementos participarem de cada comissão.

    Combinação de N elementos tomados P a P= (N!)/[P!(N-P)!] =3!/[2!1!]=3
    Nosso NÚMERO DE EVENTOS FAVORÁVEIS = 3

    Tendo o total de comissões possíveis (ESPAÇO AMOSTRAL), formadas com aquelas 4 pessoas, e o total de comissões formadas com a presença obrigatória de Carlão (NÚMERO DE EVENTOS FAVORÁVEIS), podemos calcular a probabilidade de Carlão pertencer a comissão sorteada por Dona Matilde:
    P=3/4=0,75=75%  
  • Probabilidade = quero / tenho   5 amigos comissão de 3 pessoas. Como Denilson não está na comissão TENHO C 4,3 = 4   Só que QUERO que o Carlão participe = C 3,2 (ou seja os outros 3 amigos disputando 2 vagas) = 3   Probabilidade 3 / 4 = 75%

    bons estudos!
  • 3 possibilidades de Carlão ser selecionado na comisão DIVIDIDO pelo total dos amigos que é 4 EXCLUINDO Denílson .

    3/4 = 0,75 x 100 = 75%
  • De um jeito mais rápido.
    A, B, C e E.   D está fora.
    Sendo assim, cada um tem 25% em cada possibilidade.
    São 3 possibilidades.
    25 + 25 + 25 = 75%
  • Considernado
    A -> Arnos
    B -> Bruce
    C -> Carlão
    D -> Denilson
    E -> Eleonora

    Se D está fora então podemos constituir os seguintes grupos;
    (A, B, C)
    (A, C, E)
    (A, B, E)
    (B, C, E)

    Ou seja, dos 4 grupos formados Carlão aparece em 3, 3/4 = 0,75
  • Quanta conta pra uma questão que se faz de cabeça. Você tem 3 lugares, 4 pessoas. Só existe uma posição, de um total de 4, que faz o Carlão ficar fora: Ou ele tá fora, ou ele tá na primeira posição, ou tá na segunda posição, ou tá na terceira. Ou seja, ele tem 3 vagas para ocupar e participar da comissão, de um total de 4. 3/4 = 75%.

  • probabilidade = casos favoraveis / casos possíveis


    casos possíveis = C4,3 = 4!/3! = 4

    casos favoraveis = C3,1 = 3!/2! = 3


    probabilidade = 0,75 = 75%


    Ou visualmente:

    ABC

    ABE

    ACE

    BCE


    Gabarito: E


ID
136039
Banca
ESAF
Órgão
MPOG
Ano
2010
Provas
Disciplina
Raciocínio Lógico
Assuntos

Em uma urna existem 200 bolas misturadas, diferindo apenas na cor e na numeração. As bolas azuis estão numeradas de 1 a 50, as bolas amarelas estão numeradas de 51 a 150 e as bolas vermelhas estão numeradas de 151 a 200. Ao se retirar da urna três bolas escolhidas ao acaso, com reposição, qual a probabilidade de as três bolas serem da mesma cor e com os respectivos números pares?

Alternativas
Comentários
  • Total bolas__200____1° bola da cor______N°Par(x/2)______3 bolas(x^3) azuis______50_____50/200 =1/4__________1/8________1/8*1/8*1/8 = 1/512 amarelas__100_____100/200=1/2__________1/4________1/4*1/4*1/4 = 1/64vermelhas__50_____50/200 =1/4__________1/8________1/8*1/8*1/8 = 1/512 Probalidade total =1/512+1/64+1/512 = 1/512+8/512+1/512 = 10/512Gabarito (a)
  • Bolas Azuis:
    Total = 50
    Pares: 25

    Bolas Amarelas:
    Total = 100
    Pares: 50

    Bolas Vermelhas:
    Total = 50
    Pares: 25

    Probabilidade de três azuis e pares:
    P1 = (25/200).(25/200).(25/200)
    P1 = (1/8).(1/8).(1/8)
    P1 = 1/512

    Probabilidade de três amarelas e pares:
    P2 = (50/200).(50/200).(50/200)
    P2 = (2/8).(2/8).(2/8)
    P2 = 8/512

    Probabilidade de três vermelhas e pares:
    P3 = (25/200).(25/200).(25/200)
    P3 = (1/8).(1/8).(1/8)
    P3 = 1/512

    Probabilidade total:
    Pt = P1 + P2 + P3
    Pt = 1/512 + 8/512 + 1/512
    Pt = 10/512

    Portanto, resposta letra "a"

    fonte:http://raciociniologico.50webs.com/APO2010/APO2010.html#Quest%C3%A3o%2009

ID
136042
Banca
ESAF
Órgão
MPOG
Ano
2010
Provas
Disciplina
Raciocínio Lógico
Assuntos

As apostas na Mega-Sena consistem na escolha de 6 a 15 números distintos, de 1 a 60, marcados em volante próprio. No caso da escolha de 6 números tem-se a aposta mínima e no caso da escolha de 15 números tem-se a aposta máxima. Como ganha na Mega-sena quem acerta todos os seis números sorteados, o valor mais próximo da probabilidade de um apostador ganhar na Mega-sena ao fazer a aposta máxima é o inverso de:

Alternativas
Comentários
  • O número possível de resultados para a megasena, vamos chama-lo de M, é a combinação de 60 em grupos de 6, portanto:

    M = 60! / (54! * 6!)

    Logo, a probabilidade de ocorrer um determinado jogo é o inverso de M.

    Na aposta máxima, o jogador pode marcar 15 dezenas quaisquer. Então, a combinação de 15 em grupos de 6 é:

    m = 15! / (9! * 6!)

    A probabilidade solicitada pelo enunciado é o resultado da divisão de M por m, assim o seu inverso é:

    1 / P = M / m = 60! / (54! * 6!) / (15! / (9! * 6!)) = (60! * 9! * 6!) / (54! * 6! * 15!) ~= 10.000.

    Portanto, alternativa correta "e".

  • Resolução 

    Total de Jogos Possíveis = C60,6 = 60!/(6! x 54!) = 50.063.860 

    Total de Jogos Possíveis com 15 números: 

    C15,6 = 15!/(6! x 9!) = 5.005 

    Probabilidade de Acertar = 5.005/50.063.860 = 10.002,77 

    GABARITO: E 

  • Alguém tem alguma dica pra resolver isso aí sem calculadora e em menos de 3 min, que é a média normalmente para se resolver as questões das provas?
     

  • André,

    Também queimei as pestanas nesse exercício e cheguei à seguinte conclusão para facilita na resolução:
    * Vai simplificando o que der, no caso, peguei:
    - 60 / 10 - 6
    - 6 / 12 - 1/2
    - 55 / 11 - 5
    - 5 / 15 - 1 / 3
    - 58 / 14 (/2) - 29 / 7

    Quando terminou as simplificações exatas, comecei a simplificar por aproximação, visto o gabarito dar valores muito dispersos... daí comecei:
    - 7 / 3 - considerei como 2
    - 59 / 3 - considerei como 20

    cheguei ao resultado aproximado, e sem muito esforço.

    Espero ter ajudado.
  • A primeira bola do sorteio tem 15/60 de chances de ser um dos números do apostador. A segunda bola, tirada agora do universo de 59 restantes, tem 14/59 de chances de estar entre os números restantes (são 14 agora) do apostador, as outras bolas têm as seguintes probabilidades: 13/58, 12/57, 11/56 e 10/55. Então é só multiplicar tudo e ir simplificando numeradores com denominadores, o que é bem rápido.
    Chega-se, assim, à conta 1/4*1/59*13/29*1/19*1/7. Como os valores das alternativas estão bem afastados uns dos outros,59 pode virar 60, 29 pode virar 30 e 19 pode virar 20. Multiplicando os denominadores assim aproximados, chega-se a 132.240 e é só dividir (de cabeça, obviamente) por 13 para se chegar a algo um tanto maior que 10.000. Então a resposta é letra E, lembrando que na aproximação acima feita nós aumentamos um popuco os valores (que ficarão no numerador) ou seja, o resultado real seria menor que o encontrado dividindo 132.240 por 13, o que só encaixa na letra E.

  • A resolução em tempo hábil sem calculadora é muuuuuuito complicada.

    Para resolver esta o candidato deve ser um jogador assíduo que lê o verso do volante....rsrs
  • A resposta - o inverso do valor mais próximo da probabilidade de um apostador ganhar, nada mais é que resutados/combinações 

    combinações = quantas soluções são possíveis em uma aposta de 15 números 6 estarem corretos = C15,6 = 15!/6!/9!

    resultados = quantas combinações de 6 existem no conjunto de 60 números = C60,6 = 60!/6!/54!


    portanto resultados/combinações = (6!*9!*60!) / (15!*6!*54!) = 10.002

    letra E

  • Para uma questão múltipla escolha fica mais fácil resolver com algumas simplificações indicadas pelos colegas, difícil é pegar uma Cespe e acertar na lata esse tipo de questão, apesar que eu acho que essa banca não cobra uma questão que exija tanto dispêndio de cálculos no momento de resolver a questão.

    Força!

  • Gabarito: Letra E
     

    O numero de possibilidades é de C60,6 = 50.063.860

    O numero de apostas simples contidas em uma aposta máxima é de C15,6 = 5005

    A probabilidade de acertar a MEGASENA com uma aposta máxima é de: 

    5005/50.063.860 = 1 / 10002,769...

    Aproximadamente uma chance em dez mil.

  • Então, dá bem menos trabalho partir diretamente para o raciocínio: 15 números de 60, vezes, 14 números de 59, vezes,...

    Ou seja,  15/60 x 14/59 x 13/58 x 12/57 x 11/56 x 10/55

     

    Pois aí, é só simplificar ao máximo, multiplicar e por fim dividir. Se fizer por combinação, terá um trabalho a mais para chegar à mesma coisa.

    Veja:

    Se fizermos  C15,6 = 15! / 6! x (15 -6)! dividido por C60,6 = 60! / 6! x (60 - 6)!, sem efetuar totalmente o cálculo, apenas simplificando o possível:

     

    15x14x13x12x11x10x9! / 6! x 9!     dividido por 60x59x58x57x56x55x54! / 6! x 54!        Simplificar os números em azul

     

    Para efetuar a divisão, copia a primeira fração e inverte a segunda:

     

    15x14x13x12x11x10 / 6!       x       6! / 60x59x58x57x56x55         Simplificar os números em azul

     

    Resta:

    15/60 x 14/59 x 13/58 x 12/57 x 11/56 x 10/55

  • eu entendi a resolução, mas minha dúvida é: não teria que se considerar as possibilidade de se escolher os 15 números?

    na resolução proposta os 15 números já estão escolhidos e vc combina as possib; dos 6 vencedores estarem entre eles, mas não se considera todas as possibilidades que algéum tem de escolher 15 números... acho que teria que ficar

    60!/15!x45! (escolha dos 15 números possíveis) e depois multiplicar pela possibilidade do número vencedor estar entre eles

    15!/6!x9! e dividir pelos result totais

    60!/6!x54!

    alguém poderia apontar o erro nop meu raciocínio? estou quebrando a cabeça aqui

    obrigado


ID
142003
Banca
CESPE / CEBRASPE
Órgão
TRE-MT
Ano
2010
Provas
Disciplina
Raciocínio Lógico
Assuntos

Nas eleições majoritárias, em certo estado, as pesquisas de opinião mostram que a probabilidade de os eleitores votarem no candidato X à presidência da República ou no candidato Y a governador do estado é igual a 0,7; a probabilidade de votarem no candidato X é igual a 0,51 e a probabilidade de votarem no candidato Y é igual a 0,39. Nessa situação, a probabilidade de os eleitores desse estado votarem nos candidatos X e Y é igual a

Alternativas
Comentários
  • Pelos dados do enunciado, temos:

    p(X ou Y) = 0,7
    p(X) = 0,51
    p(Y) = 0,39

    Deseja-se saber quanto vale p(X e Y).

    Da teoria das probabilidades sabemos que p(X ou Y) = p(X) + p(Y) - p(X e Y). Susbstituindo os valores, temos:

    0,7 = 0,51 + 0,39 - p(X e Y)
    p(X e Y) = 0,2.

    Letra B.

    Opus Pi.
  • Alguém pode me responder?

     

    por que não posso multiplicar 0,39  por  0,51 ?

  • B)0,2

    A E B -> 

    70%=(51%+39%)-T

    70%=90%-T

    T=20%

  • Pq não se pode multiplicar 0,39 x 0,51 ????

  • Essa "teoria das probabilidades" seria, especificamente, a probabilidade da união de dois eventos?

  • Creio que não é só multiplicar as duas probabilidades. Observo que se fosse pra ser dessa forma dita anteriormente, teriamos que ter 0,51 + 0,39 sendo = 0,7 e não é isso que acontece na questão. Fiquei nessa questão por um bom tempo e tive como conclusão que, como a soma dos dois eventos não deram 0,7 imagino que exista o evento da intersecção, ou seja, exitem pessoas que votam no candidato X e no candidato Y ao mesmo tempo, então temos que utilizar a PROBABILIDADE DA UNIÃO DOS DOIS EVENTOS. P (X U Y) = P (X) + P (B) - P( A)INTERSECÇÃO B) 

  • Olá galera!


    Acredito que o "pulo do gato" estava na regra do arredondamento:


    > O termo "e" significa multiplicação, que se encontra no final da questão entre o X e Y, logo, se a questão já nos dá o percentual de 0,70 somando ambos os eleitores, falta somente multiplicar os seguintes valores: 0,51 e 0,39 = 0,1989, sendo arredondado para 0,20.

  • x+y-(x*y)=(x+Y)

    0,51+0,39-(x*y)=0,70

    0,90-0,70=(x*Y)

    0,20=(x*Y)

    B=0,2

  • por que não multiplica um pelo outro simplesmente ? alguem pode me explicar ?

  • LETRA B

  • Questão sobre "adição de probabilidades" (ou/União) -> P(XouY) = P(X) + P(Y) - P(XeY)

    "obs: P(XeY) -> E/interseção"

    0,7 = 0,39 + 0,51 - P(X e Y)

    P(X e Y) = 0,90 - 0,70

    P(X e Y) = 0,20

  • Resolvo essa questão aqui nesse vídeo

    https://youtu.be/iHy6ln2s30s

    Ou procure por "Professor em Casa - Felipe Cardoso" no YouTube =D

  • por que não multiplica um pelo outro simplesmente ?

    é o que a banca queria que você fizesse

    ASSERTIVA: Nessa situação, a probabilidade de os eleitores desse estado votarem nos candidatos X e Y é igual a

    .repare o E sinal de interseção o certo é:

    A e B na terioa dos conjuntos é (A interseção B)

    ou seja P(x)= P(A) + P(B) - (A interseção B)

    . 0,7 = 051 + 0,39 - (A interseção B)

    . 0,7= 0,9 - (A interseção B)

    . (A interseção B) = 0,9 - 0,7

    . (A interseção B) = 0,2

    .

  • Decorando a fórmula da teoria de conjuntos da para matar a questão.

    Fórmula => P(A U B)= P(A)+P(B) - P(A ^ B)

    Substituindo => 0.70 = 0.51 + 0.39 - P(A ^ B)

    0.70= 0.90 - P(A ^ B)

    0.70 - 0.90 = P(A ^ B)

    P(A ^ B) = 0.20

  • Total = 100%

    Votar em x = 51%

    Votar em y = 39%

    Votar em nenhum = 30%

    51+39+30 = 120

    120 - 100 = 20%


ID
151519
Banca
CESPE / CEBRASPE
Órgão
TJ-DFT
Ano
2008
Provas
Disciplina
Raciocínio Lógico
Assuntos

Em 2007, os brasilienses passaram mais cheques sem
fundo do que a média dos brasileiros. Em cada mil folhas de
cheque dadas pelo pagamento de algum bem ou serviço, 30 foram
devolvidas na capital federal, enquanto, na média nacional, o
volume cai para 20 a cada mil. Ao longo de 2007, no Brasil,
foram 30 milhões de devoluções, 6 milhões a menos do que em
2006. Em 2006, a inadimplência era de 21 a cada mil folhas.

Correio Braziliense, 24/1/2008 (com adaptações).

A partir das informações apresentadas acima, julgue os itens
subseqüentes.

Considere-se que, no ano de 2007, uma pessoa tenha recebido, de brasilienses, em três dias consecutivos, três folhas de cheque pelo pagamento de algum bem ou serviço. Nessa situação hipotética, a probabilidade de que exatamente uma dessas três folhas tenha sido devolvida será superior a 0,08 e inferior a 0,10.

Alternativas
Comentários
  • Caro Vinicius, não são TODOS os 3 cheques que serão devolvidos. Perceba que a assertiva fala que "exatamente uma dessas três folhas tenha sido devolvida".

    Vejamos a forma correta de fazer os cálculos:

    Probabilidade do cheque voltar = 3/100
    Probabilidade do cheque não voltar = 97/100

    Então a probabilidade de apenas um dos 3 voltar será calculada assim:

    Probabilidade de 1 voltar x probabilidade de 1 não voltar x probabilidade de 1 não voltar x permutação de 3 elementos com 2 repetidos

    A permutação deve ser calculada porque pode ser o 1º, o 2º ou o 3º cheque que voltará...

    Traduzindo:

    3/100 x 97/100 x 97/100 x P32 (É muito cruel ter que fazer essa multiplicação na prova, mas temos que nos acostumar)

    28227/1000000 x (3x2/2)

    84681/1000000

    0,084681

    : )
  • Boa tarde.
    fiz algo aqui que me pareceu mais simples, gostaria que vissem se está correto meu pensamento ou se o acerto foi uma coincidência.
    3/100 = 3% = 0,03
    total de cheques=3
    3x0,03=0,09, ou seja questão certa.
    Estou errado?
  • Sim, você está errado. O raciocínio correto é o do primeiro comentário.
  • Gabarito: CORRETO

    Professor Daniel Lustosa (AlfaCon):

     

    30/1000 = 3/100 = 3% = 0,03 (chance do cheque do brasiliense ser sem fundo).

    Ao receber três cheques a chance de um deles estar sem fundo é:

    P = 0,03 +(ou) 0,03 +(ou) 0,03 = 0,09.

  • Ele recebeu 3 cheques, com a possibilidade de 3% ser sem fundo e 97% ser legal. Ele quer saber a probabilidade de EXATAMENTE 1 ser sem fundo. São 3 as possibilidades:

     cheque ser sem fundo e os outros 2 não: 0.03 x 0,97 x 0,97 = 0,028227

    o  cheque ser sem fundo e os outros 2 não: 0,97 x 0.03 x 0,97 = 0,028227

     cheque ser sem fundo e os outros 2 não: 0,97 x 0,97 x 0.03 = 0,028227

    Souma-se as possibilidades -> 0,028227 + 0,028227 + 0,028227 = 0,084681

     

     

  • 30/1000 =3/100 = 0,03

    0,03*0,97*0,97 * (3!/2!) = 0,084

    Correto

  • CORRETA

    Questão muito boa !!!

    Probabilidade de um inadimplente em brasília =30 em 1000 = 30/1000=0,03=3%

    Probabilidade de não ter um inadimplente em brasília = 0,97=97%

    Probabilidade de um inadimplente + não ter um inadimplente em brasília =3% +97% = 100%

    -----------------------------------------------------------------------------------------------------------------------

    Quero 1 inadimplente e 2 não :

    0,03*0,97*0,97= 0,028227

    Agora temos que multiplicar por 3 .Por quê? Porque não sabemos em que posição o cheque do inadimplente está :

    0,028227 *3=0,084681≃8,4%


ID
152770
Banca
CESGRANRIO
Órgão
Petrobras
Ano
2008
Provas
Disciplina
Raciocínio Lógico
Assuntos

Dois dados comuns, "honestos", são lançados simultaneamente. A probabilidade do evento "a soma dos valores dos dados é ímpar e menor que 10" é igual a

Alternativas
Comentários
  • A questão pede para achar a soma cujos valores sejam ímpares.


    O que eu posso ajudar é só no "braço" mesmo!

    (1,2)=3

    (1,4)=5

    (1,6)=7

    (2,1)=3

    (2,3)=5

    (2,5)=7

    (3,2)=5

    (3,4)=7

    (3,6)=9

    (4,1)=5

    (4,3)=7

    (4,5)=9

    (5,2)=7

    (5,4)=9

    (6,1)=7

    (6,3)=9


    Ou seja,a soma de todos os números não podem ser superiores a 10( 6 e 5 não poderia,por exemplo,pois a soma dos dois dá 11), e tem que ser ímpar(acho que isso não precisa explicar neh,rsrs)


    Espero ter ajudado!!

                                             O

    AH MULequeEeee       <<<|>>>

                                            V V
     

  • par + par = par

    impar + impar = par

    par + impar = impar 

    • P (impar) = P (par) x P (ímpar) x P2

                 P (impar) = 3/6 x 3/6 x 2 = 1/2

     

    Soma > 10 - eventos favoráveis: (6;6) (6;5) (5;5) (5;6) = 4 eventos

    Eventos possíveis no lançamento de dois dados - 6 x 6= 36

    P (Soma dos valores maiores que 10) = 4/36 = 1/9

    • P (Soma dos valores menores que 10) = 8/9

     

    P (ímpar e menor que 10) = 1/2 x 8/9 = 4/9

     

    Resposta: C

  • Dois dados comuns, "honestos", são lançados simultaneamente. A probabilidade do evento "a soma dos valores dos dados é ímpar e menor que 10" é igual a

    Os números, ímpar, menor que 10 são.  1, 3, 5, 7, e 9.

    OBS:  O número "1" é descartado, por que, não há possibilidade, de uma soma com dois dados "honestos", resultar, em "1"

    ou seja:       (1 ,2) =3
                   (1,4) (2,3) =5
          (1,6) (2,5) (3,4) =7
                   (3,6) (4,5) =9 
     Já os outros eventos, são irrelevantes,por que, ocorrem apenas uma mundança de fatores, ou seja, a ordem dos fatores não alteram o produto.
    Basicamente ocorreram "apenas", 4 resultado,que são eles: 3, 5, 7 e 9.
     E tambêm 8 eventos.
     Ou seja, 4/8

     Não sei o motivo, de 4/9. pra mim, esta questão deveria ser anulada.
  • Não fiz com muita técnica, mas deu certo:
     
    O total de possibilidades são 36 (6x6 faces do dado).
    Para os valores darem um número ímpar E menor que 10, eu fiz as combinações manualmente:
     
    1º dado + 2º dado jogados simultaneamente:
     
    1,2 = 3. Ou seja a soma é ímpar e menor que 10! E assim sucessivamente:
     
    1,4
     
    1,6
     
    2,1
     
    2,3
     
    2,5
     
    3,2
     
    3,4
     
    3,6
     
    4,1
     
    4,3
     
    4,5
     
    5,2
     
    5,4
     
    6,1
     
    6,3
     
    O máximo de combinações para um resultado ímpar e menor que 10 são: 16.
    Logo, devemos dividir as possibilidades do evento pelo total:  16 = 4.
                                                                                                                     36     9

    Espero ter ajudado!
  • D1    D2        S = 6*6 = 36
    1          1        
    2          2
    3          3
    4          4
    5          5
    6          6

    Soma ímpar e menor que 10: a soma deve ser igual a 1, 3, 5, 7 e 9

    Soma 1: 0 possibilidades
    Soma 3: (1,2) (2,1) 2 possibilidades.     2/36
    Soma 5: (1,4) (2,3) (3,2) (4,1) 4 possibilidades       4/36
    Soma 7: (1,6) (2,5), (3,4) (4,3), (5,2) (6,1) 6 possibilidades        6/36
    Soma 9: (3,6) (4,5) (5,4) (6,3) 4 possibilidades             4/36


    2/36 + 4/36 + 6/36 + 4/36 = 16/36 = 4/9
  • par  ---------- impar

    2,4,6 -------- 1,3,5

    1 e 2

    1 e 4

    1 e 6

    3 e 2

    3 e 4

    3 e 6

    5 e 2

    5 e 4

    5 e 6 não, porque é maior que 10...

    8/36 . 2

    16/36

    8/18

    4/9


  • Mesmo com todas as explicações continuo sem entender, alguém poderia explicar de uma forma mais detalhada ainda? Obrigada. :(

  • VOU PASSAR UM BIZU PARA TODAS QUESTÕES ENVOLVENDO SOMA DE DOIS DADOS:

    FAÇA ESSA LEITURA ATENTA QUE VOÇÊ VAI OBSERVAR A LÓGICA, QUANDO CHEGA  NO 8 COMEÇA A DECRESCER NOVAMENTE 
    ATÉ O 12!!! TREINE ESCREVENDO ESSA TABELA VARIAS VEZES QUE VC VAI FICAR COM ELA NA MENTE.

    JOGANDO DOIS DADOS SOMA 2-------> 1/6.1/6= 1/36
    JOGANDO DOIS DADOS SOMA 3-------> 2/6.1/6= 2/36
    JOGANDO DOIS DADOS SOMA 4-------> 3/6.1/6= 3/36
    JOGANDO DOIS DADOS SOMA 5-------> 4/6.1/6= 4/36
    JOGANDO DOIS DADOS SOMA 6-------> 5/6.1/6= 5/36
    JOGANDO DOIS DADOS SOMA 7-------> 6/6.1/6= 6/36
    JOGANDO DOIS DADOS SOMA 8-------> 5/6.1/6= 5/36
    JOGANDO DOIS DADOS SOMA 9-------> 4/6.1/6= 4/36
    JOGANDO DOIS DADOS SOMA 10-------> 3/6.1/6= 3/36
    JOGANDO DOIS DADOS SOMA 11-------> 2/6.1/6= 2/36
    JOGANDO DOIS DADOS SOMA 12-------> 1/6.1/6= 1/36

     

    AGORA VAMOS RESOLVER A QUESTÃO USANDO ESSA TABELA!!

    A QUESTÃO PEDE A SOMA QUE SEJA MENOS QUE 10 E QUE SEJE IMPARES ESSAS SOMAS, ENTÃO;

     

    JOGANDO DOIS DADOS SOMA 3-------> 2/6.1/6= 2/36

    JOGANDO DOIS DADOS SOMA 5-------> 4/6.1/6= 4/36        

    JOGANDO DOIS DADOS SOMA 7-------> 6/6.1/6= 6/36

    JOGANDO DOIS DADOS SOMA 9-------> 4/6.1/6= 4/36

    SOMANDO TUDO E MANTENDO MEU ESPAÇO AMOSTRAL  FICA    16/36 ----> SIMPLIFICANDO 4/9

     

    ESPERO TER AJUDADO OS COLEGAS QUE FICARAM EM DUVIDA!!!

  • Fiz da seguinte maneira:

    São 36 combinações no total.

    Dessas, 4 tem a soma maior ou igual a 10 { 5+5; 6+5; 5+6; 6+6}

    Restam 32 combinações com a soma menor a 10.

    Como são 32 somas, sabe-se que a metade delas vai ser ímpar (isso eu conclui na observação mesmo).

    Assim, 16 combinações tem a soma ímpar e menor a 10.

    P =   16/36  =  8/18  =  4/9

  • P (soma impar e soma<10)

     

    par + par = par

    impar + impar = par

    par + impar = impar

     

    Dado1  Dado 2        

    2              1                                 

    4              3                               

    6              5           

     

    Possibilidades: (2+1), (2+3), (2+5), (4+1), (4+3), (4+5), (6+1), (6+3), (6+5 não serve)

                               (1+2), (3+2), (5+2), (1+4), (3+4), (5+4), (1+6), (3+6), (5+6 não serve )    

     

    Espaço Amostral = 6*6 = 36 

     

    Então a probabilidade é de 16/36 = 4/9

     

    gab C

     

  • UMA DICA SIMPLES FAZ A COLUNA.

    1= (1.2),(1,4) (1,6)

    2= (2,1),(2,3),(2,5)

    3= (3,2),(3,4),(3,6)

    4= (4,1), (4,3), (4,5)

    5= (5,2),(5,4)

    6=(6,1),(6,3)

    OUTRA DICA.

    CHEGOU NO NUMERO 2 DA COLUNA, OLHA PARA A COLUNA DE CIMA NO CASO A 1 ( ENCONTROU O NUMERO 2 INVERTE A ORDEM)

    PASSO 2 =

    6*6 = 36 TOTAL ( UM DADO TEM 6 FACES COMO É DOIS DADOS MULTIPLICA 6*6)

    PASSO 3 =

    CONTA OS NÚMEROS DE ELEMENTOS QUE DERAM NUMERO IMPAR E MENOR QUE 10

    DEU = 16

    PASSO 4 =

    16/36 = FRAÇÃO PORTANTO = DIVIDE 16/4 = 4 DIVIDE 36/4 = 9

    RESULTADO = 4/9

    GABARITO = C

  • soma igual a 02: 1+1 ----------------------------------------(1 POSIBILIDADE)

    soma igual a 03: 2+1, 1+2 --------------------------------(2 POSIBILIDADE)

    soma igual a 04: 3+1, 1+3, 2+2 ------------------------(3 POSIBILIDADE)

    soma igual a 05: 4+1, 1+4, 3+2, 2+3 ----------------(4 POSIBILIDADE)

    soma igual a 06: 5+1, 1+5, 4+2, 2+4, 3+3 --------(5 POSIBILIDADE)

    soma igual a 07: 6+1, 1+6, 5+2, 2+5, 4+3, 3+4 (6 POSIBILIDADE)

    soma igual a 08: 6+2, 2+6, 5+3, 3+5, 4+4 --------(5 POSIBILIDADE)

    soma igual a 09: 6+3, 3+6, 5+4, 4+5 ----------------(4 POSIBILIDADE)

    soma igual a 10: 6+4, 4+6, 5+5 ------------------------(3 POSIBILIDADE)

    soma igual a 11: 6+5, 5+6 --------------------------------(2 POSIBILIDADE)

    soma igual a 12: 6+6 ----------------------------------------(1 POSIBILIDADE)

    Possiblidades totais = 1+2+3+4+5+6+5+4+3+2+1 = 36 (ou 6*6)

    Ímpar e menor que 10------------= 2+4+6+4 = 16

    Portanto, 16/36 ou dividindo por 4 temos: 4/16.

    Gabarito --> (Letra C)

  • Temos 4 somas possíveís.

    Soma igual a 3: 1+2

    Soma igual a 5: 1+4; 2+3

    Soma igual a 7: 1+6; 2+5; 3+4

    Soma igual a 9: 3+6; 4+5

    São 8 somas que dão ímpares menores que 10.

    Como jogamos 2 dados são 6x6 = 36 possibilidades, mas como jogamos juntos e queremos saber a soma, a ordem não importa, então dividimos por 2.

    Poderia calcular em cima de 36 mas na soma que fiz acima eu teria que considerar os pares 2,1, mas vejam, 2,1 e 1,2 pra soma é a mesma coisa, não importa, por isso considero 18.


ID
156379
Banca
CESPE / CEBRASPE
Órgão
TRT - 1ª REGIÃO (RJ)
Ano
2008
Provas
Disciplina
Raciocínio Lógico
Assuntos

Em um setor de uma fábrica trabalham 10 pessoas que serão divididas em 2 grupos de 5 pessoas cada para realizar determinadas tarefas. João e Pedro são duas dessas pessoas. Nesse caso, a probabilidade de João e Pedro ficarem no mesmo grupo é

Alternativas
Comentários
  • O número total de possilidades de ocorre um dos grupos é feito pela C10,5 que é igual à 252 (Espaco amostral do evento).
    Agora iremos estudar as possibilidades para ocorrer a situação desejada na questão que é a probabilidade de João e Pedro ficarem juntos no mesmo grupo(independentemente de qq grupo), então:

    Considere que no primeiro grupo a probabilidade de eles ficarem juntos é
    P(A)=C8,3/C10,5=56/252

    E por outro lado, se pensarmos que eles ficaram juntos no segundo grupo teremos que a probabilidade é
    P(B)=C8,3/C10,5=56/252
    ou seja, envolve a probabilidade da união ( João e Pedro no 1 grupo ou João e Pedro no  2 grupo)

    P(A U B)= P(A) + P(B) =C8,3/C10,5 + C8,5/ C10,5= (56+56)/252 = 0,444...

    Portanto, letra D

  • LETRA D!

    Comentários do Prof. Leandro S. Vieira:
    Se João for a primeira pessoa a estar em um grupo, restarão 9 pessoas (incluindo Pedro) para serem distribuídas entre os dois grupos. Pedro concorrerá com 9 pessoas pelas 4 vagas que ainda restam no grupo onde João já está. Portanto Pedro tem 4/9 de probabilidade de ficar no mesmo grupo de João. 4/9 = 0,44 ou 44%.
     

  • Os grupos serão "A" e "B".

    Probabilidade de João ficar no grupo "A" = 1/2 (Uma possibilidade em dois grupos. Ou joão fica no "A" ou fica no "B".)
    Probabilidade de Pedro preencher uma das outras quatro vagas que sobraram no grupo "A" = 4/9 (Quatro vagas disputadas pelas 9 pessoas que restaram.)
    Logo, a probabilidade de JOÃO e PEDRO ficarem no grupo "A" será: A = 1/2 * 4/9 =  0,22222...

    O mesmo pensamento deve ser repetido para a possibilidade de JOÃO E PEDRO ficarem no grupo "B".
    B = 1/2 * 4/9 = 0,22222...

    Como a questão pede apenas a probabilidade de os dois ficarem no mesmo grupo, não importa se é no grupo "A" ou no "B".
    Então a resposta será a soma das duas propabilidades (os dois no "A" ou os dois no "B").

    Resposta = 0,22222 + 0,22222 = 0,44444

    0,42 < 0,44 < 0,46
    Portanto LETRA D.
  • Grupo A => C5,2 / C10,2 = 2/9

    Grupo B => C5,2 / C10,2 = 2/9

    Somando a probabilidade do Grupo A com a do Grupo B, teremos 4/9 = 0,44.

    GABARITO: Letra D.

     

  • Nossa, tão simples  por essa visão!!

    Mari NZH

    11 de Agosto de 2010, às 10h49

    Útil (58)

    LETRA D!

    Comentários do Prof. Leandro S. Vieira:
    Se João for a primeira pessoa a estar em um grupo, restarão 9 pessoas (incluindo Pedro) para serem distribuídas entre os dois grupos. Pedro concorrerá com 9 pessoas pelas 4 vagas que ainda restam no grupo onde João já está. Portanto Pedro tem 4/9 de probabilidade de ficar no mesmo grupo de João. 4/9 = 0,44 ou 44%.

     

     

  • TOTAL = C 10,5 * C 5,5= 252

    João e Pedro na PRIMEIRA equipe = C 8,3 * C 5,5 = 56

    João e Pedro podem estar tanto na primeira, quanto na segunda = 2! = 2*1 = 2

    56* 2 = 112

    112/252= 0,44

  • LETRA D

  • Probabilidade de ambos estarem no mesmo grupo:

    5/10 x 4/9 = 0,22

    Como eles podem estar no 1º OU no 2º, basta somar a probabilidade dos 2 grupos, ou seja, 0,22 + 0,22 = 0,44.

    Gab: Letra D.


ID
156772
Banca
CESPE / CEBRASPE
Órgão
TRT - 1ª REGIÃO (RJ)
Ano
2008
Provas
Disciplina
Raciocínio Lógico
Assuntos

De acordo com informações apresentadas no endereço
eletrônico www.trtrio.gov.br/Administrativo, em fevereiro de
2008, havia 16 empresas contratadas para atender à demanda de
diversos serviços do TRT/1.ª Região, e a quantidade de
empregados terceirizados era igual a 681.

Se, entre as 16 empresas contratadas para atender aos serviços diversos do TRT, houver 4 empresas que prestem serviços de informática e 2 empresas que cuidem da manutenção de elevadores, e uma destas for escolhida aleatoriamente para prestar contas dos custos de seus serviços, a probabilidade de que a empresa escolhida seja prestadora de serviços de informática ou realize a manutenção de elevadores será igual a

Alternativas
Comentários
  • Espaço amostral 16
    número procurado 6 (2 + 4)
    6/16 = 0,375
  • 4/16 + 2/16 = 0,375

  • LETRA C

  • no “ou” soma // no “e” é interseção (em comum). A probabilidade então será a soma das duas: 2/6 + 4/16 = 6/16 = 0,375

ID
158053
Banca
CESGRANRIO
Órgão
TJ-RO
Ano
2008
Provas
Disciplina
Raciocínio Lógico
Assuntos

Dois dados comuns, "honestos", são lançados simultaneamente. A probabilidade de que a soma dos resultados seja igual ou maior que 11 é

Alternativas
Comentários
  • Os resultados possíveis são:
     
    (6,6)
    (6,5)
    (5,6)
     
    Portanto, 3/36 ou 1/12
  • Uma dúvida que ocorrer é: Por que a probabilidade não é 2/36, já que os resultados são a variação entre 6,6 e 5,6? Como abaixo representado, onde o resultado 2 e 3 são os mesmos. Para esclarecer a dúvida é só lembrar que se tratam de 2 dados, independentes. Seria 2/36 se fosse o mesmo dado jogado  duas vezes.
    resultado 1:
    dado 1: 6 dado 2: 6

    resultado 2:
    dado 1: 5 , dado 2: 6

    resultado 3:
    dado1: 6 , dado2:  5
  • O máximo de possibilidade é (6 faces e 6 faces) = Espaço Amostral = 36

    Para que o resultado seja >=11 as possibilidades são (5,6) ou (6,5) ou (6, 6) = Evento = 3

    Lembrando que pelo PFC (princípio fundamental da contagem), "e" multiplica e "ou" soma.

    P=  evento / espaço amostral = 3/36 = 1/12

    Alternativa correta : "c"

  • Fátima considerando uma hipótese que os dados tenham cores diferentes isso significa que são opções diferentes de soma, exemplo uma dado branco e outro vermelho, o Branco cai no 5 e vermelho no 6, é diferente do do branco 6 e vermelho 5, por isso são 3 casos possíveis e 36 casos favoráveis. 3/36= 1/12 resposta letra c


ID
158056
Banca
CESGRANRIO
Órgão
TJ-RO
Ano
2008
Provas
Disciplina
Raciocínio Lógico
Assuntos

Uma urna contém dez bolas, cada uma gravada com um número diferente, de 1 a 10. Uma bola é retirada da urna aleatoriamente e X é o número marcado nesta bola. X é uma variável aleatória cujo(a)

Alternativas
Comentários
  • Se a probabilidade é 1/10 para qualquer sequência de valores, portanto ela é uniforme.
  • D

    É uniforme porque qualquer número que sair de primeira a probabilidade vai ser a mesma 1/10

  • Ficou meio estranho enunciado. Pois se for retirado cada bola sem reposição a probabilidade seria de:

    P(1) = 1/10

    P(2) = 1/9

    P(3) = 1/8

    ...

    Logo, o examinador considerou que cada bola seria reposta na urna antes de retirar outra novamente.


ID
164518
Banca
FGV
Órgão
BADESC
Ano
2010
Provas
Disciplina
Raciocínio Lógico
Assuntos

Mariano distribuiu 3 lápis, 2 borrachas e 1 caneta pelas 3 gavetas de sua cômoda. Adriana, sua esposa, abriu uma das gavetas e encontrou, dentro dela, 2 lápis e 1 caneta. Sabendo-se que nenhuma das 3 gavetas está vazia, analise as afirmativas a seguir:

I. É possível garantir que, abrindo-se qualquer outra gaveta, encontra-se pelo menos uma borracha.

II. É possível garantir que, abrindo-se qualquer outra gaveta, encontra-se um único lápis.

III. É possível encontrar, em uma das gavetas, mais de uma borracha.

Assinale:

Alternativas
Comentários
  • Resposta : Letra c)
     
    Restarão apenas 2 borrachas e um lápis e em nenhuma das gavetas poderá estar vazia.

    Sendo assim restarão as seguintes possibilidades :

     2° Gaveta                     3° Gaveta

         BB                                 L
         BL                                 B
          L                                  BB
          B                                  BL

    Verificando as afirmativas veremos que só a 3° está correta

  • Engraçado, para mim parecia que todas estavam corretas...;-)
  • Para mim a I e a III estão corretas.

    I. É possível garantir que, abrindo-se qualquer outra gaveta, encontra-se pelo menos uma borracha.
    III. É possível encontrar, em uma das gavetas, mais de uma borracha.

    Pode ser que fique uma borracha em cada gaveta e o lápis seja colocado na gaveta que já tenha outros objetos.
    Mas entendo que não há como não ter pelo menos 1 borracha nas 2 gavetas restantes.
  • Aos que ficaram com dúvida quanto à resposta, basta atentarem-se para as afirmativas:

    É POSSÍVEL GARANTIR......  não, não é possível garantir as afirmativas  I ou  II

    Já a afirmação : É POSSÍVEL ENCONTAR .... torna  válida a afirmativa III.

  • Vou tentar ajudar aqueles que não entenderam refutando as assertivas erradas:

    Veja bem:
    Se foram encontrados 2 lápis e 1 caneta, eu sei que me sobraram nas demais gavetas: 1 lápis e 2 borrachas.

    Assertiva 1:  "É possível garantir que, abrindo-se qualquer outra gaveta, encontra-se pelo menos uma borracha. "
    NÃO, pois as borrachas podem estar todas em uma gaveta e na outra estar apenas o lápis, deste modo, não tem como eu garantir (ter 100% de certeza) que independente da gaveta que eu abrir estará lá uma borracha.

    Assertiva 2: "É possível garantir que, abrindo-se qualquer outra gaveta, encontra-se um único lápis."
    NÃO, pois não pode ser "qualquer outra gaveta", já que o lápis vai estar só em UMA das gavetas. Assim, se o lápis estiver na segunda gaveta e eu abrir a 3ª; ele não vai estar lá!

    :D
  • São 3 gavetas, 3 lápis, 2 borrachas e 1 caneta. Adriana encontrou em 1 das gavetas 2 lápis e 1 caneta, o que deixa disponível somente 1 lápis e 2 borrachas, as afirmativas:

    I. É possível garantir que, abrindo-se qualquer outra gaveta, encontra-se pelo menos uma borracha. 
    Incorreto. Porque podem existir 2 borrachas em uma só gaveta.

    II. É possível garantir que, abrindo-se qualquer outra gaveta, encontra-se um único lápis. 
    Incorreto. Sobrou apenas um lápis, logo ele só pode estar dentro de uma única gaveta.

    III. É possível encontrar, em uma das gavetas, mais de uma borracha. 
    Correto. Sobraram 2 borrachas, e sim, é possível encontrá-las em uma das gavetas. Vejam que aqui a questão não fala em qualquer outra gaveta. 

    Espero ter ajudado! Bons estudos ;)
  • Questão tranquila, basta considerar:

    Nenhuma gaveta vazia;

    Gaveta 1= 2 lápis + 1 Caneta, portanto, sobrará 1 lápis e 2 Borrachas a serem distribuídas nas duas outras gavetas, sendo que nenhuma gaveta poderá ficar vazia

    Gaveta 2 = Poderá ficar: 1B, 2B, 1L ou 1Le1B

    Gaveta 3 = O que sobrar

    pelos dados informados, não podemos garantir nada, então, só nos sobra a alternativa "C".


ID
164530
Banca
FGV
Órgão
BADESC
Ano
2010
Provas
Disciplina
Raciocínio Lógico
Assuntos

Um dado é dito "comum" quando faces opostas somam sete. Deste modo, num dado comum, o 1 opõe-se ao 6, o 2 opõe-se ao 5 e o 3 opõe-se ao 4.

Lançando-se duas vezes seguidas um mesmo dado comum, os resultados obtidos são descritos por um par ordenado (a,b), em que a é o resultado obtido no 1º lançamento e b, o resultado obtido no 2º lançamento.

Assinale a alternativa que indique, corretamente, quantos pares ordenados diferentes podem ser obtidos de modo que a soma dos resultados seja sempre igual a 8.

Alternativas
Comentários
  • Para que a soma seja 8 devemos ter os seguintes pares:(2,6),(3,5),(4,4),(5,3) e (6,2)Portanto 5 pares diferentes geram soma 8.RESPOSTA: (D)
  • Marquei 3: (2,6),(3,5),(4,4) Não ví o porque da repetição invertida dos dois primeiros pares. ¬¬
  •                         Dado A e  Dado B pares ordenados diferentes =

    lançamentos             A       e       B

          1 ª                       2                 6 =      8

          2ª                        3                 7=       8

          3ª                        4                 4 =      8

          4ª                        6                 2 =     8

          5ª                        7                 3=      8

    resposta D

  • questão um pouco dúbia já que não informa se os lançamentos podem ou n serem repetidos e nem se a ordem interfere no resultado

  • Resposta: Letra D

     

    A chave dessa questão é obsevar no enunciado que ele pergunta quantos pares ordenados diferentes podem ser formados.

    A palavra diferentes se apresenta pra gerar uma certa confusão no candidato.

    O par ordenado (3, 5), por exemplo, é diferente do par ordenado (5, 3). Portanto, não é uma repetição.

    Logo temos as seguintes possibilidades:

    (2, 6) = 8

    (6, 2) = 8

    (3, 5) = 8

    (5, 3) = 8

    (4, 4) = 8 

  • so uma duvida, o par ordenado (4,4) nao seria igual? sendo q a questão pede diferentes?
  • Thiago Barbosa, são pares diferentes, não são "valores diferentes".

    Resposta: D
  • Essa questão não é de probabilidade...


ID
165436
Banca
ESAF
Órgão
MPU
Ano
2004
Provas
Disciplina
Raciocínio Lógico
Assuntos

Carlos diariamente almoça um prato de sopa no mesmo restaurante. A sopa é feita de forma aleatória por um dos três cozinheiros que lá trabalham: 40% das vezes a sopa é feita por João; 40% das vezes por José, e 20% das vezes por Maria. João salga demais a sopa 10% das vezes, José o faz em 5% das vezes e Maria 20% das vezes. Como de costume, um dia qualquer Carlos pede a sopa e, ao experimentá-la, verifica que está salgada demais. A probabilidade de que essa sopa tenha sido feita por José é igual a

Alternativas
Comentários
  • É simples, deve-se multiplicar 40% x 5% = 200% divide por 100, pois é porcentagem. A multiplicação é por que quando José cozinha e salga, o conectivo "e" deve-se multiplicar os acontecimentos, caso fosse o conectivo "ou" somariamos.
  • COMENTADA PELO PRO. RONILTON LOYOLA
     Veja que temos aqui também uma probabilidade condicional: “Qual a probabilidade de que essa sopa tenha sido feita por José, condicionada ao fato de estar salgada?”
    Então, p(E2/E1) = p(E1 E2) / p(E1), onde:
    p(E1): é a probabilidade de a sopa estar salgada (feita por João, José “ou” Maria);
    p(E1 E2):  é a probabilidade de a sopa ter sido feita por José “e” estar salgada;
    p(E2/E1): é a probabilidade de a sopa ter sido feita por José, condicionada ao fato de estar salgada.
    - Vamos calcular p(E1):
    A probabilidade de a sopa estar salgada, seja lá quem tenha feito é dada por                                p(E1) = 40%.10% + 40%.5% + 20%.20% = 10%. (veja que somamos as probabilidades por que um “ou” outro fez a sopa salgada)
    - Vamos calcular p(E1 E2)
    A probabilidade de a sopa ter sido feita por José “e” estar salgada é dada por                                    p(E1 E2) = 40%.5% = 2%.
    Logo, a probabilidade de que a sopa tenha sido feita por José, condicionada ao fato de estar salgada é dada por: p(E2/E1) = p(E1 E2) / p(E1) = 2%/10% = 2/10 = 0,20 (ou 20%).
    Gabarito: letra D.
  • Eu resolvi da seguinte forma:

    3 cozinheiros:
    De cada 100 vezes que Carlos vai ao restaurante:

    40% (40 vezes)  - sopa feita por João  -  10% sopa feita por João tá salgada  -  (10% de 40 = 4)
    40% (40 vezes)  - sopa feita por José  -  05% sopa feita por José tá salgada  -  (05% de 40 = 2)
    20% (40 vezes)  - sopa feita por Maria -  20% sopa feita por Maria tá salgada  -  (20% de 20 = 4)

    Se de cada 100 vezes que Carlos consome a sopa salgada e José fez 2 vezes (a sopa salgada) - logo, 0,20.



  • PLOBABILIDADE DA INTERSECÇÃO DE EVENTOS

    A probabilidade de intersecção de eventos ocorre quando
     o exercício pede para que seja respondida a probalidade 
    de um evento A  e um evento B.
     
    CONCLUINDO: A sopa feita por José(A) e a sopa salgado por José(B) = 0,4  x  0,05 = 0,20



    ADICIONAL :

    OU = SOMA

     E = MUTIPLICAÇÂO


  • As respostas de wandersonDryka Bravim e Rogerio Leal estão ERRADAS, pois seus cálculos dão como resposta 0,02 ou 2% e a resposta correta é 0,20 ou 20%.
  • Chance de ter sido feito e salgada por João: (4/10 * 1/10) = 0,04 
    Chance de ter sido feito e salgada por José: (4/10 * 0,5/10) = 0,02
    Chance de ter sido feito e salgada por Maria: (2/10 * 2/10) = 0,04
    Chance de ter sido feita por José = 0,02/(0,04+0,02+0,04) = 0,02/0,10 = 0,20 
     
  • Supondo que o total seja de 100 sopas, então:

    João: 40%   logo, 40 sopas. Sendo 10% salgadas, então 4 sopas salgadas;

    José: 40%   logo, 40 sopas. Sendo 5% salgadas, então 2 sopas salgadas;

    Maria: 20%  logo, 20 sopas. Sendo 20% salgadas, então 4 sopas salgadas.


    Como já sabe que a sopa está salgada, o universo passa a ser 10.

    Portanto, total de sopas salgadas por José / universo de sopas salgadas = 2/10 = 0, 2 ou 20%

  • Discordo COMPLETAMENTE desse gabarito. A pergunta da questão não é "qual é a probabilidade de a sopa estar salgada e ter sido feita por José?", e sim "a sopa ESTAVA salgada. Qual a probabilidade de ter sido feita por José?". Ou seja, você já tá partindo do pressuposto de que ela estava salgada, então não há porque incluir essa variável no cálculo da probabilidade. O cálculo deveria ser, 35% é do total das sopas são salgadas, desses 35%, 5% foram feitas por José, o que dá 14% do total do sopas salgadas. Essa deveria ser a resposta, é uma regra de 3 simples.

  • Questão clássica de probabilidade condicional. Resolução sem fórmula:

    Conforme dito no problema, sabemos que a sopa já está salgada. Isso é fato, portanto, uma CONDIÇÃO IMPOSTA, que restringirá o nosso espaço amostral para o nº de possibilidades em que a sopa está salgada apenas, não importando as probabilidades ocorridas sem a sopa estar salgada. Por isso, vamos calcular as probabilidades ocorridas com a sopa ficando salgada, para todos os 3 cozinheiros:

    JOAO -> 4/10 * 10/100 = 40/1000
    JOSE -> 4/10 * 5/100 = 20/1000
    MARIA -> 20/100 * 2/100 = 40/1000


    Somando todas as possibilidades de ficar salgada, teremos o nosso espaço amostral de 100/1000. Queremos a probabilidade de ficar salgada quando feita por José, portanto, pegamos a prob de José, e aplicamos o principio da probabilidade:

    20/1000 = 0,02 -> josé

    dividido por

    100/1000 = 0,1 -> espaço amostral

    0,02/0,1 = 20%


    .

  • Número de sucessos(no presente caso um insucesso de se fazer uma sopa salgada) após N repetições(Possibilidade do cozinheiro X fazer a sopa) do Experimento (Sopa) =

    N x p = 5%(probabilidade dele ter salgado a sopa) X 40%(probabilidade do José ter feito a sopa) = 200/10=20, 20/100= 0,2.

  • De 100 sopas, João prepara 40, José 40 e Maria 20.

     

    Das 40 sopas de João, 10% delas, (4), estão salgadas. 

    Das 40 sopas de José, 5% delas, (2), estão salgadas.

    Das 20 sopas preparadas por Maria, 20%, (4), estão salgadas. 

     

    Então se Carlos percebeu que a sopa estava salgadas ela só pode estar entre as 10. E a probabilidade de ela ter sido preparada por José é de 2/10 = 20%.

  • De cada 100 sopas tomadas por Carlos, espera-se que 40 tenham sido feitas por João, 40 por José e as 20 restantes por Maria.

           Das 40 sopas feitas por João, 10% são salgadas, ou seja, 4 são salgadas. Das 40 sopas feitas por José, 5% são salgadas, isto é, 2 sopas. E das 20 sopas de Maria, 20% são salgadas, totalizando 4 sopas salgadas.

           Assim, de cada 100 sopas tomadas por Carlos espera-se que 4 + 2 + 4 = 10 sejam salgadas. Este é o total de sopas salgadas:

    Total de sopas salgadas = 10

           Destas, apenas 2 foram feitas por José:

    Sopas salgadas de José = 2

           Portanto, a probabilidade de que a sopa salgada seja feita por José é:

    Probabilidade = 2 / 10 = 0,20

    Resposta: D

  • GABARITO: Letra D

    Essa questão é clássica de Teorema de Bayes, estudado em estatística.

    Probabilidade = Quero / Total

    Eu quero sopa salgada feita pelo José: 40x5 = 200

    O total é a probabilidade de cada um fazer a sopa x probabilidade de salgar = 40x10+40x5+20*20

    Probabilidade = 200/1000 = 0,2

  • GABARITO LETRA D

    Resolvido pelo Método de Bayes

    P(Ser José/Está Salgado) = P(Ser José) * P(Ser José/Está salgado) / P(Ser José) * P(Ser João/Está salgado) + P(Ser João) * P(Ser João/Está salgado) + P(Ser Maria) * P(Ser Maria/Está salgado)

    40/100 * 5/100 / 40/100 * 5/100 + 40/100 * 10/100 + 20/100 * 20/100

    = 200/ 10.000 / 1000 / 10.000 = 2/100 / 1/10 = 2/100 * 10 = 20 / 100 = 0,2 * 100% = 20 %


ID
166096
Banca
ESAF
Órgão
MPU
Ano
2004
Provas
Disciplina
Raciocínio Lógico
Assuntos

Maria ganhou de João nove pulseiras, quatro delas de prata e cinco delas de ouro. Maria ganhou de Pedro onze pulseiras, oito delas de prata e três delas de ouro. Maria guarda todas essas pulseiras - e apenas essas - em sua pequena caixa de jóias. Uma noite, arrumando-se apressadamente para ir ao cinema com João, Maria retira, ao acaso, uma pulseira de sua pequena caixa de jóias. Ela vê, então, que retirou uma pulseira de prata. Levando em conta tais informações, a probabilidade de que a pulseira de prata que Maria retirou seja uma das pulseiras que ganhou de João é igual a

Alternativas
Comentários
  • Maria ganhou de João 9 pulseiras : 4 prata e 5 ouro

    Maria ganhou de Pedro 11 pulseiras: 8 prata e 3 ouro

    As pulseiras de ouro são só para encher linguiça, pois sabemos que Maria já tirou, ao acaso, a pulseira de prata então:

    As 4 pulseiras de prata que João deu são divididas pelo total de pulseiras de prata que têm na caixa de jóias:

    4/12 simplificando 1/3 Resposta "A"

  • Trata-se de probabilidade condicionada. O comentário acima foi, ao meu ver, correto. Nesse tipo de questão, caso não consiga ter o raciocínio acima, poderá fazê-lo da seguinte maneira:
    Na probabilidade condicionada tem-se que : P(a/b)=P(a^b) / P(b)- lê-se : A probabilidade de "a" tendo em vista que ocorreu "b" é igual a probabilidade de "a" e "b"(ocorrerem simultaneamente) dividido por probabilidade de "b".
    Bom para resolvê-la é fazendo a "árvore da propabilidade"
           
                                      ---4/9--- Prata
        ---------9/20 ----Joaõ    
                                     
      ---5/9--- Ouro                     
     
     M
                                        ---8/11--- Prata
       ---------11/20 ----Pedro
                                        ---3/11---Ouro

    P(João e Prata) = 9/20 * 4/9 = 1/5 
    P (prata) = 1/5 + (11/20 * 8/11) = 3/5
    Na Fórmula temos : P(João/Prata)= (1/5 )/ (3/5) = 1/3 ///




     




     







     

  • Resposta Letra A

    Joao deu 9 pulseiras
    4 pratas
    5 ouro

    Pedro deu 11 pulseiras
    8 pratas
    3 ouro

    Ele pediu a probabilidade entre as pulseiras PRATAS

    4 + 8 = 12 então

    Joao deu 4
    P(J) = 4/12 simplificando
    P(J) = 1/3

    Bons Estudos.....
  • Mas se todas as pulseiras estao dentro da caixa, o correto nao seria fazer a probabilidade de 4/20?? Sendo 4 pulseiras pratas do João por 20 pulseiras total???
    Daí o resultado seria 1/5.

  • Questão de probabilidade condicionada. 


                    OURO    PRATA

    João             5              4

    Pedro           3               8


    Eventos favoráveis = 4

    Espaço amostral = 12


    P = 4/12

    P = 1/3
  • A questão afirma que Maria já retirou uma pulseira da caixa, e ela é de prata. Portanto, deve-se calcular a probabilidade levando em consideração somente as pulseiras de prata. Caso a questão pedisse a probabilidade de Maria pegar uma pulseira de prata dada por Joao antes que a moça a tirasse da caixa, seria necessário levar em consideração as de ouro também.
  • Não consigo entender o motivo de não se considerarem todas as 20 pulseiras no espaço amostral...

  • Hugo, no enunciado diz que é uma de prata que ela retirou, ou seja, o espaço amostral é de apenas 12 pulseiras de prata.

  • Ao meu ver, o epaço amostral é de 20, pois ela retirou uma pulseira DA CAIXA, como diz o enunciado, e nesta há as 20 pulseiras! Questão passível de anulaçao!! 

  • Na caixa tinham 20 pulseiras. Dessas 20 somente 4 eram de prata e forama dadas por joão. Então a probabilidade de pegar uma de prata dada por João é 4/20! 

    Não consigo ver da forma que os colegas falaram! 

  • Essa Maria... ( ͡° ͜ʖ ͡°)

  • A questão é f-d-p, mas o examinador deixa claro "...a probabilidade de que a pulseira de prata que Maria retirou seja uma das pulseiras que ganhou de João..."

    O espaço amostral é as pulseiras de prata, não todas as pulseiras de dentro da caixa.

    Se vc visualizar fica melhor:

    Ela pega a pulseira de prata olha pra pulseira de prata e olha "será que foi o c-o-r-n-o do João que me deu essa pulseira de prata?"

  • Letra A

    a. Determinar o espaço amostral: o espaço amostral é condicional, pois no texto temos que “Maria retira, ao acaso, uma pulseira de sua pequena caixa de joias. Ela vê, então, que retirou uma pulseira de prata”. O espaço amostral é igual a pulseiras de prata.

    Número de casos possíveis = 12 pulseiras.

    Determinar o evento: Número de casos favoráveis, ou seja, pulseiras que ganhou de João.

    Número de casos favoráveis = pulseiras de prata que ganhou de João = 4 pulseiras.

    P(p) = número de casos favoráveis = 4 = número de casos possíveis 12 = 4/12 = 1/3.

    Questão comentada pelo Prof. Josimar Padilha 

  • Ao todo Maria tem 12 pulseiras de prata, sendo 4 dadas por João e 8 por Pedro. Se sabemos que a pulseira escolhida é de prata, então o total de possibilidades de escolha dessa pulseira é:

    Total = 12

           Destas 12, apenas 4 nos são “favoráveis”, pois foram dadas por João (atendendo a condição do enunciado). Assim, a probabilidade de Maria ter escolhido uma pulseira de prata dada por João é:

    Probabilidade = favoráveis / total = 4 / 12 = 1/3

    Resposta: A

  • SER DE JOÃO 4..

    SABENDO QUE É DE PRATA 12...

    P(J/P)/P

    P(4/12)/12 (SER DE JOÃO/SABENDO Q É DE PRATA)/ SER PRATA

    = P(4/12).. 1/3

  • Algo é certo : a Maria é danadinha...

  • Probabilidade condicional --> altera o espaço amostral

    É a nata!

  • Com probabilidade condicional fica mais fácil enxergar: qual a probabilidade da pulseira de prata ser de João, dado que a pulseira seja de prata?

    A = (pulseiras de prata de João)

    B = (pulseiras de prata)

    P(A|B) = P(A∩B)/P(B)

    P(A|B) = 4/12 = 1/3

  • Ela vê que a pulseira é de prata após retirara-lá da caixa!!! O enunciado não cita condição nenhuma para o evento. Então temos qie levar em conta o momento da ação. Não afianta querer defender a banca! Eles tb são passíveis de erro!!!

ID
166102
Banca
ESAF
Órgão
MPU
Ano
2004
Provas
Disciplina
Raciocínio Lógico
Assuntos

Marcelo Augusto tem cinco filhos: Primus, Secundus, Tertius, Quartus e Quintus. Ele sorteará, entre seus cinco filhos, três entradas para a peça Júlio César, de Sheakespeare. A probabilidade de que Primus e Secundus, ambos, estejam entre os sorteados, ou que Tertius e Quintus, ambos, estejam entre os sorteados, ou que sejam sorteados Secundus, Tertius e Quartus, é igual a

Alternativas
Comentários
  • Primus =P
    Secundus = S,
    Tertius = T e assim por diante.

    1º caso
    P e S sorteados. Falta mais um. Qtos podem ser? 3! Então P e S podem formar 3 grupos diferentes de sorteados: P,S e T ou P,S e Qa ou P, S e Qi.

    2ª caso
    T e Qi, pelo menos raciocínio de P e S, sabemos que T e Qi podem formar 3 grupos diferentes.

    3º caso
    S, T e Qa. Sendo apenas 3 sorteados, não resta mais ingressos, então S, T e Qa formam apenas 1 grupo.

    Somando as possibilidades 3 + 3 + 1, nas condições do enunciado, poderemos formar 7 grupos.

    Qtas grupos podemos formar, sem impor nenhuma condição de sorteio? Resp.: Só fazer combinação de 5 em 3.
    C(5, 3) = 10.
    São 10 casos possíveis.

    Nossos favoráveis são 7 (de acordo com o enunciado (quenstão))
    Os possível são 10.
    A probabilidade é 7/10 = 0,7

    Resp.: 0,7

  • P (primus e secundus) = 1/5 x 1/4 x 3/3 x P3 = 6/20

    P (tertius e quintus) = 1/5 x 1/4 x 3/3 x P3 = 6/20 

    P (Secundus, Tertius, Quintus) = 1/5 x 1/4 x 1/3 x P3 = 6/60 = 2/20

    6/20 + 6/20 + 2/20 = 14/20 = 7/10 = 0,7

  • Suzane, o quê é esse P3 que você colocou?Desculpe não entendi...
  • Simplificado!

    (Primus e Secundus) = 3/5 x 2/4 =  6/20
    (Tertius e Quintus) = 3/5 x 2/4 =  6/20
    (Secundus, Tertius, Quintus) = 3/5 x 2/4 x 1/3 =  6/60
    6/20 + 6/20 + 6/60 = 14/20 = 7/10 = 0,7

  • GABARITO: C
    Olá pessoal,
    Note que sortear Primus Secundus Tertius é a mesma coisa que sortear Primus Tertius Secundus, portanto a ordem não importa, portanto, trata-se de combinação.
    Assim, vamos "combinar" os 5 ingressos em grupos de 3: 
    Total de formas de sortear = C 5, 3 = 5!/3!2! = 5*4/2 = 10 - Formas que nos interessam:
    Primus Secundus Tertius
    Primus Secundus Quartus 
    Primus Secundus Quintus
    Tertius Quintus Primus
    Tertius Quintus Secundus 
    Tertius Quintus Quartus 
    Secundus Tertius Quartus
    Portanto a probabilidade é: formas que nos interessam/ total de formas = 7/10 = 0,7
    Espero ter ajudado, bons estudos!!!!
  • Total de formas de sorteio:  5.4.3/3! = 10   (há repetição,devemos elimina-la com 3!)
    P  S  X = 1.1.3 = 3 (grupos)
    T  Qi X = 1.1.3 = 3 (grupos)
    S  T Qa = 1            (grupo)
    P( P S X ou T Q X ou S T Qa ) =  (3 + 3+ 1)/10 = 7/10 = 0,7
                                                                        
  • A possibilidade de escolhas de 3 dentre os 5 eh igual a: C(5;3) = 10 possibilidades

    A probabilidade de Primus e Secundus estarem entre os escolhidos eh a msm de Tertius e Quintus estarem entre os escolhidos, e eh igual a: 1x1x3 = 3 (*jah q dois tem lugar certo, teriamos mais 3 possibilidades de chamar os outros ainda naum escolhidos!!!)

    A última probabilidade de termos Secundus, Tertius e Quartus juntos eh igual a 1. (soh existe um jeito disso acontecer!!!)

    Somando as probabilidades: 3 + 3 + 1 = 7

    P = 7/10 = 0,7
  • O que eu fiz: dividi 3 por quatro que deu 0,75.

    A resposta correta é 0,70 o que me deu um número aproximado.


  • Observe que queremos montar grupos de filhos, onde a ordem de escolha dos mesmos não torna um grupo diferente do outro – logo estamos diante de um caso de combinação.

           O total de grupos de 3 filhos que podemos montar a partir dos 5 filhos é:

    Total = C(5,3) = 5 x 4 x 3 / (3 x 2 x 1) = 10

           Destes grupos, vejamos quantos nos interessam:

    -         grupos com Primus e Secundus: neste caso, falta apenas escolher mais 1 filho entre os 3 filhos restantes. Logo, existem 3 possibilidades.

    -         grupos com Tertius e Quintos: novamente, falta apenas escolher mais 1 filho dentre os 3 restantes, existindo ao todo 3 possibilidades.

    -         grupos contendo Secundus, Tertius e Quartus: neste caso já estão escolhidos os 3 filhos, sendo esta a única possibilidade

    Assim, o número de possibilidades favoráveis é:

    Favoráveis = 3 + 3 + 1 = 7

           Portanto, a probabilidade de escolher um grupo com as características pedidas pelo enunciado é:

    Probabilidade = Favoráveis / Total = 7 / 10 = 0,7

    Resposta: C

  • São CINCO filhos e apenas TRÊS entradas, portanto são 60 possibilidades no total (5*4*3=60).

    Porém, o enunciado quer a probabilidade de que Primus e Secundus, AMBOS, sejam sorteados. Com isso, temos dois irmãos que devem ser sorteados e outros três que independem, e três entradas disponíveis:

    P(P,S)=(2*3*1)*3=18

    OU

    A probabilidade de que Tertius e Quintus, AMBOS, sejam sorteados. Seguindo o mesmo raciocínio, pois nada muda:

    P(T,QN)=(2*3*1)*3=18

    OU

    A probabilidade de que Secundus, Tertius e Quartus sejam sorteados. Agora temos três irmãos para três entradas:

    P(S,T,QT)= 3*2*1=6

    Logo, a probabilidade é a soma, visto que temos o "OU", pelo total de possibilidades:

    P(T)=(18+18+6)/60

    P(T)= 0,7

    Gabarito, C.


ID
166108
Banca
ESAF
Órgão
MPU
Ano
2004
Provas
Disciplina
Raciocínio Lógico
Assuntos

Luís é prisioneiro do temível imperador Ivan. Ivan coloca Luís à frente de três portas e lhe diz: "Atrás de uma destas portas encontra-se uma barra de ouro, atrás de cada uma das outras, um tigre feroz. Eu sei onde cada um deles está. Podes escolher uma porta qualquer. Feita tua escolha, abrirei uma das portas, entre as que não escolheste, atrás da qual sei que se encontra um dos tigres, para que tu mesmo vejas uma das feras. Aí, se quiseres, poderás mudar a tua escolha". Luís, então, escolhe uma porta e o imperador abre uma das portas não-escolhidas por Luís e lhe mostra um tigre. Luís, após ver a fera, e aproveitandose do que dissera o imperador, muda sua escolha e diz: "Temível imperador, não quero mais a porta que escolhi; quero, entre as duas portas que eu não havia escolhido, aquela que não abriste". A probabilidade de que, agora, nessa nova escolha, Luís tenha escolhido a porta que conduz à barra de ouro é igual a

Alternativas
Comentários
  • Resposta:   2/3

    Escolhendo a primeira porta você tem 1/3 de probabilidade de ter escolhido a barra de ouro, resultando em uma probabilidade de 2/3 que a barra de ouro esteja em uma das outras portas.

    Como o imperador mostrou uma das portas não escolhidas com o tigre, Luis não teve dúvida, trocou sua porta com 1/3, pela outra não aberta com 2/3 de chances.

  • Esta questão está mais para interpretação de texto do que lógica, para chegar a resposta correta é preciso atentar para a seguinte fala de Luís: " quero, entre as duas portas que eu não havia escolhido"

  • São dois casos favoráveis e três casos possíveis.
    Assim, se Luís sempre trocar de porta, sua chance de ganhar o tesouro será de 2/3
    resposta: c
    bons estudos!
  • São 3 os casos possíveis:
    - Se Luis escolher primeiro a que tem ouro, Ivan abrirá 1 porta com o tigre e sobrará 1 porta com o tigre.
    - Se Luis escolher primeiro a primeira que tem o tigre, Ivan abrirá 1 porta com o tigre e sobrará 1 porta com ouro.
    - Se Luis escolher primeiro a segunda que tem o tigre, Ivan abrirá 1 porta com o tigre e sobrará 1 porta com ouro.
    Notemos então que a chance de que a porta que sobrou tenha ouro é de 2/3.
     

  • Não concordo com o gabarito!

    Se uma das portas já havia sido aberta, então a probalidade de encontrar a barra de ouro nas outras duas restantes é 1/2 !

    alguém entende o meu raciocínio? rsrsrs...
  • Questão simples: desenhe 3 portas - porta 1 com ouro - porta 2 com tigre - porta 3 com tigre
    - Se Luis escolher inicialmente a porta 1, o imperador abrirá a 2 e por fim Luis ficará com a 3 (dentre as duas que ele não escolheu, aquela que o imperador não abriu) - ou seja se Luis escolhe a porta 3, não tem nenhuma possibilidade dele encontrar ouro.
    - Se luis escolher inicialmente a porta 2, o imperador abrirá a 3 (para mostrar o tigre) e Luis ficará com a porta 1, encontrando a barra de ouro
    - Se Luis escolher inicialmente a porta 3, o imperador abrirá a porta 2 (mostrar o tigre) e Luis ficará com a porta 1 de novo, encontrando a barra de ouro.
    Dessa forma, Luis dentre as 3 possibilidades possíveis, encontra a barra de ouro em 2 delas.
    P = número de eventos favoráveis dividido pelo total de eventos possíveis = 2 por 3.



  • Essa questão de RL é MUITO BOA!
    Funciona da seguinte maneira:
    o imperador SEMPRE, isso mesmo, SEMPRE abrirá uma outra porta após a escolha do Fulano de Tal. Sabe-se que o imperador não é burro, ou seja, ele não abriria a porta em que a barra de ouro está, pois assim, obrigatoriamente Fulano de Tal que não é burro nem nada iria trocar para esta!
    Então vamos colocar duas situações:
    1- Fulano de Tal, rapaz carismático, gente boa, escolhe a porta ERRADA (2/3 de chance de isto acontecer). Então o imperador a fim de mexer com os sentimentos do pobre rapaz abrirá a uma porta. MAS É ÓBVIO que ele não irá abrir a porta em que o ouro está; ele não vai entregar assim tão fácil ! Sendo assim, mesmo sem saber o que há na porta que o imperador não abriu, Fulano de Tal TROCA de porta !
    2- Fulano de Tal, escolhe a porta CERTA (1/3 de chance de isto acontecer) e tudo que foi citado acima, acontece novamente, o imperador então abrirá qualquer uma das duas portas, já que nenhuma das duas contêm a barra de ouro(dado que Fulano já acertou a porta) ele não precisa se preocupar em abrir a porta X ou Y.
    --------
    Considerando que Fulano SEMPRE trocará de porta quando o imperador perguntar, suas chances aumentarão de 1/3 para 2/3.
    Eu queria poder desenhar, pois assim fica mais fácil pra explicar, todavia creio que este comentário ajudará alguém!!
  • Portas:
    A = Tigre
    B = Tigre
    C = Ouro
    >>>>>>>>>>>>>
    CASO 1 - Escolhe a porta (
    A) > imperador abre a porta (B) > Fulano troca para a (C) e ganha o Ouro - 1/3 de sair vivo e rico

    CASO 2 - Escolhe a porta (B) > imperador abre a porta (A) > Fulano troca para a (C) e ganha o Ouro - + 1/3 = 2/3 de sair vivo e rico
    CASO 3 - Escolhe a porta (C) > imperador abre a porta (A ou B) > Fulano troca para a (A ou B) e morre
    Sendo assim,
    conclui-se que escolher a porta errada na primeira escolha = viver, como temos 2 portas erradas em 3 = 2/3
  • Pra que esses comentários gigantescos? É simples e o pessoal dificulta...
    São 3 portas.
    A chance de abrir a porta de ouro é de 1/3 obviamente.
    Como o imperador já mostrou uma das portas, resta para o fulano escolher entre as outras duas portas, ou seja, 2/3.

    Pronto!
  • Concordo com a Rosyani que a probabiliadde deveria ser 1/2.
    Havia 3 portas, 1 com ouro e duas com tigres. Ora, depois de aberta uma das portas e ter verificado que ali estava um tigre, qual a certeza que temos quanto às outras duas?  Que uma está com tigre e a outra com ouro. Ora, se em minha nova escolha eu tenho duas portas para escolher e apenas uma está com o ouro, a probabilidade é 1/2. É como pensar em uma moeda com faces cara e coroa, qual a probabilidade de tirar cara? 1/2.
  • CONCORDO PLENAMENTE COM OS DOIS COLEGAS QUE DISSERAM QUE A PROBABILIDADE É 1/2.O ENUNCIADO DIZ: A probabilidade de que, agora, nessa nova escolha...  LUÍS SERIA UM PERFEITO IDIOTA SE ESCOLHESSE A PORTA JA ABERTA (ESTA PORTA SAI DO CAMPO UNIVERSO). RESTAM-LHE DUAS PORTAS, UMA TEM O OURO A OUTRA NÃO= 1/2.
  • Esse é o tipo da pegadinha babaca que acerta quem erra a questão. É óbvio que se você já tem uma porta aberta e tem o tigre, nas outras duas ou tem um ouro ou tem outro tigre. A probabilidade de ele ficar com o ouro é 1/2 se ficar na mesma porta que escolheu, e de 1/2 se escolher a outra porta. A pessoa que fez a prova falou no enunciado onde estava um dos tigres, mas na elaboração da resposta considerou que o prisioneiro poderia escolher como opção a porta que ele já sabia que tinha o tigre. Não sei quem é o mais burro, se é o cara que fez a questão ou se é o prisioneiro que escolheria a porta que ele já sabe que tem um dos tigres.

  • muito fácil!!!!  qualquer uma é 2/3

  • A resposta 1/2 é pegadinha! Procurem por Monty Hall.

     

    Inicialmente...

    PORTA 1                    PORTA 2                  PORTA 3 

       ?                                    ?                                 ?      

     1/3 ouro                         1/3 ouro                     1/3 ouro   -------> Porta 2 + Porta 3 = 2/3 ouro

     

     

    Vamos supor que Luis escolheu a porta 1 e que o imperador abriu a 3.

     

    PORTA 1 - Luis escolheu           PORTA 2                  PORTA 3 - imperador abriu

           ?                                                 ?                             com tigre!  

      1/3 ouro                                        2/3 ouro                       0/3 ouro   -------> Porta 2 + Porta 3 = 2/3 ouro. Tem que dar 3/3 no total!

     

    Luis trocou de porta

     

    Pensando na situação inicial...

    Se Luis escolheu a porta com ouro, se trocar ---> vai perder!

    Se Luis escolheu uma porta com tigre, se trocar ---> vai ganhar

    A chance de Luis escolher uma porta com tigre inicialmente é maior (2/3), por isso é vantajoso trocar.

     A probabilidade de escolher o ouro tem muito a ver com a probabilidade de ele ter escolhido uma porta com tigre inicialmente, não é independente

     

     

    Qual a probabilidade de Luis ''ganhar'' (achar o ouro) ao trocar de porta?

    2/3, porque é a mesma probabilidade de ele ter escolhido a errada no início. Se ele escolheu a errada, ao trocar de porta ---> vai escolher a certa. (letra C)

  • De acordo com o Prof Guilherme Neves do Ponto, temos a seguinte explicação.

    Essa é uma questão plagiada. O problema original é comumente denominado “Problema de Monty Hall” ou “Problema dos Bodes” que surgiu a partir de um concurso televisivo dos Estados Unidos da América chamado Let’s Make a Deal, exibido na década de 1970.

    Eis o problema original:

    Em um programa de auditório, o convidado deve escolher uma dentre três portas. Atrás de uma das portas há um carro e atrás de cada uma das outras duas há um bode. O convidado ganhará o que estiver atrás da porta; devemos supor neste problema que o convidado prefere ganhar o carro. O procedimento para escolha da porta é o seguinte: o convidado escolhe inicialmente, em caráter provisório, uma das três portas. O apresentador do programa, que sabe o que há atrás de cada porta, abre neste momento uma das outras duas portas, sempre revelando um dos dois bodes. O convidado agora tem a opção de ficar com a primeira porta que ele escolheu ou trocar pela outra porta fechada. Suponha que o candidato escolha por trocar pela outra porta fechada. A probabilidade de que, agora, nessa nova escolha, o convidado tenha escolhido a porta que conduz ao carro é igual a quanto?

    A tentação é responder o seguinte: que importância tem mudar ou não, uma vez que restam somente duas portas? Sabe-se que por trás de uma das duas está o automóvel, e que a probabilidade de que ele esteja atrás de uma ou de outra é a metade. Será que isso é verdade?

    Estas são as três possíveis configurações:

    Porta 1________Porta 2_______Porta 3

    Posição 1 _________Carro_________Bode_________Bode

    Posição 2_________Bode_________ Carro__________Bode

    Posição 3_________Bode _______ Bode___________ Carro

    Suponhamos que temos a posição 1.

    Possibilidade 1: O candidato escolhe a porta 1. O apresentador abre a porta 2 ou a porta 3. O apresentador tem liberdade neste caso.

    Se o candidato trocar, PERDE.

    Se não trocar, GANHA.

    Possibilidade 2: O candidato escolhe a porta 2. O apresentador só pode abrir a porta 3.

    Se o candidato trocar, GANHA.

    Se o candidato não trocar, PERDE.

    Possibilidade 3: O candidato escolhe a porta 3. O apresentador só pode abrir a porta 2.

    Se o candidato trocar, GANHA.

    Se o candidato não trocar, PERDE.

    Em resumo, o candidato GANHA em duas das vezes se trocar e só GANHA uma vez se não o fizer. Ou seja, GANHA o dobro das vezes se trocarem.

    A resposta do problema é 2/3. Letra C.

     

     

     

  • Comentário do amigo Igor Gomes foi o melhor!!!!!!!!!11 Valeu

  • Pra quem como eu não entendeu nenhuma explicação sobre essa questão, segue a única que eu consegui entender: https://atitudereflexiva.wordpress.com/2016/05/04/o-problema-de-monty-hall/

    Se considermos que cada porta apresenta uma probalidade de 1/3 de conter o ouro, quando uma porta sai da jogada, ficamos com 1/3 + 1/3 = 2/3, ou seja, as chances de acertar que antes eram de  1/3 agora amentam e passam a ser de  2/3.

  • Lembra-se do quadro “Porta dos Desesperados”, do programa do Sérgio Mallandro? Ou eu estou velho demais? O problema aqui é exatamente o mesmo. 

    -         se na 1ª escolha Luis tiver selecionado a porta premiada, ao trocar (após Ivan revelar um dos tigres) ele certamente irá para a porta com o outro tigre.

    -         se na 1ª escolha Luis tiver selecionado uma porta com um tigre, ao trocar (após Ivan revelar o outro tigre) ele certamente irá para a porta premiada.

    Portanto, o que realmente interessa analisar é a 1ª escolha de Luis, pois é ela quem vai determinar o seu futuro. A chance de ele ter escolhido uma porta com tigre na 1ª escolha é de 2 em 3 disponíveis, ou seja, 2/3. Como vimos acima, feito isso ele certamente irá ganhar o prêmio, pois após Ivan revelar a localização do outro tigre, Luis chegará à porta premiada.

    Resposta: C

  • A melhor explicação é a do Eduardo Barbosa Lemes, de onde podemos concluir que a resposta está ligada ao fato dele ter trocado sua opção.

    Troca de opção: resposta 2/3

    Sem troca de opção: resposta 1/3

    Pra resposta ser 1/2 Só se a questão fosse qual a probabilidade de recolher ouro se tivesse duas portas e uma delas tivesse ouro e a outra um tigre. Na primeira vez que fiz marquei 1/2 e caí na pegadinha.

  • Só acertei essa questão por causa do filme "Quebrando a Banca".


ID
169795
Banca
ESAF
Órgão
MPOG
Ano
2008
Provas
Disciplina
Raciocínio Lógico
Assuntos

Uma urna contém 5 bolas pretas, 3 brancas e 2 vermelhas. Retirando-se, aleatoriamente, três bolas sem reposição, a probabilidade de se obter todas da mesma cor é igual a:

Alternativas
Comentários
  • Bolas pretas : 5/10 x 4/9 x 3/8 = 1/12

    Bolas brancas : 3/10 x 2/9 x 2/8 = 1/120

    Pp+Pb = 1/12 + 1/120 = 11/120

    Resp.: c

  •  Bolas pretas : 5/10 x 4/9 x 3/8 = 1/12

    Bolas brancas : 3/10 x 2/9 x 2/8 = 1/120

    Pp+Pb = 1/12 + 1/120 = 11/120

    Resp.: c

     

    Por que não seria 1/8????

  • 5 bolas pretas 3 bolas brancas 2 bolas vermelhas total = 10 bolas se queremos 3 bolas da mesma cor, sem reposição, elas só podem ser pretas ou brancas, já que a quantidade total de vermelhas é 2. Portanto temos: Probabilidade de tirarmos 3 bolas pretas: P(pretas) = (5/10)x(4/9)x(3/8) = 60/720 Probabilidade de tirarmos 3 bolas brancas: P(brancas) = (3/10)x(2/9)x(1/8) = 6/720 Probabilidade de tirarmos 3 bolas pretas ou tês bolas brancas: P(Total) = P(pretas) + P(brancas) = 60/720 + 6/720 = 66/720 = 11/120 Resposta letra "c" bons estudos!
  • Letra C.

     

    Resolução da questão no vídeo:

     

    https://www.youtube.com/watch?v=wrVY9ikxlv4

     

    Minuto 47:45

  • Resolvo essa questão aqui nesse vídeo

    https://youtu.be/clj38vAq82E

    Ou procure por "Professor em Casa - Felipe Cardoso" no YouTube =D


ID
185119
Banca
CESGRANRIO
Órgão
Petrobras
Ano
2010
Provas
Disciplina
Raciocínio Lógico
Assuntos

Foi observado que uma loja de departamentos recebe, por hora, cerca de 250 clientes. Destes,

. 120 se dirigem ao setor de vestuário;
. 90, ao setor de cosméticos;
. 80, ao setor de cinevídeo;
. 50, se dirigem aos setores de vestuário e de cosméticos;
. 30, aos setores de cosméticos e de cinevídeo e
. 30, aos setores de vestuário e cinevídeo.

Observou-se, ainda, que 50 clientes se dirigem a outros setores que não vestuário ou cosméticos ou cinevídeo. Qual a probabilidade de um cliente entrar nessa loja de departamentos e se dirigir aos setores de vestuário, de cosméticos e de cinevídeo ?

Alternativas
Comentários
  • Iremos considerar apenas 200 clientes posto que "50 clientes se dirigem a outros setores que não vestuário ou cosméticos ou cinevídeo."

    Também, iremos considerar o nº de pessoas que visitam os três setores: vestuário, cosméticos e cinevídeo. A partir daqui, chamaremos este grupo de "x"
    . Precisamos achar o número de pessoas que visitam estes três setores.

    200 = 120 + 90 + 80 - 50 - 30 - 30 + x (somaremos a quantidade de visitas aos três setores e subtrairemos setores em conjunto)
    200 = 180 + x
    x = 20.

    A probabilidade (P) = 20/250 = 0,08.

    ATENÇÂO!

    Note que foi dividida a quantidade de clientes que passam pelos 3 setores, pelo número total de clientes que visitam a loja, tendo em vista que é este o conjunto universo e não 200, porque a pergunta não se restringe aos que visitam apenas aos setores mencionados.
  • Oi Leonardo! Gostei da forma como vc resolveu a questão. Eu fiz de uma forma mais visual, separando os conjuntos dentro de circulos e encontrando a intersecção, mas do seu jeito é mais rápido! Muito bom!
  • Não entendi pq vc somou o x...
    Na verdade, teria que subtrair, não? Já que nos 120, 80 e 90 já estão incluídos x.
  • olá juliana é devido a fórmula: n(AuBuC)= N(A)+N(B)+N(C) - N(AnB) -N(AnC) - N(BnC) + N(AnBnC)
    leia-se n como intersecção
    espero ter ajudado blz
  • A banca quer saber qual é a probabilidade de um cliente entrar nessa loja de departamentos e se dirigir aos setores de vestuário, de cosméticos e de cinevídeo. Assim, segue a análise: 

    • 50 clientes se dirigem a outros setores que não vestuário ou cosméticos ou cinevídeo, ou seja, esses 50 clientes não participam de nenhum dos três conjuntos; 

    • Chamaremos de X a interseção entre ambos os conjuntos, assim temos que:

    • Pessoas que se dirigem somente aos setores de vestuário e de cosméticos = 50 – X

    • Pessoas que se dirigem somente aos setores de cosméticos e de cinevídeo = 30 – X

    • Pessoas que se dirigem somente aos setores de vestuário e cinevídeo = 30 – X 

    Com isso, para descobrir outras partes do conjunto, temos:

    Para o conjunto A: VESTIÁRIO 

    120 – (80 – X)

    120 – 80 + X

    40 + X

    Para o conjunto B: COSMÉTICO

    90 – (80 – X)

    90 – 80 + X

    10 + X

    Para o conjunto C: CINEVÍDEO

    80 – (60 – X)

    80 – 60 + X

    20 + X

    Assim, para encontrar o valor de X, basta somar todos os conjuntos e igualar a 200. Veja: 

    40 + X + 50 - X + 10 + X + X + 30 – X + 30 – X + 20 + X = 200

    40 + 50 + 10 + 30 + 30 + 20 + X = 200

    40 + 50 + 10 + 30 + 30 + 20 + X = 200

    180 + X = 200

    X = 200 – 180 

    X = 20

    Como o examinador quer saber a probabilidade de um cliente entrar nessa loja de departamentos e se dirigir aos setores de vestuário, de cosméticos e de cinevídeo, basta fazer o seguinte cálculo: 

    P= 20/250=0,08

    Resposta: A


ID
191350
Banca
CESPE / CEBRASPE
Órgão
Caixa
Ano
2010
Provas
Disciplina
Raciocínio Lógico
Assuntos

Em uma pesquisa de opinião, foram entrevistados 2.400 eleitores de determinado estado da Federação, acerca dos candidatos A, ao Senado Federal, e B, à Câmara dos Deputados, nas próximas eleições. Das pessoas entrevistadas, 800 votariam no candidato A e não votariam em B, 600 votariam em B e não votariam em A e 600 não votariam em nenhum desses dois candidatos.

Com base nessa pesquisa, a probabilidade de um eleitor desse estado, escolhido ao acaso,

Alternativas
Comentários
  • Número de eleitores entrevistados: 2400 

    Número que vota no canditado A: 800

    Número que vota no canditado B: 600

    Número que não vota em ninguém: 600

    800 + 600 + 600 = 2000

    Logo, faltam 400 eleitores que votam em A ou B. (os de cima só votavam em um ou nenhum)

    800 + 600 + 400 = 1800

    1800 / 2400 = 0,75

  •  Discordo do raciocínio do comentário anterior, apesar de o resultado ser o mesmo. Vejamos:

     

    Grupo 1: Votar APENAS em A = 800

    Grupo 2: Votar APENAS em B = 600

    Grupo 3: Votar NEM em A Nem em B = 600

    Grupo 4:  Votar nos DOIS = 400 (este último chega-se por dedução, pois se não vota somente em A, ou somente em B, ou não vota em nenhum dos dois, só pode fazer parte do grupo dos que votam nos DOIS, A E B simultaneamente)

     

    Letra A)  Errada => Votar em apenas UM desses dois candidatos = 800 + 600 = 1400, 1400/2400 = 0,58 e não 0,5;

    Letra B) Errada => Não votar no candidato A = 600 + 600 (grupo 2 e 3) = 1/2

    Letra C) CORRETA => Votar em A OU B = 800 + 600 + 400 (somente se exclui o grupo 3 que não vota em ninguém) 1800/2400 = 0,75

    Letra D) Errada => Votar em A E B = 400/2400 = 0,16 e não 0,2

    Letra E) Errada => Votar em B e não em A = 600/2400 = 1/4 e não 1/3.

     

    Abraços e boa sorte

     

  • CONCORDO COM O RACIOCÍNIO E NÃO COM O COMENTÁRIO DO DANIEL,POIS EXISTE UM GRUPO DE 400 PESSOAS QUE VOTAM NOS DOIS, QUE PODEM VOTAR EM BRANCO OU ANULAR O VOTO. NÃO DÁ PRA DEDUZIR SE A QUESTÃO NAO FALOU. TAMBÉM CONCORDO COM OS CÁLCULOS ABAIXO APENAS NA QUESTÃO D, DISCORDO DO CÁLCULO POIS VOTAR NOS CANDIDATOS A E B SERIA: 800=600=1400/2400= 0,58%

  • Complicado!!! Em momento algum a banca falou que os 400 restantes votaram alguém, tenho que deduzir que eles votaram nos dois candidatos????
  •   CANDIDATO A CANDIDATO B EM NENHUM DELES NOS DOIS
    Que votam no 800 600 600 400

    Letra C: 800 Candidato A + 600 Candidato B+ 400 que votam nos Dois= 1800

    1800/2400= 0,75
  • Pessoal o item C não poderia ser interpretado de outra forma? Não significa que OU se vota em A OU em B? Mas não nos 2, ou 1 ou outro?! Alguém dá essa luz?
  • ? ?????? ?? ????????  também entendi dessa forma!!! Por que 400 representa A e B e não A ou B. Se alguém puder ajudar...
  • eu também pensei assim!!!  eu achei A ou B não incluia a intersecção A e B. Pelo visto pensei errado...
  • Porque é a regra básica dos conjuntos,

    quando se diz o que está em "A ou B", significa tudo que está em A, tudo que está em B e tudo que está na intersecção.

    quando se diz o que está em "A e B", significa tudo que está somente na intersecção.

    e quando se diz o que está "ou em A ou em B", ai sim significa ou o que está em um, ou o que está em outro, tirando-se a intersecção.
  • É exatamente como um colega mais acima comentou. Chegamos por dedução que os 400 q sobraram, só podem ter votado nos dois, A e B. Pois, se tivessem votado em branco, ou nulo ou deixado de votar, esses 400 estariam no grupo dos que não votaram em nenhum dos dois!
  • Análise das alternativas:
    a)      Votar em apenas um desses dois candidatos será igual a 0,50.
    A + B = 800 + 600 = 1400
    1400/2400 = 7/12 = 0,58
    b)      Não vota no candidato A será igual a  1/3.   
    B + nenhum = 600 + 600
    1200/2400 = 1/2 = 0,50
    c)       Votar no candidato A ou no B será igual a 0,75.
    Probabilidade do candidato A:
    800/2400 = 1/2  = 0,5
    Probabilidade do candidato B:
    600/2400 = 1/4 = 0,25
    Somando temos = 0,75
    d)      Votar nos candidatos A e B será igual a 0,2.
    400/2400 = 1/6 = 0,16
    e)      Votar no candidato B e não votar no candidato A será igual a 1/3.
    Calculo letra B
     
    Letra C correta!!
  • Kelly Costa, permita-me comentar que no seu comentário da letra 'C" seria corrigido para :

    c) Votar no candidato A ou no B será igual a 0,75.

    Probabilidade do candidato A:

    800 + 400/2400 = 1/2 = 0,5

    Probabilidade do candidato B:

    600/2400 = 1/4 = 0,25

    Somando temos = 0,75   mas foi um ótimo comentário seu.......


     

  • Kelly, na passagem da letra c que você descreveu não seria 800/2400=1/3 ?

  • Galera,não entendi porque a "c" está correta, pois ao meu ver aplica-se a propriedade das probabilidades que diz: P(A OU B)=  P(A)  + P(B) - P(A interseção B). Quem puder ajudar será de GRANDE!!! valia. 

  • V = 1800 (quantidade de votantes: 2400 - 600 não-votantes = 1800)    

    P(V) = n(V) / n(S)

    P(V) = 1800 / 2400 = 0,75


    P(V) = probabilidade de votar em A ou B

    n(V) = quantidade de votantes

    n(S) = número de elementos do espaço amostral (total de pessoas)


  • Votar no candidato A ou B não é votar no candidato A e B, letra C correta?


  • LETRA C

  • Gabarito: C.

    Nós temos 2400 eleitores, dos quais:

    • 800 votam em A e não votam em B;
    • 600 votam B e não votam A;
    • 600 não votam em nenhum dos dois.

    Conclusões com base nos valores:

    Se 600 não votam em nenhum dos dois candidatos, significa que o total que vota em A e B vale 2400 - 600 = 1800 eleitores.

    Desses 1800 nós sabemos que 800 votam e A e não votam em B, significa, então, que 800 votam APENAS em A. Note que como eles não votam em B, eles não entram na interseção.

    De maneira igual, dos 1800 600 votam em B e não votam em A, ou seja, 600 votam apenas em B.

    Como 800 votam apenas em A e 600 votam apenas em B, nós temos 1400. Falta o valor da interseção para fechar os 1800, que é 400.

    Agora que achamos os valores dos conjuntos, vamos aos itens:

    a) Errado. É maior que 0,5.

    Pode votar apenas em A OU apenas em B = 800/2400 + 600/2400 = 1400/2400 = 0,583 = 58,3%.

    b) Errado. É 0,5.

    Não votar no candidato A é o mesmo que votar apenas no candidato B ou não votar em nenhum.

    P = 600/2400 + 600/2400 = 1200/2400 = 1/2 = 0,5 = 50%.

    c) Certo.

    Como 600 não votam, significa que 1800 votam em A ou em B.

    P = 1800/2400 = 18/24 = 3/4 = 0,75 = 75%.

    d) Errado. Vale 1/6.

    Votar em A e B é a interseção, que tem 400 eleitores.

    P = 400/2400 = 4/24 = 1/6.

    e) Errado. Vale 1/4.

    Votar no B e não votar no A é o mesmo que votar apenas em B.

    P = 600/2400 = 1/4.

    Qualquer equívoco, mandem mensagem.

    Bons estudos!

  • Na realidade 400 eleitores votam em A e B (intersecção)

    A ou B representaria a união do conjunto de todos que votam em A e em B.

    ou seja:

    Só A + Só B + A intersecção B = A União B

    800 + 600 + 400 = 1800

    1800/2400 = 0,75


ID
204394
Banca
FEC
Órgão
MPA
Ano
2010
Provas
Disciplina
Raciocínio Lógico
Assuntos

Se anotarmos em pedaços de papel todos os anagramas que podem ser obtidos a partir da palavra BRASIL, escrevendo um anagrama em cada pedaço de papel, podemos dizer que a probabilidade de sortearmos um desses papéis e sair um anagrama começado por uma vogal, é de, aproximadamente:

Alternativas
Comentários
  •  

           Muito simples, existem duas vogais na BRASIL dentre as seis que compoem a mesma palavra, então basta pegar as duas vogais e dividir pelo numero total de letras que compoem essa palavra, ou seja, 2 dividido por 6 = 1/3 = 33,3%

  • Primeiro conta-se as possibilidades de anagramas que é 6! = 720

    depois acha-se a possibilidade de Anagramas começados com a vogal A (ou seja em 6 retira-se 1) da 5! = 120

    depois acha-se a possibilidade de Anagramas começados com a vogal I (ou seja em 6 retira-se 1) da 5! = 120

    Somando dará 240 possibilidades de começar com uma vogal de um total de 720 anagramas possiveis:

    chega-se ao resultado de 33,3 % de chances

  • COMENTÁRIOS DO PROF. RONILTON LOYOLA
    Vamos, primeiramente, determinar o número de anagramas da palavra BRASIL. Esse é o número de casos possíveis. Fazendo a permutação das 6 letras da palavra, vem:  n(S) = 6! =720.
    Agora vamos determinar o número de anagramas que começam por vogal:
    - começam pela vogal A: 5! = 120;
    - começam pela vogal I: 5! = 120.
    Então, o número de anagramas que começam por vogal, isto é, o número de casos favoráveis é dado por n(E) = 120 + 120 = 240.
    Logo, a probabilidade procurada é:  p(E) = n(E)/n(S) = 240/720 = 1/3 = 33,3% (aproximadamente)
    Gabarito: letra B.
  • 6 letras formam 100%. quero saber quantos % formam 2 letras:

    6=100%
    2=X%

    6X= 200
    X= 33,3%

    "Rapidin" e "bora" p proxima.
  • Brasil - 6 letras
    6x5x4x3x2x1 = 720

    Agora, Brasil, começando por vogal. Temos duas vogais em "Brasil".
    2x5x4x3x2x1 = 240 (o 2 no começo significa as vogais, logo, sobram 5 letras para permutarmos)

    240/720 = 0,33 ou 33,3%

    Abraços!
  • 2 vogais do total de 6 letras, logo será 2/6 = 1/3= 0.33


ID
217138
Banca
CESGRANRIO
Órgão
IBGE
Ano
2009
Provas
Disciplina
Raciocínio Lógico
Assuntos

Uma urna contém 4 bolas brancas e 6 bolas pretas. Para que, nessa urna, as bolas brancas passem a representar 50% do total de bolas, é suficiente

Alternativas
Comentários
  • 4 bolas brancas

    6 bolas pretas

    para ficar 50% pra cada só adicionar 2 bolas brancas

     

    Resposta Letra B

    Bons Estudos Pessoal !!

    Paulo.

  • Para resolver:

    1) Inicialmente temos 4 bolas brancas e 6 pretas;
    2) Acrescento 2 bolas brancas, portanto ficamos com (2 bolas brancas acrescidas) + 4 (bolas brancas já existentes) + 6 (bolas pretas já existentes);
    3) Somando todas as bolas chego ao total geral de 12 bolas (todas as pretas + todas as brancas).
    4) O total de bolas representa 100 %, logo tenho que saber quanto em termos de percentagem representa cada bola, para isso basta pegar 100/12 = 8,3333333...
    5) Pego o resultado e multiplico pela quantidade de bolas brancas, ou seja, 8,333... x 6 (bolas brancas) = 50
    6) Logo com o aumento de duas bolas é possível alcançar o resultado de 50% de bolas brancas;

    Ou simplesmente pensar que o total das bolas são 12, se eu dividir dá exatamente a metade 6, sendo que a metade é 50% !

    Espero ter ajudado!
  • a banca queria nos testar.... srrsrs
     

    que venha mais desse grau de dificuldade pra nos !!!
  • Incialmente temos: 4B (40%) e 6P (60%) = 10 bolas

    Para que as bolas brancas representem 50% deve ter a mesma quantidade de bolas pretas.

    Acrescentando mais duas bolas brancas ficaremos com 6B e 6P.
  • Incialmente temos: 4B (40%) e 6P (60%) = 10 bolas

    Para que as bolas brancas representem 50% deve ter a mesma quantidade de bolas pretas.

    Acrescentando mais duas bolas brancas ficaremos com 6B e 6P.
  • Isso é questão pra ver se o cara é retardado é?

  • Com certeza Jorge Barreto! Perdi meu tempo lendo isto.

  • meu Deus uma dessa não cai na minha prova kkkk

  • deu ate medo marcar, sera q é so isso mesmo? rsrs

  • será que alguem consegue errar uma dessa?

  • aquela questão que de tão fácil tenho medo de responder...rs


ID
230221
Banca
FUNCAB
Órgão
DETRAN-PE
Ano
2010
Provas
Disciplina
Raciocínio Lógico
Assuntos

No pátio do DETRAN um inspetor ficou encarregado de vistoriar 10 carros, sendo 5 carros de cor prata, 3 carros de cor preta e 2 carros de cor vermelha. Para iniciar as inspeções ele sorteou aleatoriamente, de uma única vez, dois desses carros.A probabilidade de que pelo menos um dos dois carros sorteados seja de cor preta é igual a:

Alternativas
Comentários
  • Verifica-se que se trata de eventos complementares ao identificarmos que o oposto de sair pelo menos um carro preto no sorteio é não sair carro preto. Sendo assim:

    P(não ser preto) = 7/10 . 6/9 = 42/90 = 7/15.

    Como o que se deseja na questão é que pelo menos um dos carros sorteados seja preto, então:

    P(pelo menos um preto) = 1 - 7/15 = 8/15.

    Resp.: C.
  • Resolução pela Probabilidade Complementar.

    S= 10 carros: {5 prata, 3 pretos, 2 vermelhos}

    Sorteio de 2 carros. Qual a probabilidade de ter pelo menos, ou seja, no mínimo, 01 carro preto?

    I: preto prata
    II preto vermelho
    III: preto preto

    P(Ã): Probabilidade de não ocorrer carro preto: 5 + 2 = 7 (Evento) OBS: sem reposição

    1º evento: 7/10        2º evento: 6/9

    7/10 * 6/9 = 42/90 = 7/15

    P(A) + P(Ã) = 1
    P(A) + 7/15 = 1
    P(A) = 1 - 7/15
    P(A) = 8/15

  • Existem C(10;2) = 45 maneiras de escolher dois carros. Desse total, há C(7;2) = 21 maneiras de escolher de forma que nenhum dos dois seja preto. Portanto, há 45 - 21 = 24 formas de escolher de modo que pelo menos um seja preto. Portanto, a probabilidade pedida é 24/45 = 8/15.

    Resposta: c.

    Opus Pi.

  • Resolvi da seguinte maneira:

    2/10 + 1/3 = (6+10)/30 = 16/30 = 8/15

    Mas ainda considero mais confiáveis os raciocínios demonstrados nos outros comentários. 

  • e pq se eu fizer a primeira probabilidade de ocorrer que seria 3/10 x a segunda que é 2/9 não dá o mesmo resultado?


  • Manuelle, a questão pede a probabilidade de ocorrer "PELO MENOS UM carro preto" no sorteio, não a probabilidade de ambos os carros serem pretos.

  • Eu sofro com a matemática básica, por gentileza, me tirem a seguinte dúvida.

    Na parte P(A) = 1-7/15 = 8/15. Eu não estou entendendo o motivo de 1-7 ser 8.

  • Hudson, na verdade é 1 -  7  =>    15  -  7  =  8 

                                             15         15     15    15


  • 01 Hudson, também não estava entendendo, mas é o seguinte.

    total 15/15   - 7/15 = 8/15.  

     

  • TOTAL DE CARROS: C(10;2) =  10! / 2! 8! = 45 

    NENHUM CARRO PRETO: C(7;2) = 7! / 2! 5! = 21

     

    Logo, o total de possibilidades de que pelo menos um dos dois carros sorteados seja de cor preta é igual a: 

    45 - 21 = 24

     

    Logo, a probabilidade de que pelo menos um dos dois carros sorteados seja de cor preta é igual a:

    24/45 = 8/15


ID
232243
Banca
FUNIVERSA
Órgão
CEB-DISTRIBUIÇÃO S/A
Ano
2010
Provas
Disciplina
Raciocínio Lógico
Assuntos

O mau funcionamento de uma das máquinas de uma indústria fez com que 10% das peças produzidas em um determinado lote apresentassem defeito. Escolhendo-se aleatoriamente cinco peças desse lote, a probabilidade aproximada de que menos de três delas apresentem esse defeito, se cada peça retirada é reposta antes de se retirar a próxima, é de

Alternativas
Comentários
  • Para resolver esta questão, devemos inicialmente encontrar a probabilidade de encontrarmos 0 defeito, 1 defeito, 2 defeitos, 3 defeitos, 4 defeitos e 5 defeitos.

    Sabe-se que prob defeito = 0,1 e  prob não defeito = 0,9

    p(0) = (0,9*0,9*0,9*0,9*0,9)*1 = 0,59049

    p(1) = (0,9*0,9*0,9*0,9*0,1)*5 = 0,32805

    Obs: Devemos multiplicar no final por 5, pois há 5 possibilidades de termos 1 defeito (primeira peça, segunda peça, terceira peça, quarta peça ou quinta peça).

    Matematicamente é representado por C (5,1) = 5! / (1!*4!) = 5

    p(2) = (0,9*0,9*0,9*0,1*0,1)*10 = 0,0729

    Obs: Devemos multiplicar no final por 10, pois há 10 possibilidades de termos 2 defeitos (primeira e segunda peças, primeira e terceira peças, primeira e quarta peças, primeira e quinta peças, segunda e terceira peças, segunda e quarta peças, segunda e quinta peças, terceira e quarta peças, terceira e quinta peças, quarta e quinta peças)

    Matematicamente é representado por C (5,2) = 5! / (2!*3!) = 10

    p(3) = (0,9*0,9*0,1*0,1*0,1)*10 = 0,0081

    Multiplica- se por 10, pois C (5,3) = 5! / (3!*2!) = 10

    p(4) = (0,9*0,1*0,1*0,1*0,1)*5 = 0,00045

    Multiplica- se por 10, pois C (5,4) = 5! / (4!*1!) = 5

    p(5) = (0,1*0,1*0,1*0,1*0,1)*1 = 0,00001

    p(<3) = p(0) +p(1)+p(2) = 0,59049 + 0,32805 + 0,0729 = 0,99144   ----> Alternativa E

    Uma dica útil para ganhar tempo é calcular somente p(3), p(4) e p(5) e fazer 1 - (p(3)+ p(4) + p(5)) = 1 - (0,0081+0,00045+0,00001) = 1 - 0,0856 = 0,99144.

  • Resolução:
    São menos de três peças, ou seja, duas em cinco.
    Atentar - (cada peça retirada é reposta antes de se retirar a próxima)
    Digamos que em cada 100 peças, 10 apresentem defeito (10%).
    Probabilidade são os casos favoráveis (10)/ sobre os possíveis (100).
    Qual a chance de jogar o primeiro dado e sair uma peça defeituosa?10 possibilidades em 100.
    A peça defeituosa é reposta e q
    ual a chance de jogar o segundo dado e sair uma peça defeituosa? 10 possibilidades em 100.
    Multiplica-se as duas possibilidades: 10/100*10/100 =100/10.000 = 1/100 =0,01 ou 1%
    1% é a chance de 2 peças defeituosas serem escolhidas em 5, ou 99% de chance de eles não serem escolhidas.
    Gente, só um toque! As bancas quando preparam essas questões, elas pensam no tempo de execução do candidato, ou seja, vc tem 3 ou 4 minutos para responder uma questão e não uma hora. Ou vc sabe, ou vc pula para próxima questão. Não adianta ficar fazendo mil e uma contas.
  • Na questão foi informada a porcentagem de peças defeituosas

    DEFEITUOSAS = D = 10%

    Logo,

    PERFEITAS = P= 90%

    "se cada peça retirada é reposta antes de se retirar a próxima" é outro dado a ser levado em consideração, visto que, sendo um evento com retiradas sucessivas e com reposição, se trata de uma probabilidade binominal.

    Além disso, MENOS DE TRÊS PEÇAS PODE SER= 0 PEÇA OU 1 PEÇA OU 2 PEÇAS.

    RESOLUÇÃO: CALCULAR A PROBABILIDADE DE CADA RETIRADA X PERMUTAÇÃO (QUANDO NECESSÁRIO)

    PROBABILIDADE DE 0 PEÇA DEFEITUOSA

    PxPxPxPxP = 9/10 x 9/10 x 9/10 x 9/10 x 9/10 = 59049/100000 (não precisa permutar, afinal só temos as mesmas peças aqui, as perfeitas)

    PROBABILIDADE DE 1 PEÇA DEFEITUOSA

    PxPxPxPxD = 9/10 x 9/10 x 9/10 x 9/10 x 1/10 = 6561/100000

    (multiplica pela permutação de 5 com repetição de 4 (peças perfeitas) = 5) = 32805/100000

    PROBABILIDADE DE 2 PEÇAS DEFEITUOSAS

    PxPxPxDxD = 9/10 x 9/10 x 9/10 x 1/10 x 1/10 = 729/10000

    (multiplica pela permutação de 5 com repetição de 3 e 2 (peças perfeitas e peças defeituosas, respectivamente) = 10) = 7290/10000

    0 PEÇA OU 1 PEÇA OU 2 PEÇAS = PROBABILIDADE DE 0 PEÇA DEFEITUOSA + PROBABILIDADE DE 1 PEÇA DEFEITUOSA + PROBABILIDADE DE 2 PEÇAS DEFEITUOSAS

    59049/10000 + 32805/10000 + 7290/10000 = 99144/100000 = 0,99 = 99%


ID
244474
Banca
UPENET/IAUPE
Órgão
SERES-PE
Ano
2010
Provas
Disciplina
Raciocínio Lógico
Assuntos

Para ter acesso a um dado setor, um visitante precisa passar por 4 verificações independentes de segurança, dispostas uma após a outra em sequência. A probabilidade de um visitante mal intencionado qualquer passar pela primeira verificação é de 50%; de passar pela segunda verificação é de 12%; de passar pela terceira verificação é de 25% e de passar pela quarta verificação é de 15%. Nessas condições, é CORRETO afirmar que

Alternativas
Comentários
  • A probabilidade de o visitante mal intencionado ter acesso a esse setor é de: 50/100 x 12/100 x 25/100 x 15/100 = 0,00225, ou seja, 0,225%.

    a probabilidade de o visitante mal intencionado ter seu acesso negado a esse setor é de: 100% - 0,225% = 99,775%

  • As quatro verificações são independentes: eventos independentes multiplica-se as probabilidades.

    Probabilidade de acesso:

    1º) 50% = 50/100
    2º) 12% = 12/100
    3º 25% = 25/100
    4º 15% = 15/100

    50/100 * 12/100 * 25/100 * 15/100 = 225000/100000000 = 0,00225 = 0,225%

    0,225 é menor que 1%. Alternativa E
  • Cada vez vai ficando menor.

    50/100 x12/100 x25/100 x 15/100 = 225000/100000000 =  0,00225  transformando em pocentagem passa duas casas para direita.

     0,00225 = 0,225%

    Letra E

  • 50 - 12 =38 - 25 =13 - 15 = - 2% ou seja, menor que 1%. Letra E

  • 50%/100 x 12%100 x 25%/100 x 15%/100 = 0,00225 ( isto é, menos de 1%) Gabarito é letra "E".


ID
245149
Banca
CESPE / CEBRASPE
Órgão
TRT - 21ª Região (RN)
Ano
2010
Provas
Disciplina
Raciocínio Lógico
Assuntos

Suponha que determinado partido político pretenda ter candidatos
próprios para os cargos de governador, senador e deputado federal
e que tenha, hoje, 5 possíveis nomes para o cargo de governador,
7 para o cargo de senador e 12 para o cargo de deputado federal.
Como todos os pré-candidatos são muito bons, o partido decidiu que
a escolha da chapa (governador, senador e deputado federal) será
por sorteio. Considerando que todos os nomes têm chances iguais
de serem escolhidos, julgue os itens seguintes.

Caso João e Roberto sejam pré-candidatos ao cargo de senador e Maria e Ana sejam pré-candidatas ao cargo de deputado federal, a chance de que a chapa sorteada tenha qualquer um desses nomes será maior que 49%.

Alternativas
Comentários
  • Calcula-se a chance de pelo menos um entre o João, o Roberto, a Maria e a Ana serem sorteados para disputarem os cargos assinalados pela questão:

    P = (Chance de só o João ser sorteado) + (Chance de só o Roberto ser sorteado) + (Chance de só a Maria ser sorteada) + (Chance de só a Ana ser sorteada)

    P = (1/7 . 10/12) + (1/7 . 10/12) + (5/7 . 1/12) + (5/7 . 1/12)

    P = 2 (1/7 . 10/12) + 2 (5/7 . 1/12)

    P = 30/84

    P ~ 36%

  • Errado. Queremos encontrar a probabilidade de a chapa sorteada tenha qualquer um desses nomes. Então, podemos fazer do seguinte modo:

    Primeiro, determinar a probabilidade de a chapa não tenha um desses nomes;

    Segundo, determinar, com o resultado da situação anterior, o seu complementar ( PROBABILIDADE DO EVENTO COMPLEMENTAR). E logo, encontraremos o que se pede.

     

    P(A) . P(B) = 5/7 . 10/12 = 25/42

    1 - P(A) . P(B) = 1 - 25/42 = 17/42  que é aproximadamente 40 %.

     

    Espero que tenho ajudado. 

  • Caso João e Roberto sejam pré-candidatos ao cargo de senador e Maria e Ana sejam pré-candidatas ao cargo de deputado federal, a chance de que a chapa sorteada tenha qualquer um desses nomes será maior que 49%.

    No momento em que diz que a chapa tenha qualquer um dos nomes isso significa que OU João , OU Roberto, OU Maria, OU Ana serão sorteados!
    Ao mencionar OU quer dizer que somarei as propabilidade de qualquer um dos quatro ser sorteado.

    Por partes:
    probabilidade do João ser sorteado: 1/7
                               do Roberto                 : 1/7  (são 7 candidatos ao cargo de senador)
                               da Maria                     : 1/12
                               da Ana                         : 1/12  (são 12 candidatos ao cargo de deputado federal)

    Somando as probabilidades, já oque se procura é a probabilidade de qualquer um ser sorteado, : 1/7 +1/7+ 1/12 + 1/12 = 0,45
    Então a probabilidade será de 45% e não maior que 49%

    ps: se estivesse pedindo a probabilidade de TODOS  serem sorteados, em vez de somar multiplicarei as probabilidades.

    ERRADA
  • TEMOS  5  GOVERNADORES, 7 SENADORES E 12 DEPUTADOS FEDERAIS.
    QUANTIDADES DE CHAPAS: 5C1 X 7C1 X 12C1 = 420 CHAPAS.

    CHAPAS COM JOÃO OU ROBERTO

    5 X 2 X 12= 120

    CHAPAS COM ANA OU MARIA

    5 X 7 X 2 = 70

    120 + 70 = 190.

    420 ---------- 100%
    190 ----------  X

    X= 190 X 120/420 = X = 45,24%

    PORTANTO, SERÁ MENOR QUE 49%
  • P() de não ter nenhum dos nomes é: 5/7 * 10/12 = 50/84

    Logo, a P() de ter pelo menos um dos nomes é 34/84 = aprox. 40%.

    FONTE: http://www.forumconcurseiros.com/forum/showthread.php?t=283734
  • Gente, o único comentário correto é o da Marília, que é o 3º comentário. A questão é de interpretação, pois a questão é clara ao dizer que deseja a probabilidade de qualquer desses nomes estarem presentes. Isso significa optar pelo OU e não pelo E. Por isso é uma soma de probabilidade e não uma multiplicação.
  • 5 x 2 x 12 = 120

    5 x 7 x 2 = 70

    70+120=190

    5 x 7 x 12 = 420

    190/420 = 19/42 = 45,2 %

  • PRIMEIRO ANTES DA PROBABILIDADE O CAMARADA TEM Q ENTENDER A PARTE DE ANALISE COMBINATORIA !!! 


    OU SEJA QUESTÃO UM TANTO COMPLICADA -- JUNTOU ANALISE E PROBABILIDADE !!!!



    GOV - 5, SEN - 7, DF - 12


    CHAPA SERA DE (GOVXSENXDF)


    JOAO E ROB. ------ 5X2(AQUI SO PODE ESSES )X12=120


    MARIA E ANA-----5X7X2(AQUI SO PODE ESSES )= 190 


    TUDO 5X7X12 = 420


    PROBABILIDADE DO OU (LER A QUESTAO )120/420 + 70/420=190/420 


    MARAVILHA MULTIPLICA POR 100  === 45,2% MENOR 

  • - Comentário do prof. Arthur Lima (ESTRATÉGIA CONCURSOS)

    1) A chance de que a chapa escolhida tenha qualquer destes nomes é igual a 100% menos a chance de que a chapa não tenha nenhum destes nomes. Para não ter nenhum destes nomes, restam 5 pré-candidatos a governador, 5 para senador (excluimos João e Roberto) e 10 para deputado (excluimos Maria e Ana). Existem 5x5x10 = 250 formas de se formar um trio com estas pessoas. Ao todo, haviam 5x7x12 = 420 formas

    2) Portanto:
    Probabilidade (ter qualquer dos nomes) = 100% - Probabilidade (não ter nenhum)
    Probabilidade (ter qualquer dos nomes) = 100% - 250/420 = 40,5%


    Gabarito: ERRADO

  • Cuidado amigo Rogério Figueiredo!

    A estrutura está correta, porém há um erro no final:

    A divisão 250/420 = 0,59 nos dá um resultado errôneo.

    O correto é:

    420 (total) - 250 (total sem as 4 pessoas) = 170

    Agora sim podemos efetuar a divisão:

    170 / 420 = 0,40, chegando aos 40%

     

    Serei PRF!

  • Eu calculei a probabilidade de nenhum deles participarem da chapa e deu certo. Achei mais simples, porém, não sei se é o correto nessa situação.

  • Probabilidade para dois senadores: 2/7 = 0,28

    Probabilidade para dois deputados federais: 2/12 = 0,16

    0.28 + 0.16 = 44%

  • Gabarito: Errado.

    Fiz pelo evento complementar.

    100% - o que eu não quero = o que eu quero.

    O que eu não quero? Que as chapas sejam formadas sem os nomes citados. Então:

    5 possibilidades para governador x 5 possibilidades para senador x 10 possibilidades para deputado federal = 250.

    Qual o total de possibilidades? Total = 5 x 7 x 12 = 420.

    1 - 250/420 = 1 - 0,59 = 0,41 = 41%.

    Portanto, é inferior a 49%.

    Bons estudos!


ID
245158
Banca
CESPE / CEBRASPE
Órgão
TRT - 21ª Região (RN)
Ano
2010
Provas
Disciplina
Raciocínio Lógico
Assuntos

Suponha que determinado partido político pretenda ter candidatos
próprios para os cargos de governador, senador e deputado federal
e que tenha, hoje, 5 possíveis nomes para o cargo de governador,
7 para o cargo de senador e 12 para o cargo de deputado federal.
Como todos os pré-candidatos são muito bons, o partido decidiu que
a escolha da chapa (governador, senador e deputado federal) será
por sorteio. Considerando que todos os nomes têm chances iguais
de serem escolhidos, julgue os itens seguintes.

Considerando que Mariana seja pré-candidata ao cargo de governador e Carlos seja pré-candidato ao cargo de senador, então a probabilidade de que a chapa sorteada ou não tenha o nome de Maria ou não tenha o nome de Carlos será inferior a 0,75.

Alternativas
Comentários
  • A questão pede conhecimentos de disjunção exclusiva ("ou... ou...") e de probabilidade.

    A tabela-verdade da disjunção exclusiva é:

    p | q | ou p ou q

    V | F |    F

    V | F |    V

    F | V |    V

    F | V |    F

    Calcula-se a probabilidade pedida considerando-se:

    P = [ (Prob. de Maria não ser sorteada, para a disputa de Dep. Federal) x (Prob. de Carlos ser sorteado) ] + [ (Prob. de Maria ser sorteada) x (Prob. de Carlos não ser sorteado) ]

    P = (11/12 . 1/7) + (1/12 . 6/7)

    P ~ 20% ~ 0,2

  • Eu pensei dessa forma:

    Se temos 1 candidato, frente aos outros 4, que estão na disputa pelo cargo de candidato ao governo, então a probabilidade de ele não se escolhido é:

    1-1/5 ou 4/5 = 0,8

     

    Se temos 1 candidato, frente aos outros 6, que estão na disputa pelo cargo de candidato ao senado, então a probabilidade de ele não se escolhido é:

    1-1/7 ou 6/7 = 0,8571

    Então, ou Maria não é a escolhida ou Carlos não é o escolhido para compor a chapa, então temos uma conjunção:

    4/5 x 6/7 = 0,8 x 0,8571 = 0,6857 ---> 68,57% de não temos um ou o outro na composição da chapa. 

    Olhando a assertiva, vejo que é certo, pois a probabilidade é menor que 0,75 ou 75%

  • "ou não tenha o nome de Mariana ou não tenha o nome de Carlos"ou uma coisa ou outra...
     Ter Mariana(governador) e não ter Carlos(senador) = 1/5 * 6/7 = 6/35
    Não ter Mariana e ter Carlos = 4/5 * 1/7 = 4/35
    então: 6/35 + 4/35 = 10/35 = aprox. 0,286.

  • (I) Probabilidades de Chapas:

    5 gov . 7 sen . 12 df = 420 chapas.

    (II) Ter Maria e Não ter Carlos:

    1 gov(Maria) . 6 sen (tira carlos) . 12 df = 72 chapas

    (III) Ter Carlos e Não ter Maria:

    4 gov(tira maria) . 1sen(Carlos) . 12 df = 48 chapas

    OU um Ou outro = soma as chapas (72+48=120)

     

    120/420 = 2/7 = 0,2857

    *essa questão foi anulada, apenas pelo motivo de ter o nome de "mariana" e dps se referir a "maria".
  • "ou não tenha o nome de Mariana ou não tenha o nome de Carlos" = ou um ou outro, um tem que ser V o outro F. Dessa forma:

    1. Ter Mariana e não ter Carlos = 1/5 * 6/7 = 6/35

    2. Não ter Mariana e ter Carlos = 4/5 * 1/7 = 4/35

    Há duas P(): 6/35 + 4/35 = 10/35 = aprox. 0,29.

    FONTE: http://www.forumconcurseiros.com/forum/showthread.php?t=283734
  • Enunciado ambíguo:

    ou não tenha o nome de Maria ou não tenha o nome de Carlos, poderia ser reescrito da seguinte forma:

    ~Maria v ~Carlos

    Possibilidade de chapas sem Maria: 4/5

    Possibilidade de chapas sem Carlos: 6/7

    Lembrando da fórmula da união dos conjuntos A e B = P(A)+P(B)-P(A e B)

    Aplicando

    4/5 + 6/7 - (4/5)*(6/7) = 34/35

    Logo, a probabilidade se ter uma chapa ou sem Maria ou sem o Carlos é de 34/35 = 97,1%, portanto enunciado errado.


ID
249277
Banca
CESPE / CEBRASPE
Órgão
DETRAN-ES
Ano
2010
Provas
Disciplina
Raciocínio Lógico
Assuntos

Considere que, em uma amostra composta por 210 pessoas atendidas em unidade de atendimento do DETRAN, 105 foram ao DETRAN para resolver pendências relacionadas à documentação de veículos; 70, para resolver problemas relacionados a multas; e 70, para resolver problemas não relacionados à documentação de veículos ou a multas. A respeito dessa situação hipotética, julgue o item.

Entre as 210 pessoas da amostra, para se selecionar, ao acaso, ao menos duas que tenham procurado a unidade do DETRAN para solucionar pendências relacionadas à documentação de veículos ou ao menos duas que a tenham procurado para resolver problemas relacionados a multas, o menor número de pessoas que devem ser selecionadas será igual a 73.

Alternativas
Comentários
  • Devemos ter o número mínimo de pessoas para garantirmos que tenha pelo menos 2 pessoas para resolver problemas de multas ou 2 pessoas para resolver problemas de habilitação.
    Neste caso, do total de 210 pessoas, não devemos considerar as 70 pessoas que foram resolver outros problemas no Detran.
    Logo, se escolhermos 70 pessoas, no pior caso, podemos ter escolhido estas 70 pessoas com problemas extras ( não relativos a CNH ou multas)
    Se escolhermos 71, temos a certeza de que pelo menos 1 dessas está com problemas de CNH ou multa
    Ao escolhermos 72, podemos ter três situações :
    -> pelo menos 2  pessoas com problemas com CNH ( neste caso é o que a questão exige)
    -> pelo menos 1 pessoa com problema de CNH e outra com problema de multa ( não é o que a questão exige)
    ou ainda
    ->pelo menos 2  pessoas com problemas com multa( neste caso é o que a questão exige)
    Além dos 70 que são as pessoas com problemas diferentes de CNH e multa

    Como a situação do meio "invalida" a primeira e a última, devemos ter mais uma pessoa que com certeza esta será com problema de CNH ou multa, totalizando "pelo menos 2 com problemas de CNH ou 2 com problemas de multas"

    Totalizando -> 73 pessoas


  • Entre as 210 pessoas:
    A: 140 pessoas foram ao DETRAN para resolver pendências relacionadas à documentação de veículos OU à multa.
    B: 70 pessoas foram resolver outros problemas.


    Deve haver no mínimo 3 pessoas do grupo A, entre as pessoas selecionadas, para assegurar que ao menos duas tenham ido ao Detran para resolver a mesma pendência (Há 2 tipos de pendência relacionados ao grupo A).
    Para garantir que haverá ao menos 3 pessoas do grupo A, no mínimo 73 pessoas devem ser selecionadas (no pior caso possível incluirá todas as 70 pessoas do grupo B e 3 do grupo A).

    Portanto, o número mínimo de pessoas a selecionar é 73.
    Gabarito:certo
  • Essa questao nao estaria melhor em conjuntos ou diagramas logicos? Acho que naotem nada de a nalise combinatoria nessa questao...
  • De fato, Leandro! Não se trata de análise combinatória. Essa questão é de probabilidade.

    Saudações vascaínas.

  • nao entendi nada!


  • princípio das casa dos pombos :)

  • Gabarito: CORRETO

    Veja que, se selecionarmos 70 pessoas, pode ser que as 70 façam parte do grupo que foi resolver outros problemas. Se escolhermos mais uma (71), esta certamente foi resolver problemas de documentação ou de multas. Se escolhermos mais uma, chegando a 72, esta também foi resolver problemas de documentação ou de multas. Mas pode ser que a 71ª tenha ido resolver apenas um desses problemas (ex.: documentação) e a 72ª tenha ido resolver apenas o outro (multas). Ao escolher a 73ª, esta também certamente foi resolver problemas de documentação ou de multas. Seja qual for, podemos garantir que agora temos pelo menos 2 pessoas que foram resolver problemas de documentação ou de multas.



    Fonte: ESTRATÉGIA CONCURSOS

  • Método da pior hipótese.

  • Veja que, se selecionarmos 70 pessoas, pode ser que as 70 façam parte do grupo que foi resolver outros problemas. Se escolhermos mais uma (71), esta certamente foi resolver problemas de documentação ou de multas. Se escolhermos mais uma, chegando a 72, esta também foi resolver problemas de documentação ou de multas. Mas pode ser que a 71ª tenha ido resolver apenas um desses problemas (ex.: documentação) e a 72ª tenha ido resolver apenas o outro (multas). Ao escolher a 73ª, essa também certamente foi resolver problemas de documentação ou de multas. Seja qual for, podemos garantir que agora temos pelo menos 2 pessoas que foram resolver problemas de documentação ou de multas. Item CERTO.

  • Eu entendi assim:

    Total → 210 pessoas

    Documentação de veículos → 105 pessoas (chamaremos de grupo DC)

    Multas → 70 pessoas (chamaremos de grupo M)

    Nem doc. de veiculos e nem multas → 70 pessoas (chamaremos de grupo OUTROS)

    Se as 70 pessoas do grupo OUTROS não estão contidas nos grupos DC e M, podemos inferir que o grupo formado pelas pessoas dos grupos DC + M = 140, ou seja, existem elementos comuns aos dois grupos.

    Vejam: 140 (DC+M) somadas às 70 (OUTROS) dá um total de 210 pessoas.

    .

    Com base nessa informação, iremos deduzir que para selecionarmos, de forma aleatória, um quantitativo de pessoas que NECESSARIAMENTE possa garantir que ao menos duas foram ao Detran para resolver problemas relacionados aos grupos DC ou M, é necessário que esse novo grupo tenha, no mínimo, 73 pessoas, pois 70 poderão ser do grupo OUTROS e as TRÊS pessoas restantes, ao menos duas, foram resolver problemas de documentação ou de multas;

    .

    Espero ter ajudado.

    .

  • Fiz pelo princìpio da casa dos pombos: que é na pior das hipóteses selecionei todos aqueles que nao quero, para depois aparecer o que quero.

    Se eu selecionar as 70 que ñ foram resolver nada, já terei escolhido 70 pessoas.

    Tenho ainda de selecionar duas: que foi SÓ DOC ou que foi SÓ MULTA.

    Nessa situação, seleciono 2 PESSOAS: que pode ser 1 DOC e 1 MULTA.

    Ainda sim, está faltando 1 PESSOA para completar.

    LOGO:

    70+2+1=73

  • Testando a pior hipótese

    70 são outros

    1 doc

    1 pra multa ... ainda não cheguei ao que quero poisso quero 2 iguais

    Mais 1 que pode ser doc ou multa

    70+1+1+1=73

  • Aqui a ideia é simples... Temos um grupo que não foi resolver problemas nem de multa e nem de cnh (70 pessoas). Então, o pior cenário seria selecionar 70 pessoas e essas estarem nesse grupo. Se a gente pegar mais uma, ela ou vai resolver problema de multa ou cnh. Se pegar mais uma ela pode também resolver ou multa ou cnh (aqui totalizou 72). O problema pediu ao menos duas que faça ambos. Logo se selecionarmos mais uma (total de 73), é a nós garantido que ao menos 2 resolverão ou cnh ou multa. Portanto, está certa a questão.


ID
249802
Banca
ESAF
Órgão
SMF-RJ
Ano
2010
Provas
Disciplina
Raciocínio Lógico
Assuntos

Em cada um de um certo número par de cofres são colocadas uma moeda de ouro, uma de prata e uma de bronze. Em uma segunda etapa, em cada um de metade dos cofres, escolhidos ao acaso, é colocada uma moeda de ouro, e em cada um dos cofres restantes, uma moeda de prata. Por fi m, em cada um de metade dos cofres, escolhidos ao acaso, coloca-se uma moeda de ouro, e em cada um dos cofres restantes, uma moeda de bronze. Desse modo, cada cofre fi cou com cinco moedas. Ao se escolher um cofre ao acaso, qual é a probabilidade de ele conter três moedas de ouro?

Alternativas
Comentários
  • A probabilidade de um cofre qualquer conter pelo menos uma moeda de ouro é 100% porque em cada cofre foram colocadas uma moeda de ouro, uma de prata e uma de bronze. Após o segundo evento, apenas a metade dos cofres possui duas moedas de ouro, isto é, a probabilidade de um cofre qualquer ter duas moedas de ouro é de 50%. Após o terceiro evento, a probabilidade de um cofre qualquer ter três moedas de ouro é de:

    P = [Prob. Evento 1] x [Prob. Evento 2] x [Prob. Evento 3]
    P = 1 x 1/2 x 1/2 = 0,25
  • Eu representei... Achei que assim ficou mais fácil. Fui colocando as moedas de acordo com o comando da questão. Como ele não deu um número exato de cofres, mas disse que era par, eu coloquei 4 cofres.

    Cofre 1: 1 etapa ouro, prata, bronze/ 2 etapa ouro/ 3 etapa ouro

    Cofre 2: 1 etapa ouro, prata e bronze/ 2 etapa ouro/ 3 etapa ouro

    Cofre 3: 1 etapa ouro, prata e bronze/ prata/ bronze

    Cofre 4: 1 etapa ouro, prata e bronze/ prata/ bronze

     Desse modo, cada cofre fi cou com cinco moedas. Ao se escolher um cofre ao acaso, qual é a probabilidade de ele conter três moedas de ouro?

    Numero de elementos favoráveis                    LOGO     1/4         =     1/4 * 100 = 25   
    numero total de elementos                                                                 

  •  1) todos os potes começam com 3 moedas (uma de cada)
    2) 2 potes são escolhidos ao acaso para ganhar 2 moedas de ouro enquanto os outros dois restantes ganham 2 de prata
    Então temos: 
    Pote 1: O, P, B, O
    Pote 2: O, P, B, O
    Pote 3: O, P, B, P
    Pote 4: O, P, B, P

    3) a terceira instrução manda escolher dois potes aleatorios para colocar 2 moedas de ouro + 2 moedas de bronze, então ficamos com 3 possibilidades distintas:
    1- Pote 1 e 2 com 3 moedas de ouro cada
    2- Pote 1 com 3 moedas de ouro e pote 2 e 3 com 2 moedas
    3- Todos os potes com 2 moedas de ouro

    Se o cara fizer a possibilidade 1: ele terá 50% de chance de pegar 1 pote com 3 moedas de ouro
    Se o cara fizer a possibilidade 2: ele terá 25% de chance de pegar 1 pote com 3 moedas de ouro
    Se o cara fizer a possibilidade 3: ele terá 00% de chance de pegar 1 pote com 3 moedas de ouro

    chegando nessa etapa é só fazer a probabilidade dele ter escolhido uma dessas possibilidades:

    (1/3 x 50%) + (1/3 x 25%) + (1/3 x 0%) = 1/6 + 1/12 = 3/12 = 1/4 = 25% = 0,25 (D)

  • Vinicius, você mandou bem! Ótima explicação!

  • Link da resolução: https://www.youtube.com/watch?v=wAuMHR3pKeE

  • Valeu, Alcides!

  • Resposta D

    Difícil foi entender o encunciado

     

    #sefazal

  • Complementando o amigo Alcides Camargos

    https://www.youtube.com/watch?v=wAuMHR3pKeE (tempo: 10min 50s)

  • Vamos seguir os passos do enunciado, considerando que temos um número par de cofres, neste caso 2xN cofres.

    - Em cada um de um certo número par de cofres são colocadas uma moeda de ouro, uma de prata e uma de bronze.

           Portanto, cada um dos 2N cofres tem 1 moeda de cada tipo.

    - Em uma segunda etapa, em cada um de metade dos cofres, escolhidos ao acaso, é colocada uma moeda de ouro, e em cada um dos cofres restantes, uma moeda de prata.

           Portanto, N cofres passam a ter 2 moedas de ouro, 1 de prata e 1 de bronze; e N cofres passam a ter 1 moeda de ouro, 2 de prata e 1 de bronze.

     Por fim, em cada um de metade dos cofres, escolhidos ao acaso, coloca-se uma moeda de ouro, e em cada um dos cofres restantes, uma moeda de bronze.

           Até aqui, veja que N cofres possuem 2 moedas de ouro e outros N possuem apenas uma. Ao escolher, ao acaso, metade dos cofres para colocar mais uma moeda de ouro, serão escolhidos novamente N cofres. Porém estes não serão, necessariamente, os mesmos N cofres que já tem 2 moedas de ouro. A chance de escolher um cofre que já possui 2 moedas de ouro é P = N/2N = 1/2. Portanto, espera-se que 1/2 dos N cofres que já tinham 2 moedas de ouro passem a ter 3. Isto é, N/2 cofres do total de 2N cofres terão 3 moedas de ouro.

    Ao se escolher um cofre ao acaso, qual é a probabilidade de ele conter três moedas de ouro?

           Essa probabilidade será dada por:

    Resposta: D


ID
254890
Banca
CESPE / CEBRASPE
Órgão
PC-ES
Ano
2011
Provas
Disciplina
Raciocínio Lógico
Assuntos

Para o bom desempenho das funções dos agentes, os departamentos de polícia frequentemente realizam compras de equipamentos. Para certa compra licitada, um fabricante ofereceu 6 modelos de radiotransmissores. Com base nessa situação, julgue o item que se segue.

Suponha que, para cada lote de 3 radiotransmissores de determinado modelo, a probabilidade de 1 deles apresentar defeito é 0,25, de 2 deles apresentarem defeito é 0,025, e de 3 apresentarem defeito é 0,0005. Nessa situação, considerandose que, se pelo menos 1 dos radiotransmissores de um lote apresentar defeito, todo o lote será rejeitado, é correto afirmar que a probabilidade de se rejeitar um lote é inferior a 25%.

Alternativas
Comentários
  • Se o enunciado diz que a probabilidade de 1 radiotransmissor apresentar defeito é de 25% (0,25) e também fala que se apenas um deles der problema o lote todo será devolvido, fica claro que a probabilidade de se rejeitar um lote será igual a 25%.

    Bons estudos!
  • Então pq a resposta dessa questão é ERRADO no gabarito?
  •      A probabilidade de um apresentar o defeito é de 0,25 (P1=0,25), a de dois é 0,025(p=0,025) e a de três é de 0,005(P3=0,005).
        Para que o lote seja rejeitado precisa-se de pelo menos um. Então, pode ser P1 OU P2 OU P3 (P1 +P2 +P3)
        Assim, 0,25+0,025+0,005 > 0,25

    Abraços e firmes nos estudos!
  • Amigos, já disse em outra assertiva e afirmo nesta, algumas questões de probabilidade (e outras) têm a própria resposta na pergunta.

    Vejam,  se a probabilidade de 1 deles apresentar defeito é 0,25....PAROU AQUI...!!! Nem precisa mais ler, com certeza não será inferior a 25% no mínimo igual ou maior.
  • Questão ERRADA.
    Discordo de alguns comentários acima, pois alguns chegaram à resposta por dedução, porém, de forma incorreta.
    A questão é lógica, visto que ela quer a probabilidade de se rejeitar um lote.
    Ora, todas as proposições traz pelo menos 1 lote rejeitado, então, soma-se TUDO.
    Basta somar 0,25 + 0,025 + 0,005 = 27,5% (superior a 25%)
    Bons estudos.
  • 0,25 + 0,025 + 0,005 = 0,28 ou 28%.

  • Tipos de questões que, infelizmente, não voltam mais rsrsrs...

  • 25% de 100%= 4... se fala de 3 radiotransmissores, questão errada. não precisava nem fazer conta!

  • RESPOSTA E

    >> defeito é 0,25, de 2 deles apresentarem defeito é 0,025, e de 3 apresentarem defeito é 0,0005. >> se pelo menos 1 >> de se rejeitar um lote é inferior a 25%.

    temos que ler palavras por palavra.

    #SEFAZ-AL


ID
256120
Banca
FCC
Órgão
Banco do Brasil
Ano
2011
Provas
Disciplina
Raciocínio Lógico
Assuntos

Para disputar a final de um torneio internacional de natação, classificaram-se 8 atletas: 3 norte-americanos, 1 australiano, 1 japonês, 1 francês e 2 brasileiros. Considerando que todos os atletas classificados são ótimos e têm iguais condições de receber uma medalha (de ouro, prata ou bronze), a probabilidade de que pelo menos um brasileiro esteja entre os três primeiros colocados é igual a:

Alternativas
Comentários
  • Bem vamos lá. Eu me atrapalhei muito pra fazer essa questão do modo direto então eu fiz de outra forma...
    Sabendo que “1” é 100% das chances e “a + b = 1”, sendo “a” a quantidade de possibilidades de pelo menos 1 brasileiro ganhe e “b” que nenhum brasileiro ganhe:
    Para que nenhum brasileiro ganhe os outros tem q ganhar ou seja, que os outros 6 cheguem nas 3 primeiras posições, logo:
    Retirando os 2 brasileiros das 3 primeiras posições; 6 pra primeira posição dos 8 totais, 5 pra segunda posição dos 7 restantes e 4 pra terceira posição dos 6 restantes, fica: b = 6/8*5/7*4/6 = 5/14
    E... 1 – 5/14 = 9/14
  • Um raciocinio mais simples e rapido é pensar assim, primeiramente vamos excluir os 2 brasileiros e depois tirar o complementar, e assim teremos a solução!
    Logo nos resta 6 atletas. A forma que eles podem se organizar é 6/8*5/7*4*6 = 5/14.
    E o seu complementar é 1 - 5/14 = 9/14.

    Pense de forma prática, ganhe tempo.
  • PROBABILIDADE:
     SÃO 8 ATLETAS: 2 BRASILEIROS E 6 NÃO BRASILEIROS
    PROBABILIDADE DE NÃO SER BRASILEIRO:
    P(3)=6/8*5/7*4/6= 5/14
    PROBABILIDADE DE PELO MENOS 1 SER BRASILEIRO: P(Ñ BR) = 1 - (Ñ BR)
    P= 1-(5/14) = 9/14
  • Olha só, dica para quando aparecer na questão `pelo menos um`: encontra a probabilidade do evento pedido não acontecer, ou seja, de não termos brasileiros no pódio. Depois, diminui de 1, ok?
    Ficam as probabilidades:
    Medalha de ouro (não brasileiro) = (3 + 1 + 1 + 1)/8 = 6/8 = 3/4
    Medalha de prata (não brasileiro) = 5/7 (temos que retirar o medalha de ouro do cálculo, ok?)
    Madalha de bronze (não brasileiro) = 4/6 = 2/3 (agora, retiramos do cálculo os medalhas de ouro e prata!)
    Então:
    P (não termos brasileiros no pódio) = 3/4 x 5/7 x 2/3 = 5/14
    Agora, ao diminuirmos de 1, você encontrará a probabilidade de pelo menos 1 brasileiro no pódio, correto?
    P (pelo menos 1 brasileiro no pódio) = 1 - P (não termos brasileiros no pódio)
    = 1 - 5/14 = 14/14 - 5/14 = 9/14
    Resposta: letra D.

    http://www.euvoupassar.com.br/?go=artigos&a=8zq3aiSFlU4auS2IFR5FRLj9O_4rhGeouppdVNcuItw~
  • O enunciado pede que calculemos a probabilidade de que pelo menos um brasileiro esteja entre os três primeiros colocados, assim, vamos calcular a probabilidade de não termos nenhum brasileiro no pódio.

    Feito isso, vamos subtrair o resultado por um, já que nos é pedido a probabilidade de que pelo menos um brasileiro esteja no pódio. Lembrando que é sem reposição no cálculo de cada probabilidade abaixo:


    Letra D.










  • Temos o caso de eventos complementares.


    A prob. do evento ocorrer somado a de ele não ocorrer é igual a 1.

    P(A) + P(Ã) = 1


    Sendo assim, basta calcular a prob. de ele não ocorrer, subtrair por 1 (100%), daí teremos a prob. de pelo menos um brasileiro ir ao pódio.


    N = não ir ao pódio

    P = N * N * N

    P = 6/8 * 5/7 * 4/6 = 120/336 = 5/14 (probabilidade de nenhum brasileiro ir ao pódio)


    A probabilidade de ao menos um ir ao pódio é de 9/14.

  • Eu fiz assim:

    Maneira 1: pela exclusão (+ simples)

    Total de possibilidades: T = 8 x 7 x 6

    Possibilidades de pódio sem nenhum dos dois brasileiros: P = 6 x 5 x 4 

    T / P = da divisão resulta = 5/14 (esse é o resultado sem nenhum brasileiro) >> Para achar o resultado com pelo menos 1 brasileiro é o que resta da fração, ou seja, 9/14.

    ***

    Maneira 2: resultado direto

    Total de possibilidades: T = 8 x 7 x 6 (mesma coisa)

    Possibilidades de pódio com 1 brasileiro: P1 = 2 x 6 x 5 x 3  

    O "2"  foi utilizado porque são dois brasileiros e qualquer um dos deles pode assumir o pódio.

    O "3" foi utilizado porque o brasileiro pode ficar com qualquer medalha.

    Possibilidades de pódio com 2 brasileiros: P2 = 2 x 1 x 6 x 3

    O "2"  foi utilizado porque são dois brasileiros e qualquer um deles pode assumir o pódio.

    O "1" foi utilizado porque o outro brasileiro também fará parte.

    O "3" foi utilizado porque os brasileiros podem ficar com qualquer medalha.

    Possibilidade de pódio com pelo menos 1 brasileiro: os dois P são somados = P1 + P2

    P / T = P1 + P2 / T = 9/14

  • Solução em vídeo: https://youtu.be/nN6rsT8GzYk

  • Resolvo essa questão aqui nesse vídeo

    https://youtu.be/2G9ul_1Zhmw

    Ou procure por "Professor em Casa - Felipe Cardoso" no YouTube =D

  • Galera boa tarde, assim como nosso amigo Felipe, também gravei um vídeo comentando esta questão

    https://youtu.be/ieEp1abVidM


ID
257851
Banca
FUNIVERSA
Órgão
EMBRATUR
Ano
2011
Provas
Disciplina
Raciocínio Lógico
Assuntos

Os aposentos de um hotel internacional estão distribuídos em corredores com uma dúzia de apartamentos de cada lado. Um dos corredores está com os apartamentos ocupados, exceto um deles, que fica no meio do corredor. Um hóspede chega ao hotel e será acomodado naquele aposento. Se os hóspedes já acomodados naquele corredor forem redistribuídos aleatoriamente no mesmo corredor, e, exatamente, dez deles tiverem a mesma nacionalidade do hóspede que será acomodado agora, então a probabilidade de que os três vizinhos — à esquerda, à direita e à frente — do novo hóspede sejam seus compatriotas é

Alternativas
Comentários
  • Desculpe se eu estiver errada, mas O que que esta questão está fazendo aqui??? rs
  • nao vejo razao pra ela nao estar

  • então , por favor responda....
  • Letra B - Temos um total de 23 já hospedados, sendo 10 destes compatriotas (CP) do novo hospede (NH), distribuição:
    _  _  _  _    _     CP1   _   _  _  _  _  _
    _  _  _  _  CP2  NH  CP3  _  _  _  _  _

    Portanto a probabilidade de os 3 vizinhos - à direita, à esquerda e à frente - do novo hóspede serem seus compatriotas é:
    P = 10/23 . 9/22 . 8/21
    P = (10 . 9 . 4) / (23 . 11 . 21)
    P = 360 / 5313
    Portanto temos: 0,05 (5%)  <  360/5313  <  0,10 (10%)

    OBS: A observação de que ele ficará no meio do corredor, serve somente para não constestarmos o fato de ele ter vizinhos tanto do seu lado direito, como do seu lado esquerdo.

ID
265516
Banca
CESPE / CEBRASPE
Órgão
TRE-ES
Ano
2011
Provas
Disciplina
Raciocínio Lógico
Assuntos

Uma escola promove, anualmente, um projeto para
incentivar a participação de seus alunos nos processos eleitorais. A
cada ano, são escolhidos 5 professores, que orientarão um grupo de
100 alunos em várias atividades. No início deste ano de 2011, a
escola conta com 35 professores, dos quais 15 já participaram do
projeto em anos anteriores; dos 800 alunos matriculados, 300 já
participaram do projeto em outras oportunidades e 600 já são
eleitores.

Com base na situação apresentada acima, julgue os itens a seguir.

Escolhendo-se ao acaso, na escola, um professor e um aluno, a probabilidade de ambos ainda não terem participado do projeto é inferior a 35%.

Alternativas
Comentários
  • Probabilidade do professor não ter participado = 20/35
    Probabilidade do aluno não ter participado do projeto = 500/800

    Probalidade de ambos não terem participado: 20/35 x 500/800 = 5/14 (com as devidas simplificações) = 35,7%
  • Meus cálculos de alunos que não participaram deu 550. Do mesmo jeito o gabarito deu certo.
  • Meu cáculo na calculadora deu 0,35625. Agora ruim mesmo é fazer isso na caneta depois de tantos anos usando máquinas.
  • Que isso galera... o comentario do Vascaino ai em cima ta certo pow.
    20/35 x 500/800 = 4/7 x 5/8 =
    = 5/14 = 0.357... = ~ 35,7%
    Sem mais, simples assim
  • Realmente, nesse caso, os vascaínos estão certos... rs

  • Não tem que existir permutação de 2? claro que mesmo existindo, a resposta será "errado" do mesmo jeito. mas...

  • Não concordo com o gabarito pelo seguinte:

     "dos 800 alunos matriculados, 300 já participaram do projeto em outras oportunidades e 600 já são eleitores".

    Temos 800 alunos no total. Claro que temos uma intersecção. senão teríamos 600 + 300 = 900 alunos??? errado.

    Note que, desse total 300 já participaram e, 600 já são eleitores. Isto quer dizer, por outras palavras que temos 300 que já participaram + 300 que já são eleitores (num total de 600 que já são eleitores) e, por fim, 200 que não participaram e consequentemente não são eleitores. (600 + 200 = 800 alunos na totalidade)

    por isso a conta é 20/35 x 200/800 = 14%

    Quem já é eleitor ou já participou não entra na conta!

    Não concordo com o gabarito!

  • Gabarito: Errado.

    Wellingon Farias,

    Teu comentário não procede. Uma escola incentivar alunos em processo eleitorais, não quer dizer que os processos ocorrem dentro da escola. Eu posso ser eleitor já, por exemplo, e não ter participado do projeto da escola. Assim como posso não ser eleitor ainda e participar da parada da escola. São coisas diferentes. Não há intreseção ou incoerência com os dados apresentados. Você interpretou de maneira equivocada, amigo.

    20/35 dos professores x 5/8 dos alunos = 5/14 = 35,7%.

    Não há o que questionar.

    Bons estudos!


ID
266017
Banca
CESPE / CEBRASPE
Órgão
PC-ES
Ano
2011
Provas
Disciplina
Raciocínio Lógico
Assuntos

Para o bom desempenho das funções dos agentes, os departamentos
de polícia frequentemente realizam compras de equipamentos. Para
certa compra licitada, um fabricante ofereceu 6 modelos de
radiotransmissores. Com base nessa situação, julgue o item que se
segue.

Suponha que, para cada lote de 3 radiotransmissores de determinado modelo, a probabilidade de 1 deles apresentar defeito é 0,25, de 2 deles apresentarem defeito é 0,025, e de 3 apresentarem defeito é 0,0005. Nessa situação, considerando- se que, se pelo menos 1 dos radiotransmissores de um lote apresentar defeito, todo o lote será rejeitado, é correto afirmar que a probabilidade de se rejeitar um lote é inferior a 25%.

Alternativas
Comentários
  • Para que um lote seja rejeitado, basta que UM dos radiotransmissores apresente defeito. Temos a possibilidade de UM (0,25=25%) está com defeito, ou DOIS(0,025=2,5%) estarem com defeito ou (soma) TRÊS(0,0005=0,05%) estarem com defeito. A probabilidade que uma dessas caixas seja rejeitada basta que dê uma dessas situações. OU uma OU outra.....Logo, 25%+2,5%+0,05%=27,55%



    QUESITO ERRADO

    ATÉ MAIS!

    ;)
  • Então Edson, pensei até da mesma forma que você, só não aprofundei muito como fez...rs...Acho q nem precisaríamos de fazer conta e no entanto, já economizaríamos tempo. Poderíamos resolver essa questão por simples exclusão, se pensarmos:

    Se a questão nos fala que a probabilidade de 1 apresentar defeito é 0,25, ou seja, 25%, e para rejeitar um lote pelo menos 1 tem q apresentar defeito, logo a probabilidade de se rejeitar um lote não tem como ser inferior que 25%. Questão Errada.
     
    Mas de qualquer forma, acho interessante sempre "tirarmos a prova" com cálculos, pois questões como essas podem parecer óbvias, mas pode ser no final um “pega” né?! Ainda mais vindo do Cespe! Rs... 
     

    Bons Estudos!! :)
  • Certissimo Elaine, bons estudos!
    []s
  • a metodologia para qualificar, não parece das melhores assim ....retiro meu comentário sobre a questão..rs
  • Nessa situação, considerando- se que, se pelo menos 1 dos radiotransmissores de um lote apresentar defeito, todo o lote será rejeitado, é correto afirmar que a probabilidade de se rejeitar um lote é inferior a 25%.-errado. a probabilidade de 1 estar com defeito é igual -e não inferior- a 25%. O lote é rejeitado com no mínimo 25% de possibilidades.

    Com o restante da informação: 1/4+1/40+1/2000=27.55%
  • Alguém saberia informar como ele chegou nessas porcentagens?
    1 -> 3 ----> 0,25
    2 -> 3 ----> 0,025
    3 -> 3 ----> 0,0005

    Sei que o problema já deu na questão, mas como dúvida mesmo...
  • A questão é clara: pelo menos um apresentar defeito. A questão diz também que a probabilidade de um apresentar defeito é de 0.25, ou seja, 25%. Não tem muito o que inventar: rejeitar um lote não é inferior a 25%, é no mínimo 25%.

  • Resposta do Cursinho Damásio:

    Errado.
    P1: probabilidade de apresentar uma falha : 0,25
    P2: probabilidade de apresentar duas falhas: 0,025
    P3: probabilidade de apresentar três falhas : 0,0005
    Probabilidade de P1 OU P2 OU P3 = 25% + 2,5% + 0,5 % = 28%
  • O item está errado. Mas essa resposta de 27,5% também está errada! Percebam que os eventos não são mutuamente exclusivos. Se dois rádios estão com defeito, então certamente um rádio está com defeito! A regrinha de somar as probabilidades é apenas para mutuamente exclusivos, ou seja, um evento só ocorre se necessariamente o outro não ocorrer.
    Para eventos não mutuamente exclusivos, temos que tirar a probabilidade da interseção: P (A ou B) = P(A) + P(B) - P(A interseçaõ B).
    Voltando a questão, como P(um rádio) = 0.25, o máximo que podemos saber é que a probabilidade é no mínimo 25%, nada mais.
    ;]
  • Ola. é isso ai sigo o mesmo comentario do Yuri Maia
    se apresentar um defeito é 25% acima de um erro, CONTINUA 25%
    agora, se a questao pedisse pra analizar 3 lotes ai seria diferente.
    mais como e so um
    25% e questao errada pois ele fala inferior a 25%
  • é também acho que essa conta não precisa ser feita , só pelo fato da questão falar que um  deles apresentar defeito é de 0,25 ou 25% já deixa a questão errada quando fala que a probabilidade de se rejeitar um lote é inferior a 25%, sendo que o lote tem 3 transmissores e um deles com defeito já equivale a 25%.





    abraço!!!!!
  • A cada 3 tem 25% de chance de 1 estar defeituoso, entao no MÍNIMO existe 25% de chance do lote ser rejeitado, nao precisa nem calcular a probabilidade de 2 e 3 defeituoso.

  • essa não precisava nem fazer a conta...........ela foi elaborada pra pegar bisonho mesmo

  • Que rádio bem ruim.

  • é correto afirmar que a probabilidade de se rejeitar um lote é de 25% + mais 2,5% + 0,05%.

  • Eu não fiz conta nenhuma se pelo menos um apresentar defeito o lote é rejeitado, logo a questão fala que a probabilidade de um apresentar defeito é de 0,25 então tem-se que a resposta não é inferior a 0,25 mas sim igual a 0,25

  • Um lote tem três rádios, a probabilidade de 1 apresentar defeito é inferior a 25%?

    Errado.

    1/3 = 33,333

    Pensei dessa forma.

  • 1-(3/4)= 1/4 = 25%.

  • 0,8255% do Lote ser Rejeitado (0,25% se o transmissor A estiver com defeito + 0,25% se o transmissor B estiver com defeito + 0,25% se o transmissor C estiver com defeito + 0,025% se os transmissores A e B estiverem com defeito + 0,025% se os transmissores B e C estiverem com defeito + 0,025% se os transmissores A e C estiverem com defeito + 0,0005 se os transmissores A e B e C estiverem com defeito) ou seja,

    somando todas as possibilidades 0,8255% de probabilidade de ser rejeitado e 0,174% de ser aprovado

    Eita que lote ruim da pega


ID
266875
Banca
CESPE / CEBRASPE
Órgão
PREVIC
Ano
2011
Provas
Disciplina
Raciocínio Lógico
Assuntos

Considerando que, em uma concessionária de veículos, tenha sido
verificado que a probabilidade de um comprador adquirir um carro
de cor metálica é 1,8 vez maior que a de adquirir um carro de cor
sólida e sabendo que, em determinado período, dois carros foram
comprados, nessa concessionária, de forma independente, julgue os
itens a seguir.

A probabilidade de que os dois carros comprados sejam de cor metálica é 3,24 vezes maior que a probabilidade de que eles sejam de cor sólida.

Alternativas
Comentários
  • 1.8x*1.8x = 3.24x^2; Sólidos = x^2; então 3.24x^2/x^2 = 3.24. Correto.

  • Pm=1,8*Ps

    Pm+Ps=1 (Pois só da pra comprar carro metálico OU carro sólido, as probabilidades se completam)

    Logo, Ps=10/28 e Pm=18/28

     

    Assim,

    A probabilidade de que sejam comprados 2 carros de cor sólida é: A=(10/28)² (pois a compra de cada carro é independente)

    A probabilidade de que sejam comprados 2 carros de cor metálica é: B=(18/28)² (pois a compra de cada carro é independente)

    Logo,

    B = 3,24A ou 324/784 = 3,24*(100/784)

     

    CERTA

     

     

  • A probabilidade de que sejam comprados 2 carros de cor sólida é: 1/2 * 1/2 = 0,25

    A probabilidade de que sejam comprados 2 carros de cor metálica é: 1,8/2 * 1,8/2 = 0,81

    0,81 / 0,25 = 3,24

     

  • Sendo P(x) a probabilidade de se comprar um carro de cor metálica

    e P'(x) a probabilidade de se comprar um carro de cor sólida,sabemos que:

    P(x) = 1,8P'(x) , qual a probabilidade dos dois carros comprados serem da cor metálica?

    Resposta: P(x) * P(x) = 1,8P'(x) * 1,8P'(x) = 3,24P'(x)²

    qual a probabilidade dos dois carros comprados serem da cor sólida?

    Resposta: P'(x) * P'(x) = P'(x)²

    Assim se dividirmos a probabilidade de termos dois carros metálicos, pela probabilidade de termos dois carros de cor sólida, saberemos quantas vezes a primeira probabilidade é maior que a segunda, logo:

    3,24P'(x)²/P'(x)² = 3,24

    Portanto a assertiva está CORRETA

  • P(M) = 1,8 P(S)

     

    Levando em consideração que só há como existir carros de cor metálica e sólida na concessionária, então:

     

    1,8P(S) + P(S) =1

     

    2,8P(S)=1

     

    P(S)= 1/2,8=10/28

     

    Logo, P(M) = 1,8*(10/28) = 18/28

     

    Metalíca e Metálica = 18/28 * 18/28 = 324/784

     

    Sólida e Sólida = 10/28 * 10/28 = 100/784

     

    CORRETO

  • 1,8 x 1,8 = 3,24


ID
266878
Banca
CESPE / CEBRASPE
Órgão
PREVIC
Ano
2011
Provas
Disciplina
Raciocínio Lógico
Assuntos

Considerando que, em uma concessionária de veículos, tenha sido
verificado que a probabilidade de um comprador adquirir um carro
de cor metálica é 1,8 vez maior que a de adquirir um carro de cor
sólida e sabendo que, em determinado período, dois carros foram
comprados, nessa concessionária, de forma independente, julgue os
itens a seguir.

A probabilidade de que somente um dos dois carros comprados seja de cor metálica é superior a 50%.

Alternativas
Comentários
  • 1.8x * x + x* 1.8x = 180/784 + 180/784 = 360/784 < 392/784 = 50%

  • Depois de um bom tempo acredito que entendi direito essa questão.

    Primeiro a questão quer que EXATAMENTE um dos dois carros seja de cor metálica. Então podemos montar a seguinte equação:

     

    Pm*Ps + Ps*Pm (ou seja,  probabilidade do primeiro carro metálico E  probabilidade do segundo carro sólido OU  probabilidade do Primeiro carro sólido E  probabilidade do segundo carro metálico)

     

    Temos que Pm=1,8Ps e sabe-se que Pm + Ps = 1 (já que só existe carro metálico ou carro sólido, assim, a probabilidade de um é a probabilidade de o outro não acontecer)

     

    Através de um sistema a gente descobre que 

    Pm=18/28

    Ps=10/28

     

    Voltando à equação: Pm*Ps + Ps*Pm, sabendo que os dois eventos são independentes.

     

    (18/28*10/28)+(10/28*18/28) = 180/784+180/180/784 = 360/784 = 45,91%

     

    Ua forma mais rápida de saber se é ou não menor que 50% é assim:

     

    temos 360/784, quanto seria 50% com a base 784?

    seria 784 dividido por 2 sobre 784 ---> 392/784.

     

    360/784<392/784

     

    ERRADO

     

     

     

  • ERRADO

     

    Não sei se tá certo, foi o jeito que eu fiz

     

    Colocando em números para facilitar

     

    18 carros metálicos

    10 carros sólidos

     

    Em probabilidade

     

    metálico: 64,3%

    sólido: 35,7%

     

    Para somente 1 ser metálico o outro tem que ser sólido, 65,3*35,7 = 23%

  • P(M) = 1,8 P(S)

     

    Levando em consideração que só há como existir carros de cor metálica e sólida na concessionária, então:

     

    1,8P(S) + P(S) =1

     

    2,8P(S)=1

     

    P(S)= 1/2,8=10/28

     

    Logo, P(M) = 1,8*(10/28) = 18/28

     

    "... probabilidade de que somente um dos dois carros comprados seja de cor metálica"

     

         M       e      S        ou         S       e      M                         

     

    18/28  * 10/28  +     10/28 * 18/28  =  0,46        

     

    ERRADO

     

     

  • pm + ps = 1

    pm = 1,8ps

    1,8ps + ps =1

    2,8ps=1

    ps=1/2,8 = 10/28

    pm=1,8/2,8= 18/28

    (10/28)*(18/28)= 180/784

    permutção de dois números --- ---

    = 2!

    (180/784)* 2! = 0,459 ~0,46

    Resposta: 46%

  • A probabilidade de que somente um dos dois carros comprados seja de cor metálica é superior a 50%.

    Compro o primeiro carro da cor metálica e depois um da cor solida. A outra opção é comprar primeiro um carro da cor sólida e depois o outro da cor metálica.

    -Cor Metalica: CM. Cor Sólida: CS

    CM + CS = 1 (100%)

    CM=1,8*CS.

    CM=18/28 CS=10/28

    1 CM e 1 CS ou 1 CM e 1 CS. Não pode ser os dois da cor metalica, apenas um.

    18/28*10/28 + 10/28*18/28

    =~45%

  • Na verdade ele não deixou claro se a ordem da compra de fato importa. Se a ordem importar a probabilidade será o dobro , mas como não mencionou nada cronológico não precisa multiplicar por 2


ID
267316
Banca
FUNIVERSA
Órgão
EMBRATUR
Ano
2011
Provas
Disciplina
Raciocínio Lógico
Assuntos

Ao se realizar um lançamento de um par de dados não viciados, com faces numeradas de 1 a 6, qual é a probabilidade de a soma dos pontos ser 3 ou 7?

Alternativas
Comentários
  • Resposta Letra E

    As possibilidades de se obter soma 3 são:

    1 no 1º dado e 2 no 2º dado
    2 no 1º dado e 1 no 2º dado

    Ou seja, são 2 opções.

    E, como lançando-se dois dados, cada um tendo 6 faces, poderemos obter 6.6 = 36 tipos de combinações de faces diferentes.

    Com isso, a probabilidade será igual ao número de opções desejadas (2) dividido pelo número total de opções (36):
    P= 2 / 36

    As possibilidades de se obter soma 7 são:

    1 no 1º dado e 6 no 2º dado
    2 no 1º dado e 5 no 2º dado
    3 no 1º dado e 4 no 2º dado
    4 no 1º dado e 3 no 2º dado
    5 no 1º dado e 2 no 2º dado
    6 no 1º dado e 1 no 2º dado

    Ou seja, são 6 opções.

    E, como lançando-se dois dados, cada um tendo 6 faces, poderemos obter 6.6 = 36 tipos de combinações de faces diferentes.

    Com isso, a probabilidade será igual ao número de opções desejadas (6) dividido pelo número total de opções (36):
    P= 6 / 36
     
    A QUESTÃO PEDE A PROBABILIDADE DA SOMA DOS PONTOS SER 3 OU 7. ENTÃO SOMAMOS AS DUAS PROBABILIDADES ENCONTRADAS.
    P= 2 / 36 + 6 / 36
    P= 2 / 9
  • Primeiro a probablidade da soma ser 3:
     P 1,2 = 1/6*1/6 *P2! = 2/36

     Probabilidade da soma ser 7

    P1,6 = 1/6*1/6* P2! = 2/36

    P 2;5 = 1/6*1/6 *P2! = 2/36

    P3,4 = 1/6*1/6 * P2! = 2/36

    Soma se: 2/36+2/36+2/36+2/36 = 8/36 = 2/9

    Bons Estudos !!
  • Para complementar segue espaço amostral

      1 2 3 4 5 6
    1 2 3 4 5 6 7  
      2 3 4 5 6 7 8
    3 4 5 6 7 8 9
    4 5 6 7 8 9 10
    5 6 7 8 9 10 11
    6 7 8 9 10 11 12

  • Simplificando a questão temos:

    Total dos casos possíveis = 36 (6 números no dado um x 6 números no dado 2)
    Probabiliade de conseguir o número três na soma dos dois dados = (2,1) (1,2) = 2 casos possíveis.
    Probabilidade de conseguir o número sete na soma dos dois dados = (4,3) (3,4) (5,2) (2,5) (6,1) (1,6) = 6 casos possíveis.
    Agora somo as duas probabilidades = 2+6=8.
    Feito isso, iremos dividir a probabilidade de ambos pelos casos totais = 8/36. Agora vou simplificar por 4. Oito dividido por quatro dá 2 e 36 dividido por quatro da 9. Achamos a resposta: 2/9.





     

  • Possibilidades de soma 3:

    (1,2) (2,1) = 2 possibilidades

     

    Possibilidades de soma 7:

    (1,6) (2,5) (3,4) (4,3) (5,2) (6,1) = 6 possibilidades

     

    Como são dois dados (6x6 = 36)

    e como a questão fala 3 ou 7, a dica é que o ou na probabilidade equivale a uma soma, logo:

     

    2/36 + 6/36 = 

    8/36 = 

    2/9


ID
267442
Banca
CESPE / CEBRASPE
Órgão
PREVIC
Ano
2011
Provas
Disciplina
Raciocínio Lógico
Assuntos

Estimou-se que, na região Norte do Brasil, em 2009, havia
1.074.700 analfabetos com 15 anos de idade ou mais, em uma
população total de, aproximadamente, 10.747.000 habitantes, e que
na região Centro-Oeste, no mesmo ano, havia 840.433 analfabetos
com 15 anos de idade ou mais, em uma população total de,
aproximadamente, 10.505.415 habitantes. A partir dessas
informações, julgue o item subsequente.

A probabilidade de uma pessoa com 15 anos de idade ou mais escolhida ao acaso em 2009, na região Norte ou na região Centro-Oeste, ser analfabeta é inferior a 20%.

Alternativas
Comentários
  • A questão trata de probabilidade da união, senão vejamos:
    P(analfas N ou analfas C-O) = P(analfas N) + P(analfas C-O) - P(analfas N e analfas C-O)
    logo:
    p/ regiao Norte:
    P(analfas N) = nºpossibilidades desejadas / nº possibilidades possiveis
    entao: P(analfas N) = 1074700 / 10747000 = 0,1 = 10%

    p/ regiao Centro-Oeste:
    P(analfas C-O) = 840433 / 10505415
    Nesse caso tem uma dica, já que numa questao dessas o importante é gastar o minimo de tempo possivel e evitar o trabalho braçal da divisão
    então temos que perceber q nessa divisão terão duas casas decimais, ou seja, o resultado será 0,0...
    então supondo o maior resultado possivel nessa divisao seria 0,099 = 9,9%

    portanto, aplicando na probabilidade pedida temos que:
    P(analfas N ou analfas C-O) = 10% + 9,9% - P(analfas N e analfas C-O)
    = 19,9% menos alguma coisa é logicamente um valor menor que 20%

    Gabarito: certo.
  • P(Norte) = 1.074.700 / 10.747.000 = 1/10

    P(Centro-oeste) = 840.433 / 10.505.415 = 8/100

    P(Norte) ou P(Centro-oeste) = 1/10 + 8/100 = 18/100 = 18%

  • Faremos da seguinte maneira:
    Qual nosso espaço amostral ?
    Resposta, toda a população, assim sendo: 10.747.000 + 10.505.415  = 21.252.415
    Qual a quantidade de casos favoráveis?
    Resposta todos os analfabetos com de 15 anos de idade ou mais, assim sendo: 840.433 + 1.074.700 = 1.915.133
      
    Daí, P =1915133/21252415  = 0,0901  = 9,01% (conta chata?) 
    Pois é mas basta saber se:
    P < 20% , isto é, P < 20/100, ainda P<0,2
    P = 1915133/21252415< 0,2 daí 1915133< 0,2*21252415 = 4250483
    De fato, é bem menor . Gabarito CERTO
  • Alguem pode explicar por que o comentário do Fernando não está correto? Eu pensei da mesma maneira; mesmo depois de ver a resposta correta, contudo, ainda não consigo enxergar onde está o erro desse raciocínio.
  • VITOR, EU TE EXPLICO CARA...
    VAI VER QUE PORQUE ESSA MANEIRA DE REOLVER E' TAO LOGICA E RAPIDA QUE O POVO NAO ENTENDEU A RAPIDEZ DO RACIOCINIO. TAMBEM RESOLVI ASSIM.
    SE 'E UM OU OUTRO BASTA SOMAR 10.747.000 COM 10.505.415 E 840.433 COM 1.074.700, DIVIR UM PELO OUTRO SE DER + DE 20% FALSO SE DER MENOS VERDADEIRO. ENTENDEU? RACIOCINIO RAPIDO E OBJETIVO DEMAIS PARA O POVO ENTENDER, JA VI POR AQUI PELOS COMENTARIOS DAS QUESTOES DE RACIOCINIO LOGICO QUE O POVO GOSTA MESMO DE COMPLCIAR O SIMPLES E FAZER MIL  FORMULAS ATE' PARA DIVIDIR  1 POR 1. QUER APOSTAR QUE VAO COLOCAR RUIM NO MEU COMENTARIO TBM? RSRSRSRS . LIGA PRA ISSO NAO QUE VOCE PIRA.
    10.747.000 + 10.505.415 = 21.252.415
    840.433 +1.074.700 = 1.915.133
    VEJA BEM CARA... AQUI EU NEM PRECISO FAZER A CONTA PRA VER QUE DA MENOS DE 20%... PQ SE 21.000.000 / 1.000.000 =20% ENTAO E' CLARO QUE 21.252.415 / 1.915.133 (QUASE 2.000.000) VAI DAR MENOS QUE 20%, NA VERDADE VAI DAR UM POQUINHO MAIS QUE 10%, MAS PRA QUE EU VOU FAZER ESSE CALCULO? PRA PERDER TEMPO NA PROVA? E' SO' BATER O OLHO E VER QUE DA BEM MENOS QUE 20%...MARCAR A RESPOSTA, IR APRA A PROXIMA E UM ABRACO. AGORA VAI TENTAR EXPLICAR ISSO PARA ESSE POVO Q CLASSIFICAR COMO RUIM UM COMENTARIO CORRETO IGUAL AO DO AMIGO ACIMA. NAO 'E TODO MUNDO QUE CONSEGUE ENTENDER NAO...
  • GANHANDO TEMPO!
    Escolha de um analfabeto no N ou no CO: N ou CO (somam-se as probabilidades do N e do CO)
    P(N) = 1.074.700 (analfabetos) / 10.747.000 (habitantes) = 1/10 = 10%
    P(CO) = 840.433 (analfabetos) / 10.505.415 (habitantes) = raciocinem da seguinte forma: 840.433 / 10.505.415, vejam que forçando para que esta probabilidade seja também de 10%, teríamos que aumentar o número de nalfabetos de 840.433 para 1.050.541 (1.050.541 / 10.505.415). Logo, temos menos analfabetos no CO suficientes para atingir o mínimo de 10%, já temos somente 840.433, que não atingem os 10%. Assim, a probabilidade final - de escolha de um analfabeto no N ou no CO - será 10% + P (CO), que será < 10%, ou seja, resposta correta: inferior a 20%.
  • É pensei no calculo so "andando com vírgula "  
    região Norte 10.747.000 --100%
                           1.074.700 -- 10%

               C- O ,10.505.415-- 100%
                           1.050.541,5-- 10%
    *nesta região a questão fala 840.433 analfabetos então nem calculei já marquei certo menor que 20% 
    Mas não sei c isso é muito aconselhável.
  • 1.074.700 (N) + 840.433 (C) = 1.915.133 ANALFABETOS

    10.747.000 (N) +  10.505.415 (C) = 21.252.415 HABITANTES

    Dividindo-> 0,09 = 9/100 = 9%

    Obs. Para fazer mais rápido vc pode arredondar para os valores mais próximos e vê se fica muito distante= 2 (milhões) /20 (milhões) = 10%

  • Não sei se vai ajudar alguém... mas eu fiz assim :

    1074750 / 10747000 = simplificando = 0,1 ou 10 %

    Soma isso( já que a questão pede OU e não E ) com o resultado de 840,433/ 10 505 415 . 

    Vejam que a única forma de passar de 20 porcento é esse resultado ser 11 % ou mais.No inicio da conta já vemos que vai dar 0,0 ( e mais algum número).9 é o maior número que pode vir depois.Ou seja, o máximo que a conta chegaria é 0,099999(...) = 9,9 %.Então nem fiz o resto da conta.Não passaria dos 20 nem na melhor das hipóteses.

    Não sei se deu pra entender...


  • Não pode somar as populações, pessoal. É só imaginar que o segundo grupo tivesse nada a ver com analfabetos, digamos que o número de casas na região CO. Daria um resultado diferente do que somar as populações, correto? A forma certa é quase 10% de um mais quase 10% de outro, que daria algo obviamente menor que 20%.

  • pega o numero de analfabetos no nordeste e divide pela população total

     

    1074700/10747000 = 0,1 ou 10%

     

    logo após, faça o mesmo com os numeros da região centro oeste

     

    840433/10505415 = 0,079 ou 7%

     

    como a questão pede "A probabilidade de uma pessoa com 15 anos de idade ou mais escolhida ao acaso em 2009, na região Norte OU na região Centro-Oeste, ser analfabeta é inferior a 20%."

     

    lembrando que o OU  serve para somarmos os resultados.....Portanto 10%+7% = 17%

     

    QUESTÃO CORRETA

  • Eh 18% ou 9%?

  • A probabilidade máxima seria 10%, visto que só vai escolher uma pessoa

  • 1º = 0,1

    2º vc sai aproximando depois vc soma os resultados 0,1 + 85/1050 = 0,089 + 0,1 = 0,189 = 18,9% kkkkkk

  • A forma correta de pensar quando temos o OU é da seguinda forma ( Prob 1 + Prob 2) - (P1xP2)

    P1 = 0,1

    P2= 0,0799

    P1XP2 = 0,007

    Entao : 0,1+0,07 - 0,007 = 17,2%

  • Essa questão é só bater o olho e ver que a Probabilidade do Norte é 10% do valor total, já a centro oeste é menos de 10% ( 840.433 de 10.505.415). Portanto, da menos de 20%.

  • Multipliquei os dois denominadores por 0,2 e vi que dá mais do que os dois respectivos numeradores.

  • Basta "tirar" a porcentagem de cada grupo e depois somar os dois resultados.


ID
277009
Banca
CESPE / CEBRASPE
Órgão
TRT - 21ª Região (RN)
Ano
2010
Provas
Disciplina
Raciocínio Lógico
Assuntos

Suponha que determinado partido político pretenda ter candidatos
próprios para os cargos de governador, senador e deputado federal
e que tenha, hoje, 5 possíveis nomes para o cargo de governador,
7 para o cargo de senador e 12 para o cargo de deputado federal.
Como todos os pré-candidatos são muito bons, o partido decidiu que
a escolha da chapa (governador, senador e deputado federal) será
por sorteio. Considerando que todos os nomes têm chances iguais
de serem escolhidos, julgue os itens seguintes.

A probabilidade de uma chapa ser sorteada é maior que (1/20)2

Alternativas
Comentários
  • Probabilidade...

    Governador = 1 vaga para 5 pessoas, logo, 1/5;

    Senador = 1 vaga para 7 pessoas, logo, 1/7;

    Deputado = 1 vaga para 12 pessoas, logo, 1/12.

    A chapa é formada pelos cargos de: governador E Senador E Deputado, logo, tais frações deverão ser multiplicadas e não somadas (obs.: soma-se quando o conectivo é OU).


    1/5 x 1/7 x 1/12 = 1/420 >>> a probabilidade de uma chance em 420 é MENOR que a probabilidade de uma chance em 400... é isso!! o examinador quis confundir.... olhando os números dá até vontade de marcar verdadeiro!!!
  • GOVERNADOR= 1/5
    SENADOR= 1/7
    DEPUTADO FEDERAL=1/12
    PROBABILIDADE = 1/5X1/7X1/12= 1/420.
     NESSE MOMENTO A GRANDE MAIORIA PODE TER ERRADO A QUESTÃO, POIS QUANDO OLHAMOS SOMENTE PARA O DENOMINADOR ERRAMOS A QUESTÃO. ISSO QUE A BANCA QUER!!!!! CUIDADO!!!!!

    1/400= 0,0025
    1/420= 0,0023... LOGO, 1/420 TEM PROBABILIDADE MENOR QUE 1/400.
    GABARITO: ERRADO.

  • Eu acertei a questão mas acho que o examinador comeu bola. a chance de uma chapa ser sorteada é de 100%. a chance de uma chama em especial ser sorteada é que é menor que 1/400
  • Como o tempo é fundamental na hora da prova, a maneira mais rápida de resolver a questão seria saber quantas chapas poderiam ser formadas e depois dividir por um, assim:
    5.7.12=420 chapas diferentes
    1/420<1/400 ou (1/20)² como está na prova.
  • Questão duplicada

    Q81715

  • - Comentário do prof. Arthur Lima (ESTRATÉGIA CONCURSOS)


    Existem 5x7x12 = 420 chapas possíveis. Logo a chance de uma dessas chapas ser sorteada é 1/420. Este número é menor do que 1/400, que é (1/400)², logo a assertiva está errada.


    Gabarito: ERRADO

  • 1/420 < 1/400

    ERRADO


ID
277012
Banca
CESPE / CEBRASPE
Órgão
TRT - 21ª Região (RN)
Ano
2010
Provas
Disciplina
Raciocínio Lógico
Assuntos

Suponha que determinado partido político pretenda ter candidatos
próprios para os cargos de governador, senador e deputado federal
e que tenha, hoje, 5 possíveis nomes para o cargo de governador,
7 para o cargo de senador e 12 para o cargo de deputado federal.
Como todos os pré-candidatos são muito bons, o partido decidiu que
a escolha da chapa (governador, senador e deputado federal) será
por sorteio. Considerando que todos os nomes têm chances iguais
de serem escolhidos, julgue os itens seguintes.

Considerando que José seja um dos pré-candidatos ao cargo de governador, a probabilidade de que José esteja na chapa sorteada será maior que 0,1.

Alternativas
Comentários
  • Se José é um dos 5 candidatos concorrendo à chapa do partido para o cargo de governador, sua chance de ser escolhido pelo sorteio será de:

    PJOSÉ = 1/5

    Como 1/5 = 0,2 > 0,1 = 1/10, a questão está correta.

  • P = 1/5
    P = 0,2


    correto



  • probabilidade de José estar na chapa é a mesma que probalidade de ele ser governador:
    que é: 1/5=0,2
  • Olá, discordo do caro colega: a questão sim, está CERTA, mas a probabilidade é maior, ou seja, na minha opinião, deve-se levar em conta os outros canditados também, haja visto que JOSÉ + A + B é uma chapa diferente de JOSÉ + A + C. Então eu calculei todo o espaço amostral 5 X 7 X 12 = 420  e calculei o que a questão não pede (as probabilidades de José NÃO estar na chapa): 4 X 7 X 12 = 252. Diminuindo 252 de 420 tem-se 168, que são as chances de José estar na chapa. Logo 168/420 = 0,4 ou 40% de chance de José ser escolhido nos diversos modos de chapas.
    Opinem, por favor, se eu errei, grato.
    Bons estudos....
  • Sem querer complicar, mas é a visualização do todo:

    Todas as chapas possíveis: 5 x 7 x 12 = 420

    Chapas com José: 1 x 7 x 12= 84

     84/420 = 1/5 = 0,2
  • Olá Moacir, concordo com o comentário do colega acima, segue outra forma de resolução:
    Possíveis: 5 x 7 x 12 = 420
    Chapas com José participando  J x 7 x 12 = 84
    Logo: 84 / 420 = 1 / 5 = 0,2
    Temos 0,2 > 0,1 , a questão está correta.
    Att.






  • fiquei muito desconfiada de tão facil...


ID
278038
Banca
CESPE / CEBRASPE
Órgão
TRT - 21ª Região (RN)
Ano
2010
Provas
Disciplina
Raciocínio Lógico
Assuntos

Considere o texto abaixo, a respeito de dez alunos, em que cada um
recebeu uma camiseta, e cada camiseta tinha uma única cor.

Cinco meninos; três cores.
Cinco meninas; quatro cores.
Equipes formadas,
Expectativas geradas,
De glória ou dores.

Com base nas informações acima, julgue o item que se segue.

Considerando que as camisetas sejam de apenas cinco cores diferentes, então, a probabilidade de três alunos terem camisetas da mesma cor será igual a 1.

Alternativas
Comentários
  • a solução é por lógica... ( princípio da casa do pombo)

    total de cores = 5 (c1,c2,c3,c4,c5)

    cinco meninos ---> 3 cores --->(c1,c2,c3) , logo 2 alunos com cores iguais (supondo c3,c2)
    ---> 2 alunos com a cor c3
    ---> 2 alunos com a cor c2
    ---> 1 aluno com a cor c1

    cinco meninas ---> 4 cores --->(c2,c3,c4,c5) , logo 1 aluna com cor igual (supondo na pior das hipóteses c2) ... nessas condições teremos:
    ---> 2 alunas com a cor c2 ---> total de alunos com cores iguais = 4
    ---> 1 aluna com a cor c3 ----> total de alunos com cores iguais = 3


    logo a probabilidade de 3 alunos terem a mesma cor sempre será 100% ou = 1

    fonte: http://br.answers.yahoo.com/question/index?qid=20101201011329AAklonc

    bons estudos
    paz de Cristo
  • A pergunta não é de probabilidade e sim de contagem, é uma aplicação do "Princípio da casa dos pombos": Se houver mais pombos do que casas, certamente em alguma casa haverá mais de um pombo!

    São dez alunos e cinco cores. Na divisão mais igualitária possível, haveria dois alunos com cada cor. Qualquer outro tipo de divisão irá fazer pelo menos tres alunos terem cores iguais.

    A sua divisão não foi igualitária, veja porquê:
    Cinco meninos; três cores. A única forma de não ter tres alunos com a mesma cor seria a distribuição: 2 meninos com a cor A, 2 meninos com a cor B e 1 aluno com a cor C.

    Cinco meninas; quatro cores. Uma cor não foi utilizada, seria a cor A? Então a cor B foi utilizada e temos pelo menos 3 alunos (2 meninos e 1 menina) usando a cor B. Seria a cor B a cor não utilizada pelas meninas? Então a cor A foi utilizada e temos pelo menos 3 alunos (2 meninos e 1 menina) usando a cor A.

    Seria outra cor a não utilizada pelas meninas? Então as cores A e B foram utilizadas e temos pelo menos 3 alunos (2 meninos e 1 menina) usando a cor A e outros 3 utilizando a cor B.

    Portanto, é inevitável que haja pelo menos 3 alunos utilizando a mesma cor de camiseta.

    A probabilidade de três alunos terem camisetas da mesma cor será igual a 100% = 1

    FONTE: http://br.answers.yahoo.com/question/index?qid=20110119174615AAxT0b0
  • 10 ALUNOS
    10 CAMISETAS
    10 CORES SENDO QUE 5 SÃO DIFERENTES; LOGO 5 SÃO IGUAIS.

    A PROBABILIDADE DE 3 ALUNOS TEREM A MESMA COR SERÁ DE PRIMEIRO 3/5, SEGUNDO 2/4 E O TERCEIRO 1/3.

    3/5 X 2/4 X 1/3 = 6/60 = 1/10

    OU SEJA A PROBABILIDADE É DE 1.
  • Oi pessoal!! De princípio eu errei a questão...tentei fazer combinação e tal...enfim, depois fui pensar no exercício e me veio essa resolução:

    São 5 cores diferentes: A, B, C, D, E.


    5 meninas - 4 cores diferentes, 1 cor igual:

    -ABCDA
    -ABCDB
    -ABCDC
    -ABCDD

    5 meninos - 3 cores difenrentes, 2 cores iguais

    -ABCAB
    -ABCAC
    -ABCBC

    Assim, nosso espaço amostral é 7. O evento favorável são as opções em que aparecem 3 cores igual, ou seja, todos os grupos vão possuir pelo menos 3 cores iguais....Sendo assim, a probabilidade é 1.

    Se alguém discordar da resolução por favor falar...sou péssima nessa parte de probabilidade!!
  • 10 alunos = (5 meninos e 5 meninas) ;  5 cores = (Cor1, Cor2, Cor3, Cor3, Cor4, Cor5)
    Na pior das hipóteses a distribuição ficaria assim: evitanto repitir cores
    5 meninos / 3 cores = (Cor1, Cor2, Cor3)
    5 meninas / 4 cores = (Cor4, Cor5, Cor1, Cor2) duas cores se repetirão no mínimo.
    Distribuição das meninas, na pior das hipóteses: evitanto cores repetidas
    5/4 = (2.Cor4, 1.Cor5, 1.Cor1, 1.Cor2)  como a Cor4 não se repete ela ficou com 2 meninas (ainda não tem 3 alunos c/ mesma cor)
    5/3 = (1.Cor1, 1.Cor2, 2.Cor3 + 1 menino) esse 1 menino que sobra, ao ser distribuido, vai gerar inevitavelmente 3 cores repetidas.
    Conclusão: No mínimo, no mínimo, terá 3 alunos c/ camiseta da mesma cor.

    Resposta: A probabilidade de três alunos terem(possuirem) camisetas da mesma cor será igual a 1 (100%).Ou seja, a probabilidade de não acontecer o evento é 0 (zero).

  • COMENTÁRIO RETIRADO DO SITE: EU VOU PASSAR

    Primeiramente, tem que ficar claro que estamos falando do grupo completo. O item fala em alunos, então são os 10 do texto inicial, ok?

    Agora, se os meninos utilizam 3 cores e as meninas 4, então 2 cores serão repetidas! Vamos pensar no azarado: 2 meninos usarão camiseta branca, 2 usarão camiseta preta e 1 usará cinza. Ou seja, não completaremos os 3 alunos com camisetas da mesma cor, correto?

    Se as meninas usarem: 2 com camiseta vermelha, 1 com camiseta azul (cores que ainda não apareceram), a última menina usará, OBRIGATORIAMENTE, uma camisa de cor repetida.

    Vamos melhorar!

    As 5 cores da questão: branca, preta, cinza, vermelha e azul

    Cores dos meninos: branca, preta e cinza

    Cores das meninas: vermelha, azul, branca e cinza

    Meninos: branca (2), preta (2) e cinza (1)

    Meninas: vermelha (2), azul (1), branca (1) e cinza (1)

    Total de alunos por cores: branca (3), vermelha (2), preta (2), cinza (2) e azul (1)

    Por isso, podemos concluir que 'a probabilidade de três alunos terem camisetas da mesma cor será igual a 1', ou 100%.

    Item correto.

  • Este gabarito está errado. Digamos 5 cores: A B C D E.
    Verifique que é possível:
    Meninos - AAABC
    Meninas - ABDEE
    Portanto teríamos só 2 pessoas com a cor E! E uma pessoa com a D!

  • Cores: vermelho, preto, azul, branco e rosa = V, P, A, B, R

    Escolhendo o conjunto dos meninos de forma aleatória:

    V - P - A - P - V

    Escolhendo o conjunto das meninas de forma aleatória:

    B - R - A - V - B

     

    Ao escolher  V , sempre tentando a pior hipóte, necessariamente irá repetir pela 3 vez uma camiseta.

     

    Gab Certo

     

  • Se dividir de forma igualitária sim, mas por que não poderia dividir assim nos meninos

     

    XXXXX

     

    Dessa forma a questão estaria errada


ID
280120
Banca
IADES
Órgão
CFA
Ano
2010
Provas
Disciplina
Raciocínio Lógico
Assuntos

Na Copa do Mundo 2010 da FIFA, o Brasil ficou no Grupo G junto com as seleções da Coréia do Norte, da Costa do Marfim e de Portugal. Analisando os resultados de jogos anteriores entre Brasil e Portugal, um torcedor concluiu que a chance do Brasil ganhar é 3 vezes a chance de perder, e que a chance de empatar é metade da chance de o Brasil perder. Para aquele torcedor, a probabilidade de o Brasil perder um jogo contra Portugal é

Alternativas
Comentários
  • digamos que perder é x

    chance de ganhar 3x

    chance de perder x/2

    isso tudo somado é igual a 100%=1

    x+3x+x/2=1 (M.M.C=2)

    2x+6x+x=2 ---->9x=2

    chamei 'perder de 'x', Logo, x=2/9

    espero ter ajudado!


    ;)
  • acredito que o colega tenha se equivocado quanto à chance de empatar. Assim, só pra ajudar na compreensão:

    chance de perder = x
    chance de ganhar = 3x
    chance de empatar = x/2 (metade da chance de perder)

    o resultado do jogo é a somatória das possibilidades, ou seja: perder, empatar e ganhar. Assim

    Chance de perder Chance de ganhar Chance de empatar
    X 3x x/2
    X + 3x + x/2 = 1                                   (agora, faz-se o mmc)
    2x + 6x + x = 2
    9x = 2
    X = 2/9
  • Para não utilizar fração ou mmc , eu fiz o seguinte

    empatar = x
    ganhar brasil = 6x
    ganhar portugual = 2 x

    soma = 9x

    prob ganhar portugual = 2/9

    Gabarito=B
  • O enunciado forneceu: Chance de Ganhar = 3 vezes a chance de perder
                                                Chance de empatar = Chance de perder / 2                  

    Considera-se que em uma partida pode-se Ganhar OU Perder OU Empatar e que as 3 representariam o total dos resultados possiveis (100%);

    OU = Adição
    x= Chance de perder;

    Ganhar OU Perder OU Empatar = 100%
    3x + x + x/2 = 1
    x=2/9

  • não entendi nada.....
  • Roberta, falto morrer de rir de seus comentários, vamos lá!

    em porcentagem, você sempre coloca a igualdade = 100%


    3x (ganhar) + x (perder) + x\2 (empatar) = 1 ou 100%
    tira o MMC

    =2/9
    É difícil escrever em forma de fração aqui, bons estudos!
  • Como o denominador de todas as alternativas é 9, logo só podemos distribuir o comando em 9 casas. Se a chance de ganhar é 3x a de perder e a de empatar é a metade de perder, então teremos seis casas ocupadas por ganhar, duas ocupadas por perder e apenas uma por empatar. Perder ocupa duas casas das nove disponíveis. Letra B.

  • Ganhar = 3X       Empatar = X/2      Perder = X

    Portanto:

     

    3X + X/2 + X = 1

    4X + X2 = 1

    8X/2 + X/2 = 1  (adição de fração de denominadores diferentes. Por isso 4X virou 8X/2. São iguais os valores. Foi só para poder adicionar com a outra fração)

    9X/2 = 1

    9X = 2

    X = 2/9

     

    Opção B)

  • Pra quem não manja muito como eu, suponha qualquer numero para "perder", vai encontrar valores para ''ganhar'' e ''empatar''. Ache a porcentagem para ''perder''...depois compare com as alternativas. Pra mim deu certo, encontrei que a chace do Brasil perder é de 25% e a alternativa mais próxima a isso é a B (cerca de 22%). Espero ter ajudado.

  • Pra quem realmente não tem tanta pratica e prefere tentar um jeito alternativo, coloco o modelo do caro colega @Renato Rodrigues em prática.
    Ganhar - 3x
    Empatar - 1/2x
    Perder  - x

    Atribua um valor qualquer pra X(perder), "10" por exemplo.
    Ganhar(3x) fica 3.10 = 30
    Empatar (1/2 de 10) fica =
    Perder - 10 (conforme escolhemos no exemplo)

    Ganhar + empatar + perder = 100% Logo, 30+5+10 = 100%. Resolvendo 45 = 100%. A partir dai, resolve por regra de três simples mesmo:
    Se 45 ---------100%
         10 ---------   X%                      45x = 1000 -> x = 1000/45 = 22,22%

    Sabendo que as chances de perder equivalem a 22,22%, procuremos nas alternativas a fração que corresponde a essa porcentagem:
    Pra transformar Fração em Porcentagem, basta multiplicar por 100%
    Alternativa A) 1/9 . 100 fica 11,11% (logo, não corresponde)
    Alternativa B) 2/9 . 100 fica 22,22% (Nossa resposta)

    Bons estudos

  • Alguém pode explicar a parte de tirar o MMC?

  • Eu supus que a chance de empatar era 1.

    Logo, a chance de perder (dobro) seria 2.

    Logo, a chance de ganhar (triplo) seria 6.

    Total = 9

    Perder / Total = 2 /9

  • V + E + D = 1

    Se somarmos as probabilidades temos 100% (1)

    V = 3D

    E = D/2

    Substituindo na fórmula:

    3D + D/2 + D = 1

    4D + D/2 = 1

    P = 2/9

    Só chutar e correr para o abraço


ID
283564
Banca
FUNIVERSA
Órgão
IPHAN
Ano
2009
Provas
Disciplina
Raciocínio Lógico
Assuntos

Em um instituto de pesquisa trabalham, entre outros funcionários, 3 físicos, 6 biólogos e 2 matemáticos. Deseja-se formar uma equipe com 4 desses 11 estudiosos, para realizar uma pesquisa. Se essa equipe for composta escolhendo-se os pesquisadores de forma aleatória, a probabilidade de todos os físicos serem escolhidos é um número cujo valor está compreendido entre

Alternativas
Comentários
  • Resposta letra C.

    As possibilidades dos grupos serem formados é:C11,4.

    As possibilidades dos grupos possuindo os 3 físicos é:C8,1 (só há 1 vaga sobrando para os 8 candidatos ñ físicos)

    Probabilidade=(C8,1)/(C11,4)=0,024
  • Existe um  método para não se decorar fórmulas, fazendo como análise combinatória:
    (3/11) x (2/ 10) x (1/9) x (4! / 3!)
    4! - Corresponde ao número de equipes que se  pode formar
    3! - O número de elementos que se repetem (físicos)
  • Casos favoráveis
    ----------------
    Os 3 físicos estarem presentes no grupo dos 4. 

    Temos 11 pessoas para 4 lugares, só que 3 desses 4 lugares já foram preenchidos (por 3 físicos). Ou seja, restam 8 pessoas para ocupar uma vaga. Pois 11 - 3 = 8

    Fís | Fís | Fís | e qualquer um dos outros 8.

    Então fixando o caso em que os 3 físicos estão no grupo, temos 8 casos favoráveis.

    Casos possíveis
    ---------------
    Escolher 4 pessoas num total de 11 ==> C(11,4) = 330

    Portanto: 

    P = Favoráveis / Possíveis = 8 / 330 = 0,02424

  • Bom eu fiz assim :

    o 1º fisico tinha 4 chances em 11.
    o 2º  3 chances em 10( ja que um ja estava lá ).
    0 3º 2 chances em 9.



    multiplicando4/11  X  3/10  X   2/9 = 0,024

ID
305071
Banca
CESPE / CEBRASPE
Órgão
TRT - 16ª REGIÃO (MA)
Ano
2005
Provas
Disciplina
Raciocínio Lógico
Assuntos

Uma moeda é jogada para o alto 10 vezes. Em cada jogada, pode
ocorrer 1 (cara) ou 0 (coroa) e as ocorrências são registradas em
uma seqüência de dez dígitos, como, por exemplo, 0110011010.
Considerando essas informações, julgue os próximos itens.

O número de seqüências nas quais é obtida pelo menos uma cara é inferior a 512.

Alternativas
Comentários
  • me corrija se eu estiver errado:
    N(pelo menos uma cara)=N(total de sequencias)-N(da sequencia que so possui somente coroa)
                                           =210 - 1 = 1023. 
  • Olá.
    ERRADA

    Cada moeda tem 2 faces certo? Então duas possibilidades em cada jogada, uma de ser cara e outra de ser coroa. Jogando-se uma vez haverá 2 possibilidades e jogando-se 10 vezes?

    2 x 2 x 2 x 2 x 2 x 2 x 2 x 2 x 2 x 2
    _   _    _    _    _   _    _   _    _    _

    Então 210=1024

    Assertiva falou que pelo menos uma das possibilidades tem que ser cara:
    Então exclui-se uma possibilidade ficando assim 29 = 512

    A assertiva então seria:

    O número de seqüências nas quais é obtida pelo menos uma cara é IGUAL a 512.
  • Eu creio que o comentário do João esteja certo e o do Franco errado. Nao se pode eliminar uma posição e fazer 2^9. 
    É melhor fazer como no 1o. exemplo, raciocinando por exclusão.
    O inveso, ou complementar, de obter pelo menos uma cara é não obter nenhuma cara. Sabemos que existe uma única possibilidade de não obter nenhuma cara 0000000000. Logo sobram  ([2^10] - 1)= 1023 possibilidades de não obter nenhuma cara.

    Bons estudos! :)
  • A resposta correta é:

    1 x 29   = 512

    Como são 10 lançamentos, mas um deles deve necessariamente ser "cara", temos:

    1 possiblidade (cara) em um dos lançamentos (no primeiro, por exemplo)
    2 possibilidades no segundo lançamento (cara e coroa)
    2 possibilidades no terceiro lançamento (cara e coroa)
    (...) e assim por diante até (...)
    2 possibilidades no décimo lançamento (cara e coroa)

    O que, pelo princípio multiplicativo, nos leva à  1 x 2 x 2 x 2 x 2 x 2 x 2 x 2 x 2 x 2 = 1 x 29   = 512

  • Errada. O cerne para a compreensão desse tipo de questão pode ser encontrado na expressão “PELO MENOS” UM RESULTADO CARA.

    Devemos notar que essa expressão inclui tanto a ocorrência de um único resultado cara como todas as demais ocorrências nas quais se tem AO MENOS UM RESULTADO CARA, quais sejam: 2, 3, 4, 5, 6, 7, 8, 9 ou todos os resultados iguais a cara. Se tivermos cara em todos os lançamentos, isso está de acordo com a expressão, pois “ao menos um resultado será cara”.

    Esse é o macete desse tipo de questão. Como todos os resultados que contenham o evento cara se encontram em harmonia com a expressão “ao menos uma cara”, temos de calcular a soma desses eventos. Sabemos que o total de resultados nos quais não ocorre nenhum resultado cara é igual a 0000000000, que representa uma única possibilidade entre as 1024 possíveis, já que para cada lançamento teremos 2 resultados (cara ou coroa) e teremos 10 lançamentos, ou seja, 210 = 1024.

    Portanto, teremos 1024 – 1 = 1023 possibilidades nas quais ao menos 1 resultado cara será obtido.

    Os amigos acima pecaram por calcular as possibiliades de ocorrência de um único resultado cara, sendo os demais corôa (2^9 = 512), o que é diferente de "ao menos um". Fiquem atentos... Pois se a questão afirmasse que era "igual" a 512, em vez de "inferior", como é de praxie em se tratando de CESPE, muita gente teria errado...Espero ter ajudado... Mas aceito sugestões ou correções... estamos aqui para cooperar... Forte abraço.
  • Não se exclui um número como por exemplo(0000000000), mas se exclui uma possibilidade, pois existem 512 possibilidades com pelo menos uma "CARA". Existiriam 1024 possibilidades com várias "CARAS" e não com uma "CARA" apenas. Valeu!
  • Pessoal,
    no início eu tb pensei q o correto seria 1 x 29   = 512; mas analisando a questão cheguei a conclusão q o mais correto é 210 - 1 = 1023. A questão fala em “pelo menos” que é diferente de “apenas”, ou seja, o que eu quero dizer é pode ocorrer mais de uma cara em uma mesma combinação (a única combinação que não ocorrerá cara é: 0000000000). M corrijam se estiver errado meu pensamento!!!!! VLW!
  • Questão padrão de "pelo menos um"!

    Quando ele fala "pelo menos uma cara", as possibilidades são as seguintes:

    1 CARA -      1000000000
    2 CARAS -   1100000000
    3 CARAS -   1110000000
    4 CARAS -   1111000000
    5 CARAS -   1111100000
    6 CARAS -   1111110000
    7 CARAS -   1111111000
    8 CARAS -   1111111100
    9 CARAS -   1111111110
    10 CARAS - 1111111111

    Ou seja, a única possibilidade que não pode ocorrer é 0000000000 (todas coroa). Portanto a melhor maneira de resolver esse tipo de questão é sempre calculando: ( número total de sequências possíveis) - (a única possibilidade que não pode ocorrer).

    Logo: 2x2x2x2x2x2x2x2x2x2 - 1(todas coroa) = 1024 - 1 = 1023
              
  • Caro colega Matheus, creio que vc se quivoca na sua enumeração de possibilidades. É exatamente esse erro que levou a galera a acreditar que fosse 512 possibilidades e não 1023. o segundo comentário da série está perfeito. Só pra exemplificar, qdo vc lista os resultados com uma cara, coloca apenas um deles. 1000000000 porem há 9 outros que são 0100000000, 0010000000 ...... 

    espero ter ajudado
  • Vamos fazer um teste para saber se é 512 ou 1023 a resposta?
    Considerando um código com 3 dígitos temos as seguintes possibilidades:
    000 100 010 001 110 101 011 111
    Neste caso as possíbilidades seriam 2.2.2=8
    As possíbilidades de ser pelo menos um resultado com cara (1) só não seria o 000 então temos 7 possibilidades.
    Por analogia a resposta correta é 1023, pela razão simples de que apenas 0000000000 não possui caras.

    Como resolver outros tipos de possibilidade? por exemplo pelo menos duas caras.
    Neste caso você vai diminuindo o expoente e subtraindo 1 que é do 0000...
    Pelo menos duas caras seria 512-1=511
    Pelo menos três caras seria 256-1=255

    Valeu... abraço.
  • Pessoal, eu confesso que demorei pra me convencer que o resultado da questão é 1023. Embora mesmo com o resultado 512 eu tivesse acertado a questão... Mas é simples... é soh pensar o seguinte: Jogando a moeda 10x... qual a chance de só cair coroa? ou cara?... Poisé... a resposta é 1 em 1024... certo??? 2 elevado a décima... blá blá blá... 1024... Pois bem... nas outras 1023, pelo menos uma vez aparecerá a face que você não queria... no caso da pergunta a face "Cara"... Taí a resposta da questão... se a assertiva viesse:

    O número de sequencias nas quais é obtida pelo menos uma cara é IGUAL a 512... muita gente teria rodado... inclusive eu

    Espero ter ajudado
  • Tb fiz por permutação com repetição como o colega acima, mas agora estou confusa qto à resolução da questão!!
  • Calma gente, o PELO MENOS UM é assim mesmo, demora pra entrar na cabeça. Quando ler PELO MENOS UM no enunciado já dobre sua atenção pois a CESPE faz MUUUUUIIITO isso.
    Ta certo dizer que o espaço amostral é 1024 (210). Se é PELO MENOS UMA CARA pode ter de uma a dez caras, então o mais fácil é calcular o que ele não pede e diminuir do espaço amostral. Há 1 único modo de se obter todas as combinações CARA, ou seja, TUDO CARA MESMO. Assim 1024 (espaço amostral) - 1 (o que ele não quer) = 1023 maneiras de se obter PELO MENOS UMA CARA.
    A CESPE ama fazer isso, cuidem com o PELO MENOS UM galera, já errei mtas e sigo errando ainda quando perco a atenção.
    BONS ESTUDOS!

  • As vezes a gente quebra a cabeça por não compreender bem o que a questão quer. Passei bem 1 minuto relendo a frase abaixo várias vezes e tentando entender hehe,

    "O número de seqüências nas quais é obtida pelo menos uma cara é inferior a 512."

    O número de sequências onde se obtem ao menos uma cara é o seguinte: TODAS as sequências, MENOS a que não tem nenhuma cara. A partir daí, podemos fazer o 2^10 (2.2.2.2.....) e subtrair de 1. 
  • Nem percam tempo, a resposta correta é a do Felipe. Abraços

  •  

    n(Pelo menos 1 cara) = número de ocorrências  com pelo menos 1 cara ---> 1 cara serve, 2 serve, 3 serve...10 serve.

    n(todas coroa) = 1 possibilidade 

    Todas as possibilidade em 10 lançamentos: 2elevado a 10 = 1024

    Então 1024-1 = 1023 possibilidades de n(Pelo menos 1 cara)

    Gab Errado

  • O Cespe não costuma passar muito longe do resultado do enunciado. Estranho!

  • "O número de seqüências nas quais é obtida pelo menos uma cara é inferior a 512."

    Qual o total de possibilidades? 2^10 = 1024.

    Existe a mesma quantidades de possibilidades nas quais haverá cara ou coroa, certo? 1024/2 = 512 possibilidades em que haverá pelo menos uma cara e 512 possibilidades nas quais haverá pelo menos uma coroa.

    Pronto. Questão respondida. Não é inferior, é exatamente 512 sequências.

  • Usa-se PROBABILIDADE DE UM EVENTO COMPLEMENTAR.

    P(a) = 1- P(b)

    Total de possibilidades = 2^10 = 1024

    Foi pedido pelo menos uma cara, então faço o seguinte:

    calculo justamente o que não foi pedido = SOMENTE COROAS = 1/1024.(existe apenas uma sequência em que todas são coroas) = K K K K K K K K K K

    P(a) = 1- 1/1024

    p(a)= 1023/1024

    1023 sequências possuem pelo menos uma cara.

    Gabarito: E

  • Em nenhum momento o exercício pediu pra calcular a probabilidade de acontecer caras, e sim:"O número de seqüências nas quais é obtida pelo menos uma cara é inferior a 512." Onde que ele pede pra calcular probabilidades aí? O cálculo correto é de combinações de eventos disjuntos.

  • ERRADO

    Utilizando o Triângulo de Pascal tudo fica mais fácil. A questão ficaria muito demorada, pois o enunciado pede PELO menos 1 Cara, teriamos que calcular, 1 Cara, 2 caras, ...., 10 Caras, porém, fica mais fácil usarmos a Análise Destrutiva, como assim ? Usamos as combinações de todas as possibilidades menos a que não pode acontecer, que seria nenhuma cara. Portanto, façamos 2 Elevados na 10, elevado na 10, pois cara pode ser 1, 2, 3 até 10, logo, subtraímos aquilo que não queremos, nenhuma cara, logo, 2 elevado na 0, como qualquer número elevado na 0 da 1 fica assim: 1024 - 1 = 1023 > 512


ID
309160
Banca
CESPE / CEBRASPE
Órgão
TRT - 10ª REGIÃO (DF e TO)
Ano
2004
Provas
Disciplina
Raciocínio Lógico
Assuntos

Um juiz deve analisar 12 processos de reclamações trabalhistas, sendo 4 de médicos, 5 de professores e 3 de bancários. Considere que, inicialmente, o juiz selecione aleatoriamente um grupo de 3 processos para serem analisados.
Com base nessas informações, julgue os itens a seguir.

A probabilidade de que, nesse grupo, todos os processos sejam de bancários é inferior a 0,005.

Alternativas
Comentários
  • A assertiva está correta. Vejamos o porquê. Para resolver a questão basta calcular o seguinte: P(A) = C3,3/C12,3.
    C3,3 = 1* e = C12,3 = 220**. Logo, P(A) = 1/220 = 0,0045. Logo, a probabilidade dos três processos serem os bancários é menor que 0,005.
    * Devemos selecionar as possibilidades de se escolher os três processos bancários dos três retirados por meio de uma combinação.
    ** Devemos selecionar as possibilidades de se escolher os três processos bancários dos doze possíveis.
  • A questão menciona: qual a probalidade de que nesse grupo sejam todos bancários. Afim de responder vamos colocar nomes ao bancário (para facilitar o entendimento)
    Bancário Ayrton (B.A)
    Bancário Bruno (B.B)
    Bancário Cleber. (B.C)
     Quais são as possíveis chanches dele retirar um bancário na primeira puxada de processo? 1 em 12 (pois são 12 processos) e na segunda puxada? 1 em 11 (pois um processo ele já pegou) e o 3° processo 1 em 10 (pois os outros dois processos ele já havia retirado. Então, multiplicamos as possibilidades possíveis (12x11x10=1320) Agora quais são os processos que ele tem que tirar? (são os 3 bancários) Então na primeira puxada pode ser qual quer um dos três (B.A ou B.B ou B.C) logo 1 em 3 na segunda puxada 1 em 2 e na terceira 1 em 1. Logo multiplicamos as possibilidades favoravei (3x2x1=6) Agora montaremos o quebra cabeça. Teremos que dividir a quantidade de possibilidades possíveis pela quantidade de possibilidades de quantidasdes favoráveis (1320/6=220).
     Agora qual será a reposta? 
    A probalilidade do Juiz tirar os 3 bancários puxando apenas três processos é 1 em 200 logo: (1/220=0,0045)
  • Resolução mais simples!
    3/12 x 2/11 x 1/10 = 0,004... < 0,005


  • São 3 eventos.

    Evento 1= Retirar 1 bancário.
    P = (evento favoravel) /(eventos possiveis)
    Temos 3 bancários e 9 não bancários
    P1 = 3 / 12 (este 12 é a soma do 9 com o 3 - total possível)

    Evento 2 = Retirar o segundo bancário
    Lembrando que um componente já foi retirado e que era um bancário.
    Logo, nos restam 2 bancários e os mesmos 9 que não são bancários
    P2 = 2 / (2+9) --- 2/11

    Evento 3 = Retirar o terceiro bancário
    Lembrando que dois componentes já foram retirados e que ambos eram bancários.
    Logo, nos resta 1 bancário e os mesmos 9 que não são bancários
    P3 = 1/10

    Para que o resultado esperado aconteça, todos estes 3 eventos devem acontecer.
    A probabilidade de vários eventos é a multiplicação de suas probabilidades.

    Logo, P = 3/12 x 2/11 x 1/10 = 1/220 = 0,0045 < 0,005
  • Cleber, no final você chegou a conclusão correta e eu entendi seu raciocínio, entretanto ao afirmar que: "Quais são as possíveis chanches dele retirar um bancário na primeira puxada de processo? 1 em 12", ai meu irmão, ta errado né? Seria 3 em 12, depois 2 em 11 e finalmente 1 em 10.
  • Aquele primeiro comentário foi ótimo... meu ajudou perfeitamente a entender a questão... obrigado...
  • 12 Processos, sendo 4 Médicos, 5 Professores e 3 Bancárias, ele forma um grupo de 3 com esses 12.

    Casos Favoráveis / Casos totais

    Casos totais: C12,3 = 220

    Casos favoráveis: C3,3 = 1

    P(x=3 Bancários): 1/220 = 0,0045... < 0,005

  • 3/12 X 2/11 X 1/10 = 6/1320 = 0,004


ID
309163
Banca
CESPE / CEBRASPE
Órgão
TRT - 10ª REGIÃO (DF e TO)
Ano
2004
Provas
Disciplina
Raciocínio Lógico
Assuntos

Um juiz deve analisar 12 processos de reclamações trabalhistas, sendo 4 de médicos, 5 de professores e 3 de bancários. Considere que, inicialmente, o juiz selecione aleatoriamente um grupo de 3 processos para serem analisados.
Com base nessas informações, julgue os itens a seguir.

As chances de que, nesse grupo, pelo menos um dos processos seja de professor é superior a 80%.

Alternativas
Comentários
  • Pelo menos um de professor é evento complementar a "nenhum de professor". Se tirarmos os cinco professores teremos C7,3 maneiras de se escolher o grupo de três processos. C7,3 = 35 (isso é número de maneiras de se escolher os três processos sem nenhum professor).
    O numero total de maneiras de se escolher os 12 processos é dado por C12,3 = 220 (número de casos possíveis, ou seja, número de maneiras de se escolher os três processos dentre todos os de médicos, professores e bancários).
    Agora, o número de maneiras de se escolher os três processos onde pelo menos um seja de professor é igual ao número de maneiras total menos o número de maneiras onde não tenha nenhum professor. 220 - 35 = 185 (número de casos favoráveis).

    Casos favoráveis = 185. Casos possíveis = 220. 185/220 = 0,84. Ou seja, 84%.
  • Maneira que raciocinei:
    Opções possíveis com pelo menos um professor:

    1) 1 professor + 2 não-professores (médicos + bancários - total de 7 processos)
    5 x 7/2 x 6/1 = 105 possibilidades

    2) 2 professores + 1 não-professor (médicos + bancários - total de 7 processos)
    5/2 x 4/1 x 7 = 70
    possibilidades

    3) 3 professores
    5/3 x 4/2 x 2/1 = 10
    possibilidades

    Todas as maneiras possíveis: 12/3 x 11/2 x 10/1 = 220
    possibilidades

    Logo, (105 + 70 + 10)/220 = 0,84 = 84%
  • PESSOAL,
    Com a ajuda dos 2 comentários acima, consegui resolver de uma outra forma, que na verdade é a junção das duas acima:

    pelo menos 1 professor     =  (professores) C5,1 x C7,2 (medicos e bancários) = 105
    pelo menos 2 professores =  (professores) C5,2 x C7,1 (medicos e bancários) = 70
    pelo menos 3 professores =  (professores) C5,3 x C7,0 (medicos e bancários) = 10

    essas são as possibilidades existentes de existir pelo menos 1 (diga-se 1, 2 ou 3) professor nessa seleção ao acaso, que constituem, na probabilidade pedida no enunciado, o nosso evento:
    como é 1 OU 2 OU 3, eu somo as possibilidades = 105+70+10 = 185 possibilidades

    espaço amostral = total de possibilidades incluindo medicos, professores e bancários = C12,3 = 220
    então a probabilidade de ser pelo menos 1 professor ( e não APENAS 1) =    185/220  = 0,85

    Espero ter ajudado, pois fiz questão de tentar postar esse comentário, porque perdi muito tempo quebrando a cabeça pra chegar a esse entendimento.

    NUNCA DESISTIR: 1% INSPIRAÇÃO, 99% TRANSPIRAÇÃO, 100% FÉ!!!
  • SOLUÇÃO A) FONTE: http://br.answers.yahoo.com/question/index?qid=20090228062121AAmSxKb
    2.1. Casos possiveis = C12,3
     
    2.2. Vejamos agora os casos favoráveis:
     
    2.2.1. Casos onde não há processo de professor = C7,3
     
    2.2.2. Casos onde há pelo menos um processo
             de professor (casos favoráveis) =
     
              = C12,3 - C7,3 = 220 - 35 = 185
     
    2.2.3. Pobabilidade do presente caso:
     
              (C12,3 - C7,3) / C12,3 =
     
              = 185 / 220 =
     
              = 0,84090909090909090909090909090909 =
     
              = 84,090909090909090909090909090909%
     
    Esta afirmação é VERDADEIRA.
     
    SOLUÇÃO B) FONTE: http://www.tutorbrasil.com.br/forum/matematica-concursos-publicos/probabilidade-t4963.html

    S
    OLUÇÃO C) FONTE: http://www.forumconcurseiros.com/forum/showthread.php?t=241571

    C.1 - P = 100% - (nenhum seja de Professor) = 1 - (7/12).(6/11).(5/10) = 0,841 > 0,8 (C)

    C.2 - É por exclusão mesmo, mas minha resposta foi diferente. O número de possibilidades em que não haja nenhum professor é a combinação dos outros 7 profissionais, ou seja, C7,3 = 35. Nossa probabilidade é então 1 - 35/220 = 1 - 0,159 = 0,841. O item então se torna Verdadeiro.

    DIVIRTAM-SE.....
  • CERTO - Resolvendo sem fórmulas
    Chance de pelos menos 1 ser professor = Total - Chance de nenhum ser professor

    Chance de nenhum ser de professor (Médicos + Bancários = 7)
    7/12  x  6/11  x  5/10 
    (7 x 6 x 5) / (12 x 11 x 10) - 
    Simplificando:
    (7 x 1 x 1) / (2 x 11 x 2) = 7/44


    Total = 1 (100%) = 44/44

    Chance de pelos menos 1 ser de professor = Total - Chance de nenhum ser de professor
    44/44 - 7/44
    37/44
    Chance de pelos menos 1 ser de professor = 0,84 = 84% > 80%
  • A probabilidade de nenhum processo ser de professor - P(0) - é a seguinte:
    P(0) = 7/12 x 6/11 x 5/10 ≈ 0,16
    A probabilidade de pelo menos um processo ser de professor é P = 1 - P(0):
    P = 1 - 0,16
    P = 0,84

ID
349282
Banca
FUNDEP (Gestão de Concursos)
Órgão
CODIUB
Ano
2010
Provas
Disciplina
Raciocínio Lógico
Assuntos

Suponha que a fiscalização estadual nas empresas de uma cidade seja feita de forma aleatória, com as seguintes probabilidades de ocorrência:

P (nenhuma fiscalização em um ano) = 0,10
P (uma fiscalização em um ano) = 0,20
P (duas fiscalizações em um ano) = 0,30

Nesse caso, a probabilidade de uma empresa dessa cidade ser fiscalizada uma única vez em cada um de três anos consecutivos é de

Alternativas
Comentários
  • Fonte: http://rlm101.blogspot.com.br/

    A probabilidade de uma empresa dessa cidade ser fiscalizada uma única vez em cada um de três anos consecutivos é de:
    = P(uma fiscalização em um ano) E P(uma fiscalização em outro ano) E P(uma fiscalização em um terceiro ano)
    = P(uma fiscalização em um ano) * P(uma fiscalização em um ano) * P(uma fiscalização em um ano)
    = 0,20 * 0,20 * 0,20
    = 0,008

  • neste caso a banca ja te da a probabilidade ai e so multiplicar cada probabilidade ate cpm´letar os 3 anos 0,2x0,2x0,2


ID
361966
Banca
FUNRIO
Órgão
FURP-SP
Ano
2010
Provas
Disciplina
Raciocínio Lógico
Assuntos

João lança dois dados honestos e solicita a Antônio que adivinhe a soma dos resultados nos dois lançamentos. As chances de Antônio acertar serão maiores se ele escolher

Alternativas
Comentários
  • Olá amigos do QC, um dado tem 6 números: 1, 2, 3, 4, 5, 6.
    para podermos visualizar melhor o resultado dessa questão, vamos somar todos os resultados possíveis após ter lançado os dois dados:

    1 = 2, 3, 4, 5, 6, 7

    2 = 3, 4, 5, 6, 7, 8

    3 = 4, 5, 6, 7, 8, 9

    4 = 5, 6, 7, 8, 9, 10

    5 = 6, 7, 8, 9, 10, 11

    6 = 7, 8, 9, 10, 11, 12

    de um lado da igualdade temos todos os possíveis resultado do primeiro dado e do outro lado todos os resultados das somas com os resultados do lançamento do segundo dado.
    obsenvando atentamente o 7 é o único resultado que apareceu em todas possibilidades, sendo portanto o gabarito.
    grande abraço.
  • Batalhador Campo, pode me explicar melhor essa sequência? 

    1 = 2, 3, 4, 5, 6, 7

    2 = 3, 4, 5, 6, 7, 8

    3 = 4, 5, 6, 7, 8, 9

    4 = 5, 6, 7, 8, 9, 10

    5 = 6, 7, 8, 9, 10, 11

    6 = 7, 8, 9, 10, 11, 12


    Por que a contagem sempre começa do número seguinte do número 1 ao 6? Obrigado.

  • Fonte: http://rlm101.blogspot.com.br/

    O resultado mais provável é aquele que possui o maior número de diferentes combinações para chegar ao resultado.
    Por exemplo, só há uma combinação de valores capaz de obter uma soma igual 2, quando ambos os dados são igual a 1, representado pelo conjunto {1,1}.
    Para uma soma igual a 10, há três combinações possíveis {4,6}, {5,5} e {6,4}.
    Para uma soma igual a 6, existem cinco combinações possíveis {1,5}, {2,4}, {3,3}, {4,2} e {5,1}.

    De forma resumida:
    Para somar 2, há uma combinação possível;
    Para somar3, há duas combinações possíveis;
    Para somar 4, há três combinações possíveis;
    Para somar 5, há quatro combinações possíveis;
    Para somar 6, há cinco combinações possíveis;
    Para somar 7, há seis combinações possíveis;
    Para somar 8, há cinco combinações possíveis;
    Para somar 9, há quatro combinações possíveis;
    Para somar10, há três combinações possíveis;
    Para somar 11, há duas combinações possíveis;
    Para somar 12, há uma combinação possível.

    A melhor estratégia é escolher o resultado da soma que apareça com a maior frequência, o sete.

  • Ele quer a maior possibilidade de acertar a soma dos dois dados, logo você soma cada número de 1 dado com todos os números do segundo dado e veja qual aparece mais vezes: 

    Dd 1    dd 2

    1         +  1 = 2

    1         +  2 = 3

    1         +  3 = 4

    1         +  4 = 5

    1         +  5 = 6

    1         +  6 = 7

     Assim sucessivamente   2 + 1 = 3 ... Como já tem a noção da soma inicial, já pode fazer a escada direta colocando so o resultado  das somas, igual o colega Batalhador campo fez, até usar todas as possibilidades: note que o número 7 é o que mais aparece


    1 = 2, 3, 4, 5, 6, 7

    2 = 3, 4, 5, 6, 7, 8

    3 = 4, 5, 6, 7, 8, 9

    4 = 5, 6, 7, 8, 9, 10

    5 = 6, 7, 8, 9, 10, 11

    6 = 7, 8, 9, 10, 11, 12

  • Olá Romullo André, é porque está somando o resultado dos dois dados.

    Exemplo: joguei o primeiro dado e deu 1, os possíveis resultados para o segundo dado são 1, 2, 3, 4, 5, 6

     

    somando agora o 1 com os valores possíveis encontraremos: 2, 3, 4, 5, 6, 7  seguindo esse raciocínio faremos o mesmo quando o primeiro dado der 2,

    teremos: 3, 4, 5, 6, 7, 8 e assim até a última sequência.

     

    fechando todas as possibilidades. Espero que tenha solucionado a dúvida.

     

    Grande abraço e bons estudos

     


ID
425020
Banca
UFBA
Órgão
UFBA
Ano
2009
Provas
Disciplina
Raciocínio Lógico
Assuntos

A probabilidade total de falha de um sistema que possui N componentes distintos e independentes é dada pelo produto das probabilidades de falha desses componentes.

Alternativas
Comentários
  • Questãozinha perigosa porque parece muito óbvia... 

    Pelo enunciado "N componentes distintos E independentes" já dá uma vontade de usar a fórmula P (A^B) = p (A). p (B)                       (^ = interseção )

    Mas a ideia na verdade é que as peças não dependem umas das outras.
    Imagine a tal máquina com N=2 por exemplo, se 1 quebrar a máqquina para, OU se a 2 quebrar a máquina para.
    Logo, temos que a probabilidade da máquina parar é a  UNIÃO das probabilidades de cada peça, assim é a SOMA de todas as probabilidades.
    Lembra: p (AUB) = p (A) + p (B) - p (A ^ B)            p (A ^ B) = 0 pois as peças são independentes!

    Boa sorte e bons estudos para nós!

  • Giovana, excelente sua explciacao mas matei essa questao em 5 segundos sem precisar de tanta teoria e vou divulgar ok?
    Nesse caso, a falha no sistema depende apenas da falha de um componente. Ou seja se qualquer um dos N componentes falhar, o sistema ja falhou, por isso nao seria produto das probabilidades e sim a soma da probabildiade de falha de qualquer um dos N componentes. por isso marquei errado.
    Bons Estudos
  • questao ambígua..eu raciocinei como voces, porém ao reler o enunciado "probabilidade total de falha de um sistema " achei q esse TOTAL se referia justamente a q o sistema falharia apenas se o TOTAL das peças falharem.. aí marquei correto, e errei a questao


ID
441847
Banca
FGV
Órgão
SAD-PE
Ano
2009
Provas
Disciplina
Raciocínio Lógico
Assuntos

16 funcionários de uma empresa, entre eles Pedro e Paula, que são casados, vão ser divididos por sorteio em quatro grupos de quatro pessoas e, cada grupo vai analisar um aspecto da gestão da empresa.

A probabilidade de que Pedro e Paula caiam no mesmo grupo é de:

Alternativas
Comentários
  • Fonte: http://rlm101.blogspot.com.br/

     

    Em grupo de quatro pessoas, a probabilidade de que Pedro e Paula estejam juntos é igual a:
    P(Pedro) E P(Paula) E P(Outro) E P(Outro) = 1/16 * 1/15 * 14/14 * 13/13 = 1/240.


    No cálculo acima, a ordem foi levada em consideração, mas pode haver diferentes sequências de seleção, tais quais: (Paula, outro, Pedro, outro) ou (Outro, Pedro, Paula, Outro). O total é igual a uma permutação entre quatro casas com dois elementos repetidos.


    Pr(4) = 4! / 2! = (4*3*2) / 2 = 12


    Logo, P(Pedro e Paula) = 12 * (1/240) = 1/20


    Finalmente, isso pode ocorrer em qualquer um dos 4 grupos que serão formados.
    Há ao todo 4/20 ou 20% de chance do casal ficar junto.


  • Essa sai assim:

    Quantidades de grupos possíveis formados por 4 pessoas a partir de 16 funcionários.
    C16,4 = 16! / 4!*12! = 1820.

    Seja um grupo qualquer onde Pedro e Paula estão juntos, logo só falta preencher duas vagas neste grupo. Então, temos que:

    C14,2 = 14! /2!*12! = 91.

    Mas, sabemos que existem 4 grupos, sendo que Pedro e Paula podem estar em qualquer um deles. Logo, temos:

    4*91 = 364.

    P( probabilidade ) = Resultados Favoráveis / Resultados possíveis 
    P = 364 / 1820 = 1/5 = 0,2. 
    P = 20%.

    Gaba-> D.

     



  • Alguém poderia me explicar por que se usou, no cálculo do espaço amostral, apenas C(16,4), e não C(16,4) x C(12,4) x C(8,4) x C(4,4) ??

  • LETRA D

    A ordem não importa = combinação

    16 funcionários

    4 grupos de pessoas

    A probabilidade de que Pedro e Paula caiam no mesmo grupo

    PE . PA . __ . __

    Totalidade de funcionários:

    C16,4= 1820

    Combinações tendo Pedro e Paula juntos:

    C14,2=91 (em apenas 1 grupo)

    Nos 4 grupos = 91 x 4 = 364

    364/1820 = 20%


ID
452227
Banca
CESPE / CEBRASPE
Órgão
PC-TO
Ano
2008
Provas
Disciplina
Raciocínio Lógico
Assuntos

Julgue os itens seguintes, que dizem respeito à determinação do
número de possibilidades lógicas ou probabilidade de algum
evento.

Considere que 9 rapazes e 6 moças, sendo 3 delas adolescentes, se envolvam em um tumulto e sejam detidos para interrogatório. Se a primeira pessoa chamada para ser interrogada for escolhida aleatoriamente, então a probabilidades de essa pessoa ser uma moça adolescente é igual a 0,2.

Alternativas
Comentários
  • Olá
    Gabarito C

    Questão de graça. Vamos lá.

    O total de pessoas são 15;
    3 das 6 moças são adolescentes.

    Qual a probabilidade de se interrogar uma moça adolescente?

    3/15 = 0,2
    Possíveis/Total de pessoas = Probabilidade
  • Gostaria de saber como eu faço a divisão 3/15 sem usar calculadora.


    Bons estudos!!!!!!!!!!!!!!!!!
  • Lucas, vamos lá -  3  dividido por 15
    quando o dividendo é menor que o divisor você pode acrescentar um zero (0) assim fica 30 dividido por 15, como foi acrescido de um zero, o produto  você move a virgula para a esquerda assim - como 30 dividio por 15 =  2  -  movendo a virgula para  a esquerda temos  0,2. bons estudos.

  • Eu gosto de fazer este tipo de questão por meio da porcentagem:

    15 é o total de possibilidades 
     
    9 Homes = 60% ( 9 x 100 / 15 = 60)

    6 Mulheres = 30% ( 6 x 100 / 15 = 30)

    3 Adolecentes = 20%  ou seja 0,2 ( 3 x 100 / 15 = 30) 

    Asssim se a questão pedisse a probabilidade de o escolhido ser homen seria 0,6 ou 60%. 

    A probabilidade é : casos favoráveis / casos possíveis 
    casos favoraveis 3 ( três meninas adolescentes) 
    casos possiveis 15 ( número total de pessoas) 

    É o mesmo que dizer:  o que é o numero 3 dentro do numero 15. 

    Por isso em resolvo com porcentagem!!!!



          
  • O comentário do franco está certissímo.

    São 15 pessoas dentre as quais 3 são adolescentes.

    Probabilidade = casos favoraveis (desejáveis) =    3   = 0,2
                                        casos possíveis                  15


    Resultado = letra c)
  • e = 3 (adolescentes)
    U = 6 + 9 = 15

    p(e) = n(e) / n(u)

    onde:

    P(e) = probabilidade do evento
    n(e) = número de eventos
    n(u) = número do universo

    p(e) = n(e) / n(u)
    p(e) = 3 / 15
    p(e) 0,2

    Resposta:  certo
  • 3 das 15 pessoas são moças adolescentes
    3/15 = 0,2
    Que é a propabilidade de uma delas ser chamada.
  • 15 pessoas no total, sendo:      9 homens
                                                      6 moças, dessas 3 são adolescentes.


    a probabilidade de ser uma moça-adolescente é: 3/15 = 1/5 = 0,2
    QUESTÃO CORRETA!
  • Por mais questões assim!!!

  • Eu fiz assim não sei se esta correto:

    6(3*2*1)

    C = 6*5*4= 120/6 = 20%

    GABARITO = CERTO

  • Temos ao todo 15 pessoas, das quais 3 são moças adolescentes. A probabilidade de uma delas ser escolhida é P = 3/15 = 1/5 = 0,2. Item CERTO

  • sdds dos tempos que até uma prova de DELEGADO era nesse nível fundamental kkkk

    o que a crise não faz né...

  • onde eu tava em 2008

  • Probabilidade de ser MOÇA = 6/15

    Probabilidade de ser ADOLESCENTE= 3/6

    Logo a probabilidade de ser moça E adolescente = 6/15 * 3/6 = 3/15 = 1/5 = 0,2

  • 15 Pessoas quero uma: C 15,1 = 15

    3 Meninas Adolecentes quero uma: C3,1=3

    3/15 = 0.2

  • GAB. CERTO

    O total de pessoas são 15;

    3 das 6 moças são adolescentes.

    Qual a probabilidade de se interrogar uma moça adolescente?

    3/15 = 0,2

    Possíveis/Total de pessoas = Probabilidade


ID
486868
Banca
CESPE / CEBRASPE
Órgão
MPE-RR
Ano
2008
Provas
Disciplina
Raciocínio Lógico
Assuntos

Em cada um dos próximos itens, é apresentada uma situação hipotética a
respeito de probabilidade e contagem, seguida de uma assertiva a ser
julgada.

O arquivo de um tribunal contém 100 processos, distribuídos entre as seguintes áreas: direito penal, 30; direito civil, 30; direito trabalhista, 30; direito tributário e direito agrário, 10. Nessa situação, ao se retirar, um a um, os processos desse arquivo, sem se verificar a que área se referem, para se ter a certeza de que, entre os processos retirados do arquivo, 10 se refiram a uma mesma área, será necessário que se retirem pelo menos 45 processos.

Alternativas
Comentários
  • Existem:

    direito penal,= 30
    direito civil =  30;
    direito trabalhista = 30
    direito tributário + direito agrário = 10

    Para se ter certeza

    9 + 9 + 9 + 10
    * = 37

    * =(como são dois áreas do direito juntos, repare que nenhum deles sozinho terá 10, haja visto que deve haver pelo menos um da outra área)

    Logo, questão errada.
  • ERRADA

    Questão sobre Princípio da casa dos pombos.

    Na pior hipótese possível, serão retirados:
    9 processos de direito penal,
    9 processos de direito civil,
    9 processos de direito trabalhista,
    9 processos de direito tributário e agrário.
    O próximo a ser retirado com certeza completará os 10 processos pedidos na questão.
    Ou seja, será necessário que se retirem pelo menos 37 processos.
  • Princípio da Casa de Pombos

    Se existirem pelo menos K+1 pombos, e somente K casas, pelo menos uma casa vai ter mais do que um pombo.


    Resolução

    São 100 processos e 4  "caixas": direito penal, direito civil, direito trabalhista, direito tributário e direito agrário.

    Note que as casas tem um limite máximo! Direito penal, 30; direito civil, 30; direito trabalhista, 30; direito tributário e direito agrário,10.

    A questão quer saber quantos processos precisarei puxar, no mínimo, para que dez processos sejam da mesma área.

    Imaginemos o pior.

    Eu comecei puxando um processo após o outro e os nove primeiros processos eram de direito penal, bom falta um! Aff! O décimo foi de "civil".
    Depois continuei puxando um após outro e vieram mais oito de "civil". Totalizando nove de "civil". Só um!!! quem sabe agora?
    Um e outro e foram dessa vez  mais nove de "trabalhista" e depois mais nove de "tributário e agrário". O que isso significa? 

    Significa que o próximo que eu puxar completará o décimo processo de qualquer área!

    O total que puxe foi 36, o trigésimo sétimo completará a dezena!


    Boa Sorte!
       

  • Para dizer a verdade, nem conhecia essa teoria dos pombos, mas a questão é facil de resolver:
    30 livros são de uma área em 100 livros ou seja
    30% deles é de uma área = 30/100 = 3/10
    ou seja, de cada 10 livros que tirar 3 serão de uma mesma matéria.
    Para eu tirar os dez livros então vou precisar de aproximadamente 35 livros que me darão aproximadamente 10,5 livros da mesma matéria.

    Agora vamos supor que uma matéria tivesse 45 livros.
    45/100= 4,5/10
    para tirar dez livro de uma mesma máteria precisaria tirar 22 livros.
  • meu deus, nao entendi nada.
    por que tenho que tirar 9 depois 9 depois 9 depois mais 9 ?
    ele pergunta ´´pelo menos´´ por que nao posso tirar logo 10 da primeira caixa? ou porque nao posso tirar 9 e dpois tirar 10 , totalizando 19?
  • Rafael, as escolhas são sem verificar de que área são, isso quer dizer mais ou menos o seguinte:
    Imagine que você está num escuro total. Precisa pegar 10 processos de qualquer área. Como você não consegue ver vai tirando sem saber qual é. Então imagine que está num dia de grande azar e você tirou 9 de direito penal mas depois começou a pegar só os de direito civil e tirou também 9, o azar continuou e você começou a tirar os de direito trabalhista, tirou 9 e depois tirou mais 9 de direito tributario e agrario. Bom , até aí você já retirou (4x9=36) 36 processos e não conseguiu completar 10 de nenhum. Agora, por mais azarado que você seja , com certeza com o próximo você vai completar 10 de alguma das áreas. Então com 37 processos retirados teremos CERTEZA que 10 são de uma área qualquer, só não podemos afirmar qual.
    Espero ter ajudado.
  • Está é uma situação em que não tem como se GARANTIR o resultado. O problema poderia trazer qualquer valor no lugar de 10 que mesmo assim a questão estaria ERRADA. Ex.: Se eu jogar uma moeda UM MILHÃO DE VEZES, pelo menos uma vez cairá coroa? Não, eu não tenho como garantir esse resultado. É provável que aconteça mas não há garantia alguma.

  • Galera não sabe diferenciar Probabilidade de  Princípio da casa dos pombos.

     

    Por isso sai errando ai e não entendem os comentários

     

  • ERRADO

     

    37

  • O macete desse tipo de questão é sempre pensar na pior das hipóteses. Segue a explicação da Cristiana, explicou certinho.

  • Questão sobre Princípio da casa dos pombos.

    Na pior hipótese possível, serão retirados:

    9 processos de direito penal,

    9 processos de direito civil,

    9 processos de direito trabalhista,

    9 processos de direito tributário

    9 processos de direito agrário.

    O próximo a ser retirado com certeza completará os 10 processos pedidos na questão.

    Ou seja, será necessário que se retirem pelo menos 46 processos, na pior das hipóteses se a pessoa tirar 45, não poderá garantir que tem 10 iguais.

    ERRADO

  • Gabarito: Errado.

    Atenção: O comentário do Luis Fernando Chaves está equivocado.

    É uma questão do Teorema de Dirichlet, conhecido como princípio da casa dos pombos. Nesse tipo de questão, nós temos que analisar e pensar no pior cenário possível.

    No contexto da questão, nós precisamos tirar 10 processos de um mesmo tema.

    Qual a pior forma de fazer isso? Começamos a retirada, retiramos um de civil, um trabalhista, um tributário e um agrário. Não precisa ser nessa ordem, mas a pior forma de se obter é tirando sempre um processo de tema diferente a cada retirada.

    Se fizermos isso 9 vezes, nós teremos retirados já 36 processos (9 x 4 processos). Só que nós teremos 9 processosde cada tipo, então a próxima retirada, independente de qual seja o tipo de processo, vai satisfazer a condição de 10 processos do mesmo tipo.

    Então, serão pelo menos 37 processos.

    Bons estudos!


ID
486871
Banca
CESPE / CEBRASPE
Órgão
MPE-RR
Ano
2008
Provas
Disciplina
Raciocínio Lógico
Assuntos

Em cada um dos próximos itens, é apresentada uma situação hipotética a
respeito de probabilidade e contagem, seguida de uma assertiva a ser
julgada.

Em uma urna há 100 bolas numeradas de 1 a 100. Nesse caso, a probabilidade de se retirar uma bola cuja numeração seja um múltiplo de 10 ou de 25 será inferior a 0,13.

Alternativas
Comentários
  • CERTO

    Múltiplos de 10: {10, 20, 30, 40, 50, 60, 70, 80, 90, 100}
    Múltiplos de 25:        {25,          50,         75,           100}

    12 números favoráveis em 100 números possíveis = 0,12
    Logo, menor que 0,13.
     

  • Esse "ou" significa adição?! Alguém sabe outro modo de fazer, não consegui entender bem.

    Obrigado.



  • Probabilidade  =   n° de casos favoráveis  12    =   0,12
                              n° de casos possíveis      100
  • Luccas,

    Na verdade o que os colegas fizeram foi aplicação direta da fórmula:

    P(A U B) = P(A) + P(B) - P(A ∩ B)

    P(A U B) = 10/100 + 4/100 - 2/100
                    = 12/100
                    = 0,12

    Bons estudos!
  • P(10) +P(25) - P(10  25) = 10/100 + 4/100 - 2/100 = 12/100 = 0,12
  • Alguém pode explicar?
  • Cuidado para não calcular os eventos que estão na união dos dois. (50 e 100) senão a probavilidade vai dar 0,14 e vc vai errar que nem eu.
  • Resolvendo a questão:
    Multiplos de 10 ou 25
    Quantos são os multiplos de 10? é só dividir 100 por 10 = 10
    Quantos são os multiplos de 25? 100/25 = 4
    Existe algum número que é multiplo de 10 e 25 ao mesmo tempo? é só tirar o mmc = 50, depois descobrir quantos são 100/50=2
    Por que excluir o número de multiplos comuns? porque se não excluirmos eles serão contados duas vezes.
    Entre 1 e 100 os números 50 e 100 são multiplos tanto de 10 como de 25 e devem ser contados só uma vez.
    Ou seja teoriamente temos 10 multiplos de 10 e 4 multiplos de 25 = 14 menos 2 que são os multiplos comuns = 12.
    12/100 = 0,12
    A probabilidade de se tirar um número multiplo de 10 ou de 25, neste caso, é de 0,12 ou 12% que é um número inferior (menor) que 0,13 ou 13%.
  • Boa tarde galera, entendo que não devemos contabilizar o 50 e o 100 duas vezes... afinal já estão incluidos nas dezenas e o enunciado não pede multiplos de 10 E  de 25, ele pede 10 OU 25.. então tanto faz se o 50 eh multiplo de 10 ou de 25.. certo?

    Nesse caso daria certo mas poderiam colocar um percentual diferente e aí daria errado.
    Concordam?
  • A = múltiplo de 10 -> {10, 20, 30, 40, 50, 60, 70, 80, 90, 100}. -> 10 possibilidades de sucesso
    B = múltiplo de 25 -> {25, 50, 75, 100} -> 4 possiblidades de sucesso
    múltiplos de A e B -> {50, 100} -> 2 possibilidades de sucesso
    Fórmula: P(A ou B) = P(A) + P(B) - P(A e B)
    Substituindo: P(A ou B) = 10/100 + 4/100 - 2/100 = 12/100 = 0,12
  • Galera:
     A chance de acertar na primeira tentativa é de 1 em 10, ou seja 1/10.
    Para que se tenha uma segunda tentativa, tem-se que ter errado a primeira(chance de erro na primeira tentativa 9/10). O que terá ocorrido no momento da segunda tentativa? O erro da primeira tentativa(9/10),  e a segunda tentativa(1/9).
    Multiplicando a chance de erro na primeira tentativa(por que só terei uma segunda tentativa se tiver errado a primeira) pela chance da segunda tentativa( 1 chance em 9), temos:  9/10X1/9=9/90. simplificando 1/10.
    Portanto igual à chance de acertar na primeira tentativa.


    Eu tambem errei esta maldita questão!!!!!!!
  • Que questãozinha sacana! 

  • GABARITO CORRETO

    Múltiplos de 10: {10,20,30,40,50,60,70,80,90,100}.

    Múltiplos de 25: {25,50,75,100}

    Percebam que os números 50 e 100 se repetiram, então retira-se os 2 de algum dos múltiplos, pois as bolas na urna não se repetem.

    Sobraram 12 possibilidades, que é igual a 12/100 = 0,12%.

    "Se não puder se destacar pelo talento, vença pelo esforço"

  • múltiplos de 25 que estão de 1 a 100 = 4

    múltiplos de 10 que estão de 1 a 100 = 10

    múltiplos de 10 OU 25 (AUB)

    AUB = A + B - AeB (múltiplos de 10 E 25 que estão de 1 a 100)

    AUB = 10 + 4 - 2 (50 e 100)

    AUB = 12

    P(10ou25) = 12/100 < 13/100

    GAB: C


ID
486874
Banca
CESPE / CEBRASPE
Órgão
MPE-RR
Ano
2008
Provas
Disciplina
Raciocínio Lógico
Assuntos

Em cada um dos próximos itens, é apresentada uma situação hipotética a
respeito de probabilidade e contagem, seguida de uma assertiva a ser
julgada.

Um dado não viciado é lançado duas vezes. Nesse caso, a probabilidade de se ter um número par no primeiro lançamento e um número múltiplo de 3 no segundo lançamento é igual a 1⁄6.

Alternativas
Comentários
  • CERTO
    P = par no 1º lançamento E múltiplo de 3 no 2º lançamento
    P = 3/6 X 2/6
    P = 1/6
     

  • 1° Dado (números pares) : 1     2     3   4   5  6        Logo 3/6
    2° Dado ( múltiplos de 3) :  1   2   3  4 5  6 Logo 2/6

    Pelo Princípio Fundamental da Contagem, se os fatos ocorrem um após o outro, deve multiplicá-los:

          3     
    X        2       =    6           Simplificando por 6 =   1  
          6                6           36                                      6
  • Olá,

    O espaço amostral do dado é 6

    1 2 3 4 5 6

    No Primeiro Lançamento, pede-se a probabilidade de um número par. Temos 3 números pares, então a probabilidade é de
     3 
     6

    No Segundo Lançamento, pede-se a probabilidade de um número múltiplo de 3, temos 2 possibilidades (3 e 6) então a probabilidade é de
     2 
     6

    Multiplicando as probabilidades chegamos ao valor de
     6 
    36

    Simplificado dá exatamente o valor explicitado no enunciado da questão.
                                                                                            

    Alternativa Correta.

    Até mais.
  • Galera tenho outra forma de resolução:
    Espaço amostral (S) = 36 elementos.
    Número de eventos favoráveis = 6 são eles: {(2,3); (2,6);(4,3);(4,6);(6,3);(6,6)}
    basta jogar na fórmula:
    P(A) = número de eventos favoráveis / número de eventos possíveis =
    P(A) = 6/36
    P(A) = 1/6.
    Acho dessa forma muito mais fácil, o que acham?
  • TEM UMA FORMA MAIS SIMPLES..

    1º DADO= ELE QUER 1 Nº PAR , OU SEJA,UM DADO TEM QUANTOS NUMEROS PARES? 3   OU SEJA..( 2,4,6)

    LOGO 1/3

    2º DADO= ELE QUER UM Nº MULT.DE 3 ,QUANTOS Nº MULT DE 3 TEMOS NUM DADO? 2    OU SEJA..(3,6)

    LOGO 1/2

    RESOLVENDO: 1/3 X 1/2 = 1/6

    GAB.CERTO!

     

  • GAB CORRETO

    RESUMO :

    PRIMEIRO LANÇAMENTO NÚMERO PAR

    1 2 3 4 5 6

    3/6

    SEGUNDO LANÇAMENTO MULTIPLO DE 3

    1 3 4 5 6

    2/6

    3/6 X 2/6 = 6/6

    SIMPLIFICANDO 1/6

  • GABARITO: C

    Resolução em vídeo: http://sketchtoy.com/69888310


ID
494044
Banca
FUMARC
Órgão
BDMG
Ano
2011
Provas
Disciplina
Raciocínio Lógico
Assuntos

Quando se jogam dois dados, tanto o número 6 quanto o número 7, por exemplo, podem ser obtidos de três maneiras distintas:

(5,1), (4,2), (3,3) para o 6; e

(6,1), (5,2), (4,3) para o 7.

Segundo Galileu, porém, na prática, a chance de se obter 6 é menor que a de se obter 7, porque as permutações dos pares devem ser consideradas no cálculo das probabilidades.

Considerando-se esse raciocínio, é correto afirmar que, nesse caso, a probabilidade de se obter o número 6 e a probabilidade de se obter o número 7 são, RESPECTIVAMENTE, de

Alternativas
Comentários
  • São 36 combinações possíveis:
    (1,1), (1,2), (1,3) (1,4), (1,5), (1,6), (2,1), (2,2), (2,3) (2,4), (2,5), (2,6), (3,1), (3,2), (3,3) (3,4), (3,5), (3,6), (4,1), (4,2), (4,3) (4,4), (4,5), (4,6), (5,1), (5,2), (5,3) (5,4), (5,5), (5,6), (6,1), (6,2), (6,3) (6,4), (6,5), (6,6), 

    Para se ter a soma 6, nos temos 5 combinações: (5,1), (4,2), (3,3) e  (1,5), (2,4)               = 5/36
    Para se ter a soma 7, nos temos 6 combinações: (6,1), (5,2), (4,3) e  (1,6), (2,5), (3,4)    = 1/6

    Também podemos simplesmente dobrar a quantidade de opções e excluir as que sejam iguais:
    Com 7: (6,1), (5,2), (4,3) = 3 vezes. 3 x 2 = 6 em 36 possibilidades = 1/6
    Com 6: (5,1), (4,2), (3,3) =  3 vezes. 3 x 2 = 6 - 1 que se repete, em 36 possibilidades = 5/36
  • Na prova eu errei, mas agora repensando, a questão é bem mais fácil do que eu pensava - basta saber um pouquinho sobre probabilidade com dados.

    Resposta = c

    Primeiro, calculando para o 7.
    A questao informa que para sair 7, ou é (6,1) ou (5,2) ou (3,4).
    Os dados podem ser lançados juntos ou não, tanto faz, mas para análise, vou verificar o resultado em separado de cada um deles.
    No primeiro dado, as possibilidades que me interessam são 6 em 6 (qualquer número entre 1-6) .
    Já no segundo dado, só me interessa uma possibilidade em seis (pq se já saiu 1 no primeiro dado, só me interessa o 6 no segundo (e vice-versa). Da mesma forma, se no primeiro dado saiu 2, só me interessa o 5 no segundo. E se saiu 4 no primeiro dado, só me interessa o 3.

    Assim, a possilidade de sair 7 nos dois dados  é: 6/6 * 1/6 = 6/36 = 1/6.

    Agora para o 6
    Para sair 6, eu preciso de ou (5,1) ou  (4,2) ou (3,3).
    Assim, no primeiro dado, eu preciso de ou 1, ou 2 ou 3 ou 4 ou 5 - ou seja, apenas cinco números me servem. Ou seja: 5/6
    Já no segundo dado, se saiu 1, só me interessa o 5 (e vice-versa), se saiu o 4 , só me interessa o 2 (e vice-versa) e se saiu o 3 só me interessa outro 3. Ou seja, uma possibilidade em 6.
    Assim, a possibilidade de sair 6 nos dois dados é: 5/6 * 1/6 = 5/36


  • 1/18 = 2/36
    1/12 = 3/36

    1/12 = 1/12
    1/12 = 1/12

    5/36 = 5/36
    1/36 = 1/36

    1/3 =  12/36
    1/2 = 18/56

    5/36    é maior que    1/36. Simples assim.
  • Gente, tem que simplificar, pois muitas pessoas possuem dificuldade para entender a questão.

    Vamos lá: Ao se jogar dois dados os casos possíveis são 36, pois cada um dos dois dado tem 6 números. Logo, 6 x 6 = 36.  Até aqui conseguimos achar os casos possíveis.
    Vamos aos casos favoráveis do número 6:
    (1,5) (2,4) (3,3) (4,2) (5,1) , estes são os números que somando dá seis. Logo, percebemos que há cinco caso. Dividindo os casos favoráveis  pelos casos possíveis temos: 5/36. Já achamos a probabilidade do 6.
    Agora vamos aos casos favoráveis do número 7:
    (5,2) (6,1) (1,6) (2,5) (4,3) (3,2), estes são os números que somando dá sete. Logo, percebemos que há seis casos. Calculando o casos favoráveis pelos casos possíveis temos: 6/36, simplificando por 6 temos 1/6.
    Achamos a resposta: letra C.




  • É só uma questão comum de probabilidade,  mas por causa da frase abaixo

    Segundo Galileu, porém, na prática, a chance de se obter 6 é menor que a de se obter 7, porque as permutações dos pares devem ser consideradas no cálculo das probabilidades.

    Achei que seria algo muito místico e difícil!
  • Considerando a soma 6:   5/6 * 1/6 = 5/36

    Considerando a soma 7 : 6/6 * 1/6= 1/6

    Letra C

  • A explicação de Núbia foi a melhor e mais simples. Obrigada!


ID
494050
Banca
FUMARC
Órgão
BDMG
Ano
2011
Provas
Disciplina
Raciocínio Lógico
Assuntos

Dois jogadores fizeram uma aposta em um lance de dados. Cada um lança o dado, aquele que obtiver o maior número será o vencedor. Evidentemente, há possibilidade de empate, quando os dois lançamentos mostrarem o mesmo número.

É CORRETO afirmar que a possibilidade de ocorrer um empate é de:

Alternativas
Comentários
  • São 36 combinações possíveis:
    (1,1), (1,2), (1,3) (1,4), (1,5), (1,6), (2,1), (2,2), (2,3) (2,4), (2,5), (2,6), (3,1), (3,2), (3,3) (3,4), (3,5), (3,6), (4,1), (4,2), (4,3) (4,4), (4,5), (4,6), (5,1), (5,2), (5,3) (5,4), (5,5), (5,6), (6,1), (6,2), (6,3) (6,4), (6,5), (6,6).

    Dessas 6 são iguais.
    6/36 ----> 1/6
  • CADA JOGADOR TEM 1/6 DE CHANCE  OS DOIS TEM A MESMA CHANCE DE TIRAR
    O MESMO NUMERO EM CONDIÇOES IGUAIS.
  • soma a quantidade de jogadas dos dados que são 36. Dimiua os números pares que são 6. Divida 6 por 36.
  • Os jogadores possuem 6 opções de tirarem o mesmo número os dois, dentre
    36 possibilidades aleatórias de aparecer qualquer número nos dados.

    Então há a possibilidade de 6 empates em 36 combinações diversas.


    alinemoraiss.blogspot.com
  • Quando se dará o empate? Quando o jogador A tirar 1 e o jogador B tirar 1 - Empate. No entanto, isso pode acontecer 6 vezes, afinal o dado só tem 6 numeros. AS combinações dos resultados possíveis são 36, logo a probabilidade será 6/36 = 1/6.
  • O empate só ocorrerá se o 2º jogador obtiver o mesmo resultado que o 1º jogador, qualquer que ele seja.
    Sendo assim nao é necessário impor condições para o resultado do 1º jogador, apenas para o 2º.

    Supondo que o 1º lançou um 3, só há uma possibilidade de dar empate, se o 2º tambem lançar um 3.
    Como o espaço amostral de um unico dado tem 6 elementos, e apenas uma favoravel (no exemplo o 3)
    entao fica fácil encontrar a solucao 1/6.

    Bons estudos
  • Para o empate, tanto faz qual número o primeiro jogador tire, mas o segundo terá que tirar o mesmo que o primeiro.

    Assim, no primeiro lance, como qualquer número me serve, a probabilidade é de 6/6.

    Já no caso do segundo jogador, só um número me serve - aquele que o primeiro jogador tirou. Assim, a probabilidade é de 1/6.

    Reunindo os dois - lembrar que em probabilidade "e" = multiplicação:
    6/6 * 1/6 = 6/36 = 1/6

    Resposta = letra B
  • Para um dado de 6 faces cada jogador dispõe da fração de 1/6 para cair cada face com o valor correspondente.
  • Como cada um lançará um dado, serão dois dados em jogo. Assim, cada dado tem 6 lados, portanto 6 x 6 = 36 possibilidades.

    Para haver possibilidade de empate, os dois lançamentos deverão mostrar o mesmo número.

    Então, os resultados favoráveis seriam:

    1,1 - 2,2 - 3,3 - 4,4 - 5,5 - 6,6 = Portanto serão 6 resultados favoráveis, dentro de 36 possibilidades.

    Assim: 6/36 = 1/6

    Fonte: 
    http://www.youtube.com/watch?v=6PY36eBOKto
  • b-

    se 1 tirar 6, o outro tera 1/6 de chances de empatar.

  • Se possível, em outras questões que vier comentar, guarda a parte do "bem fácil", só pra vc.


ID
496351
Banca
CESPE / CEBRASPE
Órgão
TJ-DFT
Ano
2008
Provas
Disciplina
Raciocínio Lógico
Assuntos

    Em 2007, os brasilienses passaram mais cheques sem fundo do que a média dos brasileiros. Em cada mil folhas de cheque dadas pelo pagamento de algum bem ou serviço, 30 foram devolvidas na capital federal, enquanto, na média nacional, o volume cai para 20 a cada mil. Ao longo de 2007, no Brasil, foram 30 milhões de devoluções, 6 milhões a menos do que em 2006. Em 2006, a inadimplência era de 21 a cada mil folhas.
Correio Braziliense, 24/1/2008 (com adaptações).

A partir das informações apresentadas acima, julgue o item subseqüente.

Considere-se que, no ano de 2007, uma pessoa tenha recebido, de brasilienses, em três dias consecutivos, três folhas de cheque pelo pagamento de algum bem ou serviço. Nessa situação hipotética, a probabilidade de que exatamente uma dessas três folhas tenha sido devolvida será superior a 0,08 e inferior a 0,10.

Alternativas
Comentários
  • P = (30/1000) x 3 dias = 0,09

  • 1000 ---- 30

    3 ----- X

    1000*X = 3*30

    X= 90/1000 = 0,09

  • 30/1000 = 3/100 = 0,03

    0,03 x 3 = 0,09

  • A questão quer saber a probabilidade de uma folha de cheque ser devolvida E as outras duas serem aceitas.

    Probabilidade de ser aceita: 3/100

    Probabilidade de ser rejeitada: 1 - 3/100 = 97/100

    3/100 . 97/100 . 97/100 = ~0,028

    Obs.: como a folha devolvida pode ter sido a que foi recebida em qualquer um dos 3 dias, multiplica-se essa probabilidade (0,028) por 3.

    Logo, a resposta será: 0,028 x 3 = 0,084

    0,08 < 0,084 < 0,1

    CERTO

  • Gabarito: Certo.

    É uma questão de distribuição binomial. Note que nós estamos diante de um evento que apresenta apenas sucesso ou fracasso e que é repetido "n" (no caso três) vezes.

    O sucesso, no contexto, será a devolução, que tem probabilidade de 30/1000 = 0,03.

    O fracasso, no contexto, será a aceitação, que é o evento complementar do sucesso: 1 - 0,03 = 0,97.

    Nós temos três eventos e queremos que apenas um dele seja o devolução. Então, nós temos um C(3,1) com as possibilidades possíveis, pois é possível a devolução ocorra em qualquer uma das três situações.

    Portanto:

    P(X=1) = C(3,1) x (0,03)¹ x (0,97)²

    P(X=1) = 0,084.

    De fato, superior a 0,08 e inferior a 0,10.

    Bons estudos!


ID
510547
Banca
CESPE / CEBRASPE
Órgão
MPE-TO
Ano
2006
Provas
Disciplina
Raciocínio Lógico
Assuntos

Em cada um dos itens subseqüentes, é apresentada uma situação
hipotética, seguida de uma assertiva a ser julgada.

Em um lote de 20 processos, há 3 processos cujos pareceres estão errados. Aleatoriamente, um após o outro, 3 processos foram retirados desse lote. Nesse caso, a probabilidade de que os 3 processos retirados não estejam com os pareceres errados é superior a 0,6.

Alternativas
Comentários
  • Se de 20 processos, existem 3 com pareces errados, a probabilidade de tirar 3 consecutivos ( com pareceres corretos) é calculada da seguinte forma:

    17/20 x 16/19 x 15/18 = (aprox) 0.59

    Gabarito: Errado
  • 0,596 ~ 0,60

    errado

  • Total de possibilidades de retiradas tomadas 3 a 3 = C20,3 =1.140

    Possibilidades de sucesso (retirar 3 processos q/ não estejam com os pareceres errados) C 17,3= 680

    P = Possibilidades de sucesso / Espaço amostral  =  680/1.140 = ,0596

  • ERRADO

    20 processos: 17 pareceres certos ; 3 pareceres errados.

    Três processos retirados aleatoriamente, qual a probabilidade de estarem com os pareceres corretos?

    Probabilidade = casos favoráveis / total de casos

    17/20 x 16/19 x 15/18 = 59,6%


ID
516151
Banca
FUMARC
Órgão
BDMG
Ano
2011
Provas
Disciplina
Raciocínio Lógico
Assuntos

Em um jogo, dois jogadores fazem uma aposta. Cada um vai lançar duas moedas. Aquele que obtiver um par com faces iguais, isto é, coroa-coroa ou cara-cara, será o vencedor. Evidentemente, há possibilidade de empate, quando ambos os jogadores, cada um em seu lançamento, obtiver faces iguais nas duas moedas lançadas. Há também possibilidade de não haver vencedor, ou seja, quando ambos obtiverem faces distintas no lançamento das moedas.

É CORRETO afrmar que a probabilidade de não haver vencedores é de

Alternativas
Comentários
  • As possibilidades para cada jogador é:

    (cara / cara), (cara / coroa), (coroa / cara) e (coroa / coroa).

    Ou seja, uma probabilidade de 1/2 para ele não ganhar.

    Para que nenhum dos dois ganhe, é necessário que o primeiro não ganhe e o segundo não ganhe. Portanto:

    P = (1/2) * (1/2) = 1/4

    até mais!

    ;)
  • Gabarito letra c)

    Outra forma de se explicar a resolução é:

    Que para cada possibilidade de um não ser vencedor (Caindo suas moedas [cara/coroa ou coroa/cara]) 2/4=1/2, existem iguias possibilidades do outro jogador obter em suas jogadas (cara/coroa ou coroa/cara) 2/4=1/2, assim 1/2 X 1/2= 1/4.

    Forte abraço e bons estudos a todos!
  • Desculpem-me. Mas o gabarito está errado.

    Aparentemente o enunciado parece fazer uma diferenciação entre "empate", "haver ganhador" e "não haver ganhador". Ora, em qualquer jogo ou há ganhador ou não há ganhador. Empate é uma espécie do gênero "não há ganhador" (todos poderia ser desclassificados, e.g.).  

                    Veja o espaço amostral (S) do “outcome” deste jogo:
                    Onde K=cara, C=Coroa e (ab,cd) os resultados possíveis das jogadas do 1º e 2º jogador:

    S={(kk,kk);(kk,kc);(kk,ck);(kk,cc);(ck,kk);(ck,kc);(ck,ck);(ck,cc);(kc,kk);(kc,kc);(kc,ck);(kc,cc);(cc,kk);(cc,kc);(cc,ck);(cc,cc)}

                    Perceba que em cada possibilidade de “outcome” do 1º jogador, apenas em metade das possibilidades para o 2º jogador ele ganha, na outra metade o jogo termina empatado:
                    Por exemplo, imagine que o 1º jogador tirou 2 caras (KK), então ele pode:
    1. (kk,kk) => empatar (não há ganhador)
    2. (kk,kc) => ganhar
    3. (kk,ck) => ganhar
    4. (kk,cc) => empatar (não há ganhador)
                    E assim por diante...
                    Ao visualizar o espaço amostral, percebemos que não há ganhador quando o “outcome” dos 2 jogadores forem diferentes ou quando os 2 forem iguais! Isto acontece em metade dos casos.
                    Não haver ganhador ={(kk,kk);(kk,cc);(ck,kc);(ck,ck);(kc,kc);(kc,ck);(cc,kk);(cc,cc)}
                    Probabilidade de não haver = Eventos (não ganhador)/Espaço Amostral = 8/16= ½

    Alternativa: A
  • Marcelo,

    seu comentário faz a gente ficar mais atento ao enunciado da questão.
    Mas acho que neste caso não é bem assim que funciona pois ele especificou:

    Em um jogo, dois jogadores fazem uma aposta. Cada um vai lançar duas moedas. Aquele que obtiver um par com faces iguais, isto é, coroa-coroa ou cara-cara, será o vencedor. Evidentemente, há possibilidade de empate, quando ambos os jogadores, cada um em seu lançamento, obtiver faces iguais nas duas moedas lançadas. Há também possibilidade de não haver vencedor, ou seja, quando ambos obtiverem faces distintas no lançamento das moedas.

    É CORRETO afrmar que a probabilidade de não haver vencedores é de
     

    Creio se que se a banca quisesse que o canditado extrapolasse o raciocínio como você fez, não teria entrado em detalhes como entrou. 

    Mas o que a banca considerou correto, afinal?!
     

    PS: Por que um empate não é considerado como duas vitórias? Em jogos infantis é assim... Na Xuxa quando empatava, todo mundo ganhava brinquedo! rsrs E no mundial de Xangai, dois franceses ganharam duas medalhas de ouro no nado costas na mesma prova... http://esporte.ig.com.br/maisesportes/natacaoeaquaticos/dupla+francesa+faz+historia+e+divide+ouro+no+mundial+de+xangai/n1597100539321.html
  • Levando ao pé da letra o enunciado, eu resolvi de outra forma.
    Probabilidades do jogo: (Supomos jogador A e jogador B).

    (Vitoria de A)    (Vitoria de B)   (Empate)    (Não tem vencedor)

    Percebemos que temos 4 alternativas de resultado. O enunciado pede quando NÃO TEM VENCEDOR, que é so uma das 4 possibilidades, então:
    1/4.

    Isto porque a chance de qualquer 1 dos resultados acontecer é igual.
  • Lhugo T Jr, com certeza melhor solução, direta, sem cálculos e rápida!
  • Para cada jogador, existem 4 possibilidades de resultado: CACA COCA COCO CACO.
    Combinando cada um desses resultados com os resultados do outro jogador, temos 4 x 4 = 16
    Portanto o jogo tem 16 possibilidades diferentes.

    Para o resposta, nós queremos os seguintes resultados: COCA com COCA, CACO com CACO, COCA com CACO ou CACO com COCA., ou seja "quando ambos obtiverem faces distintas".

    Portanto são 4/16 = 1/4.
  • Possibilidade para cada jogador: cara cara, cara coroa, coroa cara e coroa coroa. 

    Dessa forma, cada jogador possui 2/4 de possibilidades de ganhar. Simplificando, 1/2.

    Para o jogador 1 e o jogador 2 ganharem, será necessário multiplicar 1/2 vezes 1/2.

    O resultado é 1/4. Alternativa "c".

  • Eu entendi a explicação da Luciana só que penso que faltou considerar quando os dois acertam... Fiquei mto na dúvida. Consigo enxergar apenas 1/2. Acho que caberia recurso empate tambem não existe vencedores...


ID
568921
Banca
CESGRANRIO
Órgão
Petrobras
Ano
2010
Provas
Disciplina
Raciocínio Lógico
Assuntos

Uma prova consta de 35 questões do tipo múltipla escolha, com 5 opções cada uma, onde apenas uma opção é verdadeira. Um candidato que não sabe resolver nenhuma das questões vai respondê-las aleatoriamente. Ele sabe que as respostas certas das 35 questões estão distribuídas igualmente entre as opções A,B,C,D e E, e resolve marcar suas respostas seguindo esse critério: escolherá aleatoriamente 7 questões para marcar a opção A, outras 7 para a B, e assim sucessivamente. A probabilidade de ele acertar todas as questões é

Alternativas
Comentários
  • Sabendo que cada resposta (A, B, C, D, e E) será repetida 7 vezes, temos que o total de possibilidades de resposta será dado por "Permutação com elementos repetidos":

    35! / (7! .7! .7! .7! .7!) = 35! / 7!^5

    Como só há uma possibilidade de todas as respostas estarem certas:

    P= 1/ (35! / 7!^5)

    P= 7!^5 / 35!

  • Gab: D

    Não consegui resolver e procuro explicação sem fórmulas... Daí encontrei esta daqui neste link: link: https://www.aprovaconcursos.com.br/questoes-de-concurso/questao/224610

    Possibilidades com a letra A:

    7/35 . 6/34 . 5/33...1/29

    Possibilidades com a letra B:

    7/28 . 6/27 . 5/26...1/22

    Letra C:

    7/21...1/15

    Letra D:

    7/14...1/8

    Letra E:

    7/7...1/1

    No final, multiplicando tudo fica (7!).(7!).(7!).(7!).(7!) / 35! = [(7!)^5]/35! Letra D

  • A A A A A A A B B B B B B B C C C C C C C D D D D D D D E E E E E E E

                         1                         

                         35!          

            7! 7! 7! 7! 7! 7! 7!